Вы находитесь на странице: 1из 257

REPORT

ON
CAMPUS QUESTIONS

To

Prof. Gollapudi Subba Rao


TPO Officer
GIT
GITAM UNIVERSITY

By

ELECTRICAL & ELECTRONICS ENGINEERING


Gitam Institute of Technology,
GITAM UNIVERSITY
TCS:
QUANT( 38 questions - 40 min )

1) If log 0.317=0.3332 and log 0.318=0.3364 then find log 0.319 =


Sol: Given log 0.317=0.3332 and log 0.318=0.3364
Then
Log 0.319=log0.318+ (log0.318-log0.317)
=0.3396

2) A box of 150 packets consists of 1kg packets and 2kg packets. Total weight of box is
264kg. How many 2kg packets are there?
Sol: Given x= 2 kg Packs
y= 1 kg packs

=> x + y = 150 .......... Eqn 1


=> 2x + y = 264 .......... Eqn 2
On solving these two equations
x = 114
By using equation 1
114 + y = 150
=> y = 36

=>Number of 2 kg Packs = 114.

3) My flight takes of at 2am from a place at 18N 10E and landed 10 Hrs later at a place with
coordinates 36N70W. What is the local time when my plane landed?
a) 6:00 am b) 6:40am c) 7:40 d) 7:00 e) 8:00
Sol: (Hint: Every 1 deg longitude is equal to 4 minutes. If west to east add time else subtract
time)
Ans: 8:00
4) A Flight takes off at 2 A.M from northeast direction and travels for 11 hours to reach the
destination, which is in northwest direction. Given the latitude and longitude of source and
destination. Find the local time of destination when the flight reaches there?
Ans: 7 AM (or) 1 PM

5) A moves 3 kms east from his starting point. He then travels 5 kms north. From that point he
moves 8 kms to the east. How far is A from his starting point?
Ans: 13 kms

6) Aeroplane is flying at a particular angle and latitude, after some time latitude is given. (8 hrs
later), u r asked to find the local time of the place.

7) An Aeroplane starts from A (SOME LATITUDE IS GIVEN ACCORDING TO PLACE).At 2


AM local time to B (SOME LATITUDE). Traveling time is 10 Hours. What is the local time of
B when it reaches B?

8) A plane moves from 9°N40°E to 9°N40°W. If the plane starts at 10 am and takes 8 hours to
reach the destination, find the local arrival time.
Sol: Since it is moving from east to west longitude we need to add both
Ie, 40+40=80
Multiply the ans by 4
=>80*4=320min
Convert this min to hours i.e., 5hrs 33min
It takes 8hrs totally. So 8-5hr 30 min=2hr 30min
So the ans is 10am+2hr 30 min
Ans: 12:30 it will reach

9) The size of the bucket is N kb. The bucket fills at the rate of 0.1 kb per millisecond. A
programmer sends a program to receiver. There it waits for 10 milliseconds. And response will
be back to programmer in 20 milliseconds. How much time the program takes to get a response
back to the programmer, after it is sent?
Sol: The time being taken to fill the bucket.
After reaching program it waits there for 10ms and back to the programmer in
20 ms. then total time to get the response is
20ms +10 ms=30ms
Ans: 30ms

10) A file is transferred from one location to another in ‘buckets’. The size of the bucket is 10
kilobytes. Eh bucket gets filled at the rate of 0.0001 kilobytes per millisecond. The transmission
time from sender to receiver is 10 milliseconds per bucket. After the receipt of the bucket the
receiver sends an acknowledgement that reaches sender in 100 milliseconds. Assuming no error
during transmission, write a formula to calculate the time taken in seconds to successfully
complete the transfer of a file of size N kilobytes.
Ans: (n/1000)*(n/10)*10+ (n/100).... (Not 100% sure)

11)A fisherman's day is rated as good if he catches 9 fishes ,fair if 7 fishes and bad if 5 fishes .He
catches 53 fishes in a week n had all good, fair n bad days in the week. So how many good, fair n
bad days did the fisher man had in the week.
Sol:
good days means --- 9 fishes so 53/9=4(remainder=17) if u assume 5 then there is no chance for
bad days.
fair days means ----- 7 fishes so remaining 17 --- 17/7=1(remainder=10) if u assume 2 then there
is no chance for bad days.
bad days means -------5 fishes so remaining 10---10/5=2days.

4*9=36
7*1=7
2*5=10
36+7+10=53...
Ans: 4 good, 1 fair, 2bad. ==== total 7 days.

12) x+y+z=7--------- eq1


9*x+7*y+5*z=53 -------eq2
Sol:
Multiply eq 1 by 9,
9*x+9*y+9*z=35 -------------eq3
From eq2 and eq3
2*y+4*z=10-----eq4
Since all x, y and z are integer i should put a integer value of y such that z sud be integer
in eq 4.....And there will be two value y=1 or 3 then z = 2 or 1 from eq 4
For first y=1,z=2 then from eq1 x= 4
So 9*4+1*7+2*5=53.... Satisfied
Now for second y=3 z=1 then from eq1 x=3
So 9*3+3*7+1*5=53 ......satisfied
So finally there are two solution of this question

Ans:(x,y,z)=(4,1,2) and (3,3,1)...

13) Y catches 5 times more fishes than X. If total number of fishes caught by X and Y is 42, then
number of fishes caught by X?
Sol: let no. of fish x catches=p
No. caught by y =r
r=5p.
Given r+p=42
Then p=7, r=35
14) Three companies are working independently and receiving the savings 20%, 30%, 40%. If
the companies work combine, what will be their net savings?
Sol: Suppose total income is 100
So amount x is getting is 80
y is 70
z =60
Total=210
But total money is 300
300-210=90
So they are getting 90 rs less
90 is 30% of 300 so they r getting 30% discount

15) The ratio of incomes of C and D is 3:4.the ratio of their expenditures is 4:5.Find the ratio of
their savings if the savings of C is one fourths of his income?
Sol: incomes: 3:4
Expenditures: 4:5
3x-4y=1/4(3x)
12x-16y=3x
9x=16y
y=9x/16
(3x-4(9x/16))/ ((4x-5(9x/16)))
Ans: 12/19

16)If A can copy 50 pages in 10 hours and A and B together can copy 70 pages in 10 hours, how
much time does B takes to copy 26 pages?
Sol: A can copy 50 pages in 10 hrs.
=>A can copy 5 pages in 1hr. (50/10)
Now A & B can copy 70 pages in 10hrs.
Thus, B can copy 90 pages in 10 hrs. [Eqn. is (50+x)/2=70, where x--> no. of pages B can copy
in 10 hrs.]
So, B can copy 9 pages in 1hr.
Therefore, to copy 26 pages B will need almost 3hrs.
Since in 3hrs B can copy 27 pages

17) A can copy 50 papers in 10 hours while both A & B can copy 70 papers in 10 hours. Then
for how many hours required for B to copy 26 papers?

ANS: 13

18) A is twice efficient than B. A and B can both work together to complete a work in 7 days.
Then find in how many days A alone can complete the work?
ANS: 10.5 (11)

19) A finish the work in 10 days. B is 60% efficient than A. So how days does B take to finish
the work?
Ans: 100/6 (4 days)

20) A finishes the work in 10 days & B in 8 days individually. If A works for only 6 days then
how many days should B work to complete A's work?
Ans: 3.2 days (4 days)

21) A man, a woman, and a child can do a piece of work in 6 days. Man only can do it in 24
days. Woman can do it in 16 days and in how many days child can do the same work?
Ans: 16

22) If 20 men take 15 days to complete a job, in how many days can 25 men finish that work?
Ans. 12 days

23) One fast typist type some matter in 2hr and another slow typist type the same matter in 3hr. if
both do combine in how much time they will finish.
Ans: 1hr 12min

24) A man shapes 3 cardboards in 50 minutes, how many types of cardboard does he shape in 5
hours?
Ans: 18cardboards

25) A work is done by two people in 24 min. one of them can do this work a lonely in 40 min.
how much time required to do the same work for the second person.
Sol: (A+B) can do the work in = 1/24 min.
A alone can do the same work in = 1/40 min.
B alone can do the same work in = (A+B)’s – A’s = 1/24 – 1/40 = 1/60
=> B can do the same work in = 60 min
Ans: 60 min

26) A can do a piece of work in 20 days, which B can do in 12 days. In 9 days B does ¾ of the
work. How many days will A take to finish the remaining work?

27) Anand finishes a work in 7 days; Bittu finishes the same job in 8 days and Chandu in 6 days.
They take turns to finish the work. Anand on the first day, Bittu on the second and Chandu on
the third day and then Anand again and so on. On which day will the work get over?
A) 3rd b) 6th c) 9th d) 7th

28) 3 men finish painting a wall in 8 days. Four boys do the same job in 7 days. In how many
days will 2 men and 2 boys working together paint two such walls of the same size?

A) 6 6/13 days
B) 3 3/13 days
C) 9 2/5 days
D) 12 12/13 days

29) what's the answer for that?


A, B and C are 8 bit no's. They are as follows:
A -> 1 1 0 0 0 1 0 1
B -> 0 0 1 1 0 0 1 1
C -> 0 0 1 1 1 0 1 0 (- =minus, u=union)
Find ((A - C) u B) =?
Sol: We have to find (A-C) U B
To find A-C, We will find 2's compliment of C and them add it with A,
That will give us (A-C)
2's compliment of C=1's compliment of C+1
=11000101+1=11000110
A-C=11000101+11000110
=10001001
Now (A-C) U B is .OR. Logic operation on (A-C) and B
10001001 .OR. 00110011
The answer is = 10111011,
Whose decimal equivalent is 187.

30) A = 10010001
B = 01101010
C = 10010110
(AuB)nC =? [(A union B) intersection C =?]

31) A =0 0 0 0 1 1 1 1
B =0 0 1 1 0 0 1 1
C =0 1 0 1 0 1 0 1
( A U B ) n C Find the fourth row, having the bit pattern as an integer in an 8-bit computer, and
express the answer in its decimal value.
Ans: 29

32) A, B and C are 8 bit nos. They are as follows:


A11011011
B01111010
C01101101
Find ( (A-B) u C )=?
Hint: 109 A-B is {A} - {A n B}
Ans: 0 1 1 1 1 1 1 1 (DB)

33) If A, B and C are the mechanisms used separately to reduce the wastage of fuel by 30%, 20%
and 10%. What will be the fuel economy if they were used combined.
Ans: 20%

34) In the class of 40 students, 30 speak Hindi and 20 speak English. What is the lowest possible
number of students who speak both the languages?
(a) 5 (b) 20 (c) 15 (d) 10 (e) 30

35) In a two-dimensional array, X (9, 7), with each element occupying 4 bytes of memory,
with the address of the first element X (1, 1) is 3000, find the address of
X (8, 5).
Sol: [HINT~ Formula=Base Add + Byte reqd {N (i-1) + (j-1)}
Where,
Base Add=3000;
Byte reqd=4;
N=no of columns in array=7;
i=8; j=5;
IN ROW MAJOR ORDER]
Ans: 3212
36) If the vertex (5, 7) is placed in the memory. First vertex (1, 1)’s address is 1245 and then
address of (5, 7) is ----------
Ans: 1279

37) A 2D array is declared as A [9, 7] and each element requires 2 byte. If A [1, 1] is stored in
3000. Find the memory of A [8, 5]?
Ans: 3106

38) One circular array is given (means the memory allocation takes place like a circular fashion)
dimension (9X7) .starting address is 3000.find the address of (2, 3)
Ans: 555
39) The size of a program is N. And the memory occupied by the program is given by M =
square root of 100N. If the size of the program is increased by 1% then how much memory now
occupied?
Sol: N is increased by 1%
Therefore new value of N=N + (N/100)
=101N/100
M=sqrt (100 * (101N/100))
Hence, we get
M=sqrt (101 * N)
Ans: 0. 5 %( =SQRT 101N)

40) A bus started from bus stand at 8.00a m and after 30 min staying at destination, it returned
back to the bus stand. The destination is 27 miles from the bus stand. The speed of the bus 50
percent fast speed. At what time it retur4ns to the bus stand.
Sol: (this is the step by step solution :)
A bus cover 27 mile with 18 mph in =27/18= 1 hour 30 min.
And it wait at stand =30 min.
After this speed of return increase by 50% so 50%of 18 mph=9mph
Total speed of returning=18+9=27
Then in return it take 27/27=1 hour
Then total time in journey=1+1:30+00:30 =3 hour
So it will come at 8+3 hour=11 a.m.
So Ans==11 a.m
41) A Flight takes off at 2 A.M from northeast direction and travels for 11 hours to reach the
destination which is in North West direction. Given the latitude and longitude of source and
destination. Find the local time of destination when the flight reaches there?
Ans: 7 AM or 1.00 PM
42) My flight takes of at 2am from a place at 18N 10E and landed 10 Hrs later at a place with
coordinates 36N70W. What is the local time when my plane landed?
a) 6:00 am b) 6:40am c) 7:40 d) 7:00 e) 8:00
(Hint: Every 1 deg longitude is equal to 4 minutes. If west to east add time else subtract time)
Ans: 8:00

43) A moves 3 kms east from his starting point. He then travels 5 kms north. From that point he
moves 8 kms to the east. How far is A from his starting point?
Ans: 13 kms

44) A plane moves from 9°N40°E to 9°N40°W. If the plane starts at 10 am and takes 8 hours to
reach the destination, find the local arrival time.

45) In Madras , temperature at noon varies according to -t^2/2 + 8t + 3, where t is elapsed time.
Find how much temperature more or less in 4pm to 9pm. (May be we can solve it by Definite
Integration. Check any way}
Ans: at 9 pm 7.5 more or 385.8 (DB)

46) For Temperature a function is given according to time: ((t**2)/6) + 4t +12 what is the
temperature rise or fall between 4.AM TO 9 AM
Sol: In equation first put t=9,
We will get 34.5........................... (1)
Now put t=4,
We will get 27.............................. (2)
So Ans=34.5-27
=7.5
47) For Temperature a function is given according to time: ((t**2)/6) + 4t +12 what is the
temperature rise or fall between 5 PM to 8 PM

48) Low temperature at the night in a city is 1/3 more than 1/2 high as higher temperature in a
day. Sum of the low tem. And highest temp is 100 degrees. Then what is the low temp?
Sol: Let highest temp be x
So low temp=1/3 of x of 1/2 of x plus x/2 i.e. x/6+x/2
Total temp=x+x/6+x/2=100
Therefore, x=60
Lowest temp is 40
Ans :( 40 deg.)

49) A person had to multiply two numbers. Instead of multiplying by 35, he multiplied by 53and
the product went up by 540. What was the raised product?
a) 780
b) 1040
c) 1590
d) 1720
Sol: x*53-x*35=540=> x=30 therefore, 53*30=1590
Ans: 1590

50) How many positive integer solutions does the equation 2x+3y = 100 have?
a) 50
b) 33
c) 16
d) 35
Sol: Given 2x+3y=100, take l.c.m of 'x' coeff and 'y' coeff i.e. l.c.m of 2,3 ==6then divide 100
with 6 , which turns out 16 hence answer is 16short cut formula--- constant / (l.cm of x coeff and
y coeff)
51) The total expense of a boarding house is partly fixed and partly variable with the number of
boarders. The charge is Rs.70 per head when there are 25 boarders and Rs.60 when there are 50
boarders. Find the charge per head when there are 100 boarders.
a) 65
b) 55
c) 50
d) 45
Sol: let a = fixed cost and
k = variable cost and n = number of boarders
Total cost when 25 boarders c = 25*70 = 1750 i.e. 1750 = a + 25k
Total cost when 50 boarders c = 50*60 = 3000 i.e. 3000 = a + 50k
Solving above 2 eqns, 3000-1750 = 25k i.e. 1250 = 25k i.e. k = 50
Therefore, substituting this value of k in either of above 2 eqns we get
a = 500 (a = 3000-50*50 = 500 or a = 1750 - 25*50 = 500)
So total cost when 100 boarders = c = a + 100k = 500 + 100*50 = 5500
So cost per head = 5500/100 = 55

52) Amal bought 5 pens, 7 pencils and 4 erasers. Rajan bought 6 pens, 8 erasers and 14 pencils
for an amount which was half more than what Amal had paid. What % of the total amount paid
by Amal was paid for pens?
a) 37.5%
b) 62.5%
c) 50%
d) None of these
Sol: Let, 5 pens + 7 pencils + 4 erasers = x rupees
So 10 pens + 14 pencils + 8 erasers = 2*x rupees
Also mentioned, 6 pens + 14 pencils + 8 erasers = 1.5*x rupees
So (10-6) = 4 pens = (2-1.5) x rupees
So 4 pens = 0.5x rupees => 8 pens = x rupees
So 5 pens = 5x/8 rupees = 5/8 of total (note x rupees is total amt paid by
Amal) i.e. 5/8 = 500/8% = 62.5%
Ans: 62.5%

53) I lost Rs.68 in two races. My second race loss is Rs.6 more than the first race. My friend lost
Rs.4 more than me in the second race. What is the amount lost by my friend in the second race?
Sol: x + x+6 = rs 68
2x + 6 = 68
2x = 68-6
2x = 62
x=31
x is the amt lost in I race
x+ 6 = 31+6=37 is lost in second race
Then my friend lost 37 + 4 = 41 Rs
Ans: 41 Rs

54) A face of the clock is divided into three parts. First part hours total is equal to the sum of the
second and third part. What is the total of hours in the bigger part?
Sol: The clock normally has 12 hr
Three parts x, y, z
x+y+z=12
x=y+z
2x=12
x=6
So the largest part is 6 hrs
Ans: 6 hrs

55) (1- 1/6) (1-1/7).... (1- (1/ (n+4))) (1-(1/ (n+5))) = ?


Sol: Leaving the first numerator and last denominator, all the numerator and denominator will
cancelled out one another.
Ans: 5/ (n+5)

56) Ten boxes are there. Each ball weighs 100 gms. One ball is weighing 90 gms.
i) If there are 3 balls (n=3) in each box, how many times will it take to find 90 gms ball? ii)
Same question with n=10
iii) Same question with n=9
Sol: The chances are
When n=3
(i) nC1= 3C1 =3 for 10 boxes.. 10*3=30
(ii) nC1=10C1=10 for 10 boxes ....10*10=100
(iii) nC1=9C1=9 for 10 boxes.....10*9=90

57) With 4/5 full tank vehicle travels 12 miles, with 1/3 full tank how much distance travels?
Sol: 4/5 full tank= 12 mile
1 full tank= 12/ (4/5)
1/3 full tank= 12/ (4/5)*(1/3) = 5 miles
Ans: 5 miles

58) Wind flows 160 miles in 330min.for 80 miles how much time required
160 miles?
Sol: 1 mile = 330/160
80 miles= (330*80)/160=165 min.
Ans: 165 min.

59) A person was fined for exceeding the speed limit by 10mph.another person was also fined
for exceeding the same speed limit by twice the same if the second person was traveling at a
speed of 35 mph. find the speed limit
Sol :( x+10) =(x+35)/2
Solving the eqn we get x=15
Ans: 15

60) A sales person multiplied a number and get the answer is 3 instead of that number divided by
3.what is the answer he actually has to get.
Sol: Assume 1
1* 3 = 3
1*1/3=1/3
So he has to got 1/3
Ans: 1/3

61) The size of the bucket is N kb. The bucket fills at the rate of 0.1 kb per millisecond. A
programmer sends a program to receiver. There it waits for 10 milliseconds. And response will
be back to programmer in 20 milliseconds. How much time the program takes to get a response
back to the programmer, after it is sent?
Ans: 30 milliseconds

62) A person who decided to go weekend trip should not exceed 8 hours driving in a day average
speed of forward journey is 40 mph due to traffic in Sundays the return journey average speed is
30 mph. How far he can select a picnic spot.

Ans: Between 120 and 140 miles

63) Car is filled with four and half gallons of oil for full round trip. Fuel is taken 1/4 gallons
more in going than coming. What is the fuel consumed in coming up.
Sol: Let fuel consumed in coming up is x.
Thus equation is: x+1.25x=4.5
Ans: 2gallons

64) 40% employees are male if 60% of supervisors are male so for 100% is 26.4%so the
probability is …………..
Ans: 0.264

65) Gavaskar average in first 50 innings was 50. After the 51st innings his average was 51 how
many runs he made in the 51st innings
Sol: first 50 ings.- run= 50*50=2500
51st ings. - Avg 51. So total run =51*51=2601.
So run scored in that ings=2601-2500=101 runs.
Ans: 101 runs

66) Hansie made the following amounts in seven games of cricket in India : Rs.10, Rs.15, Rs.21,
Rs.12, Rs.18, Rs.19 and Rs.17 (all figures in crores of course).Find his average earnings.
Ans: Rs.16 crore

67) Average of 5 numbers is -10 sum of 3 numbers is 16, what is the average of other two
numbers?
Ans: -33

68) If A, B and C are the mechanisms used separately to reduce the wastage of fuel by 30%, 20%
and 10%. What will be the fuel economy if they were used combined.
Ans: 20%

69) In 80 coins one coin is counterfeit what is minimum number of weighing to find out
counterfeit coin
Sol: the minimum number of weightings needed is just 5.as shown below

(1) 80->30-30
(2) 15-15
(3) 7-7
(4) 3-3
(5) 1-1
Ans: 5.

70) 2 oranges, 3 bananas and 4 apples cost Rs.15. 3 oranges, 2 bananas, and 1 apple costs Rs 10.
What is the cost of 3 oranges, 3 bananas and 3 apples?
Sol: 2x+3y+4z=15
3x+2y+z=10
Adding
5x+5y+5z=25
x+y+z=5 that is for 1 orange, 1 banana and 1 apple requires 5Rs.
So for 3 orange, 3 bananas and 3 apples require 15Rs.
i.e. 3x+3y+3z=15
Ans: 15

71) In 8*8 chess board what is the total number of squares refers odele discovered that there are
204 squares on the board .We found that you would add the different squares
= 1 + 4 + 9 + 16+ 25 + 36 + 49 + 64. =204

Also in 3*3 tic tac toe board what is the total no of squares
Ans: 14 i.e. 9+4(bigger ones) +1 (biggest one)

If you get 100*100 board just use the formula the formula for the sum of the first n perfect
squares is

n x (n + 1) x (2n + 1)
______________________
6

72) One fast typist type some matter in 2hr and another slow typist type the same matter in 3hr.
If both do combine in how much time they will finish.
Sol: Faster one can do 1/2 of work in one hour slower one can do 1/3 of work in one hour both
they do (1/2+1/3=5/6) the work in one hour. So work will b finished in 6/5=1.2 hour
i e 1 hour 12 min.
Ans: 1 hour 12 min.

73)If Rs20/- is available to pay for typing a research report & typist A produces 42 pages and
typist B produces 28 pages. How much should typist A receive?
Sol: Find 42 % of 20 rs with respect to 70 (i.e. 28 + 42)
==> (42 * 20)/70 ==> 12 Rs
Ans: 12 Rs

74) In some game 139 members have participated every time one fellow will get bye what is the
number of matches to choose the champion to be held?
Ans: 138 matches
(Explanation: since one player gets a bye in each round, he will reach the finals of the
tournament without playing a match.
Therefore 137 matches should be played to determine the second finalist from the remaining 138
players (excluding the 1st player)
Therefore to determine the winner 138 matches should be played.)

75) ONE RECTANGULAR PLATE WITH LENGTH 8INCHES, BREADTH 11 INCHES AND
2 INCHES THICKNESS IS THERE.WHAT IS THE LENGTH OF THE CIRCULAR ROD
WITH DIAMETER 8 INCHES AND EQUAL TO VOLUME OF RECTANGULAR PLATE?
Sol: Vol. of rect. plate= 8*11*2=176
Area of rod= (22/7)*(8/2)*(8/2) = (352/7)
Vol. of rod=area*length=vol. of plate
So length of rod= vol of plate/area=176/ (352/7) =3.5
Ans: 3.5
76) One tank will fill in 6 minutes at the rate of 3cu ft /min, length of tank is 4 ft and the width is
1/2 of length, what is the depth of the tank?
Ans: 3 ft 7.5 inches

77) A power unit is there by the bank of the river of 750 meters width. A cable is made from
power unit to power a plant opposite to that of the river and 1500mts away from the power unit.
The cost of the cable below water is Rs. 15/- per meter and cost of cable on the bank is Rs.12/-
per meter. Find the total of laying the cable.
Ans: 1000 (24725 - cost) 20250
Ans: Rs. 22,500 (hint: the plant is on the other side of the plant i.e. it is not on the same
side as the river)
78) The cost of one pencil, two pens and four erasers is Rs.22 while the cost of five pencils, four
pens and two erasers is Rs.32.How much will three pencils, three pens and three erasers cost?
Sol :( let x b pencil, y b pen and z b eraser... u get x+2y+4z=22 and 5x+4y+2z=32 add
6x+6y+6z=54 div by 2 you get 27)
Ans: 27

79) A man has to get air-mail. He starts to go to airport on his motorbike. Plane comes early and
the mail is sent by a horse-cart. The man meets the cart in the middle after half an hour. He takes
the mail and returns back, by doing so, he saves twenty minutes. How early did the plane arrive?
Sol: Assume he started at 1:00, so at 1:30 he met cart .He returned home at 2:00.so it took him 1
hour for the total journey. By doing this he saved 20 min. So the actual time if the plane is not
late is 1 hour and 20 min. So the actual time of plane is at 1:40.The cart traveled a time of 10 min
before it met him. So the plane is 10 min early.
Ans: 10 min
80) Ram singh goes to his office in the city every day from his suburban house. His driver
Mangaram drops him at the railway station in the morning and picks him up in the evening.
Every evening Ram singh reaches the station at 5 o'clock. Mangaram also reaches at the same
time. One day Ram singh started early from his office and came to the station at 4 o'clock. Not
wanting to wait for the car he starts walking home. Mangaram starts at normal time, picks him
up on the way and takes him back house, half an hour early. How much time did Ram singh
walked?

81) 2 trees are there. One grows at 3/5 of the other. In 4 years total growth of the trees is 8 ft.
what growth will smaller tree have in 2 years.
Sol: THE BIG TREE GROWS 8FT IN 4 YEARS=>THE BIG TREE GROWS 4FT IN 2
YEARS.WHEN WE DIVIDE 4FT/5=.8*3=>2.4
4 (x+ (3/5) x) =88x/5=2x=5/4
After 2 years x= (3/5)*(5/4)*2 =1.5 (less than 2 feet)
82) There is a six digit code. Its first two digits, multiplied by 3 gives all ones. And the next two
digits multiplied by 6 give all twos. Remaining two digits multiplied by 9 gives all threes. Then
what is the code?
Sol: Assume the digit xx xx xx (six digits)
First Two digit xx * 3=111
xx =111/3=37
(First two digits of 1 is not divisible by 3 so we can use 111)
Second Two digit xx*6=222
xx=222/6=37
(First two digits of 2 is not divisible by 6 so we can use 222)
Third Two digit xx*9=333
xx=333/9=37
(First two digits of 3 is not divisible by 9 so we can use 333)

83) There are 4 balls and 4 boxes of colors yellow, pink, red and green. Red ball is in a box
whose color is same as that of the ball in a yellow box. Red box has green ball. In which box you
find the yellow ball?
Sol: Yellow box can have either of pink/yellow balls.
if we put a yellow ball in "yellow" box then it would imply that "yellow" is also the color of the
box which has the red ball(because according 2 d question,d box of the red ball n the ball in the
yellow box have same color)
Thus this possibility is ruled out...
Therefore the ball in yellow box must be pink, hence the color of box contain in red ball is also
pink....
=>the box color left out is "green", which is allotted to the only box left, the one which has
yellow ball.
Ans: green
84) A bag contains 20 yellow balls, 10 green balls, 5 white balls, 8 black balls, and 1 red ball.
How many minimum balls one should pick out so that to make sure the he gets at least 2 balls of
same color.
Sol: suppose he picks 5 balls of all different colors then when he picks up the sixth one, it must
match any on of the previously drawn ball color. Thus he must pick 6 balls
Ans: he should pick 6 balls totally.

85) WHAT IS THE NUMBER OF ZEROS AT THE END OF THE PRODUCT OF THE
NUMBERS FROM 1 TO 100?
Sol: For every 5 in unit place one zero is added
so between 1 to 100 there are 10 nos like 5,15,25,..,95 which has 5 in unit place.
Similarly for every no divisible by 10 one zero is added in the answer so between 1 to 100 , 11
zeros are added
For 25, 50, 75 3 extra zeros are added
So total no of zeros are 10+11+3=24

86) There are two numbers in the ratio 8:9. If the smaller of the two numbers is increased by 12
and the larger number is reduced by 19 thee the ratio of the two numbers is 5:9.
Find the larger number?
Sol: 8x: 9x initially
8x+ 12: 9x - 19 = 5x: 9x
8x+12 = 5x
-> x = 4
9x = 36 (NOT SURE ABOUT THE ANSWER)

87) There are three different boxes A, B and C. Difference between weights of A and B is 3 kgs.
And between B and C is 5 kgs. Then what is the maximum sum of the differences of all possible
combinations when two boxes are taken each time
Sol: A-B = 3
B-c = 5
A-c = 8 so sum of diff = 8+3+5 = 16 kgs
88) A and B are shooters and having their exam. A and B fall short of 10 and 2 shots respectively
to the qualifying mark. If each of them fired at least one shot and even by adding their total score
together, they fall short of the qualifying mark, what is the qualifying mark?
Sol: Because each had at least 1 shot done so 10 + 1 = 11
And 9 + 2 = 11
So the ans is 11

89) A, B, C, and D tells the following times by looking at their watches. A tells it is 3 to 12. B
tells it is 3 past 12 . C tells it is 12:2. D tells it is half a dozen too soon to 12. No two watches
show the same time. The difference between the watches is 2,3,4,5 respectively. Whose watch
shows maximum time?
Sol: A shows 11:57, B shows 12:03, C shows 12:02 and D shows 11:06 therefore,
Max time is for B

90) Falling height is proportional to square of the time. One object falls 64cm in 2sec than in
6sec from how much height the object will fall.
Sol: The falling height is proportional to the square of the time.
Now, the falling height is 64cm at 2sec
So, the proportional constant is=64/ (2*2) =16;
So, at 6sec the object fall maximum (16*6*6) cm=576cm;
Now, the object may be situated at any where.
If it is>576 only that time the object falling 576cm within 6sec .Otherwise if it is
situated<576 then it fall only that height at 6sec.

91) Last year pandit was thrice his sister's age. Next year he is only twice her age. After 5 years
what is pandit's age.
a) 2 b) 12 c) 11 d) 14
Ans: b

92) Jalia is twice older than qurban. If jalia was 4 years younger, qurban was 3 years older their
diff. between their ages is 12 years what is the sum of their ages
a) 67 b) 57 c) 36 d) none
Ans: b

93) Fathers age is 5 times his son's age. 4 years back the father was 9 times older than son. Find
the fathers' present age.
Ans. 40 years
94) Joe’s father will be twice his age 6 years from now. His mother was twice his age 2 years
before. If Joe will be 24 two years from now, what is the difference between his father's and
mother's age?
a) 4 b) 6 c) 8 d) 10

95) Anand finishes a work in 7 days; Bittu finishes the same job in 8 days and Chandu in 6 days.
They take turns to finish the work. Anand on the first day, Bittu on the second and Chandu on
the third day and then Anand again and so on. On which day will the work get over?
a) 3rd
b) 6th
c) 9th
d) 7th
In d 1st day Anand does 1/7th of total work
Similarly,
Bithu does 1/8th work in d 2nd day
Hence at the end of 3 days, work done = 1/7+1/8+1/6=73/168
Remaining work = (168-73)/168 = 95/168
Again after 6 days of work, remaining work is = (95-73)/168 = 22/168
And hence Anand completes the work on 7th day.
Ans is d) 7th day

96) If TAFJHH is coded as RBEKGI then RBDJK can be coded as ---------


Ans: qcckj
97) BFGE CODED AS CEHD THEN CODE PVHDJ
Ans: QUICK
98) Find the no. of Y‘s followed by W but that is not followed by Z.
Y W R U D D Y W Z ...............
99) If VXUPLVH is written as SURMISE, what is SHDVD ?
Ans. PEASA
(Hint: in the first word, the alphabets of the jumbled one are three alphabets
after the corresponding alphabet in the word SURMISE. S = V-3, similarly find the one for
SHDVD)

100) If DDMUQZM is coded as CENTRAL then RBDJK can be coded as -----


Ans. QCEIL
(Hint: Write both the jumbled and the coded word as a table, find the relation
between the corresponding words, i.e. C= D-1, N=M+1 & so on)

101) In the word ECONOMETRICS, if the first and second, third and forth, forth and fifth, fifth
and sixth words are interchanged up to the last letter, what would be the tenth letter from right?
Ans. word is CENOMOTEIRSC tenth word is R
102) If D_MUQZM is coded as CENTRAL then RBDJK can be coded as

103) In a certain format TUBUJPO is coded as STATION. The code of which string is
FILTER?

104) What is the code formed by reversing the First and second letters, the third and fourth
letters and so on of the string SIMULTANEOUSLY?

105) In the word ORGANISATIONAL, by reversing if the first and second the third and fourth
letters and so on of the string?

106) A power unit is there by the bank of the river of 750 meters width. A cable is made from
power unit to power a plant opposite to that of the river and 1500mts away from the power unit.
The cost of the cable below water is Rs. 15/- per meter and cost of cable on the bank is Rs.12/-
per meter. Find the total of laying the cable.
Ans: 1000 (24725 - cost) 20250
Ans: Rs. 22,500 (hint: the plant is on the other side of the plant i.e. it is not on the same
side as the river)
107) The cost of one pencil, two pens and four erasers is Rs.22 while the cost of five pencils,
four pens and two erasers is Rs.32.How much will three pencils, three pens and three erasers
cost?
Ans: 27

108) A shopkeeper bought a watch for Rs.400 and sold it for Rs.500.What is his profit
percentage?
Ans. 25%

109) What percent of 60 is 12?


Ans. 20%

110) Three men goes to a hotel to stay, the clerk says $30 per room/day so all the three plans to
stay in one room so each pays $10.After some time the clerk realizes that he made a mistake of
collecting $30 but the room cost only $25, there fore he decides to return $5 to them so he calls
the room boy and gives him $5 asking him to return. The room boy keeps $2 with him and he
returns only $3($1 for each).Now Totally all have paid $9 each($27)+room boy $2 which is
equal to $27.where did $1 go, who has made the mistake?

111) Two pencils cost 8 cents. Then 5 pencils cost?


Ans :( 20 cents)

112)Which is more economical of the following


a)2kg -- 30/- b)8kg -- 160/- c)5kg -- 80/-

113)Satish earns 240 weekly.12% of big amount + earning weekly = 540


what is the big amount
a)3200 b)3600 c)2500 d)1000
Ans: c
114) Bhanu spends 30% of his income on petrol on scooter. ¼ of the remaining on house rent
and the balance on food. If he spends Rs.300 on petrol then what is the expenditure on house
rent? a) Rs.525 b) Rs.1000 c) Rs.675 d) Rs.175
Ans: 175
115) A sporting goods store ordered an equal number of white and yellow balls. The tennis ball
company delivered 45 extra white balls, making the ratio of white balls to yellow balls 1/5: 1/6.
How many white tennis balls did the store originally order for?
a) 450 b) 270 c) 225 d) None of these
Ans: 180

116) There is a circular pizza with negligible thickness that is cut into 'x' pieces by 4 straight line
cuts. What is the maximum and minimum value of 'x' respectively?
a) 12, 6 b) 11, 6 c) 12, 5 d) 11, 5

117) Match the following:


1. Male - Boy ---> a. A type of
2. Square - Polygon ---> b. A part of
3. Roof - Building ---> c. Not a type of
4. Mushroom - Vegetables ---> d. A superset of
Ans: 1- d, 2- a, 3- b, 4- c

118) Match the following.


1. Brother - sister ---> a. Part of
2. Alsatian - dog ---> b. Sibling
3. Sentence - paragraph ---> c. Type of
4. Car - steering ---> d. Not a type of
Ans. 1-b, 2-c, 3-a, 4-d

119) Match the following


1) Scooter --------- Automobile A. A PART OF
2).Oxygen ---------- Water B. A Type of
3).Shop staff ---------- Fitters C. NOT A TYPE OF
4). Bug ---------- Reptile D. A SUPERSET OF
Ans. 1-b, 2-a, 3-d, 4-c

120) What is the largest prime number stored in a—


-----> 6 bit pattern (ANS~2^6=64, so no is 61)
------> 7 bit pattern (ANS~2^7=128, so no is 127)
-------> 8 bit pattern (ANS~2^8=256, so no is 251)
-------->9 bit pattern (ANS~2^9=512, so no is 503)

121) What is the max 3 digit Prime no?


ANS=997

122) G(0)= -1, G(1)=1, G(N)=G(N-1) - G(N-2), G(5)= ?


Ans - 2

123) G (0) =1 G (1) = -1 IF G (N) =2* (G (N-1)) – 3(G (N-2)) Then what is the value of
G (4)?

124) If f (0) =1 and f (n) = f (n-1)*n, find the value of f (4).


Ans: 24

125) If g (0) =g (1) =1 and g (n) = g (n-1) + g (n –2) find g (6);

126) What is the power of 2? a. 2068 b.2048 c.2668


Ans: 2048
127) 8 to the power of x is 32, what is the value of x?

128) Power of 4 Ans-4096

129) Which one will be the exact power of 3?


(i) 2768 (ii) 2678 (iii) 2187
130) Complete the series—
a )3,8,a,24,b,48,63 [ ANS~ a=15, b=35 ]
[HINT~DIFFERENCE IS 5, 7, 9, 11, 13, 15]
B )26,19,17,13,11, ,8,7 [ ANS=9]
[HINT~26,17,11,8 DECREASING LIKE 9,6,3 & 19,13,9,7 DECREASING
LIKE 6, 4, 2]
c)9,10,11,13,15, ,21,28 [ ANS=19 ]
[HINT~9, 11, 15, and 21 INCREASING LIKE 2, 4, 6 & 10,13,19,28 INCRESING
LIKE 3, 6, and 9]
D) 4, -5, 11, -14, 22, --- [ ANS= -27]

131) Number of faces, vertices and edges of a cube

ANS: 6,8,12

132) Find the value of—


a) @@+25-++@16, where @ denotes”square” & + denotes “square root”. [ANS=621]
b) $%$6-%$%6, where $ means “tripling” & % means “change of sign”. [ANS= -72]
c) % # % 6 + # %# 6, % means “doubling” & # mean “reciprocal”.

132) Select odd one out

1) LINUX, WINDOWS 98, SOLARIS, SMTP (ANS: SMTP)


2)MVS
3).JAVA b) LISP c) Smaltalk d)Eiffle Ans: LISP ( All other languages are OOPS)
4)1.http 2.arp 3.snmp 4.sap Ans:sap
5)1.linux 2.windows NT 3.sql server 4.Unix Ans: Sql server
6)1.SAP 2.ARP 3.WAP 4.TCP IP
7)a. Oracle b. Linux c. Ingress d. DB2
a. SMTP b. WAP c. SAP d. ARP Ans:SAP
9)a. WAP b. HTTP c. BAAN d. ARP Ans:Baan
10)a. LINUX b. UNIX c. SOLARIS d. SQL SERVER Ans:SQL SERVER
11)a. SQL b. DB2 c. SYBASE d. HTTP Ans: HTTP
12)a. Oracle b. Linux c. Ingress d. DB2 ANS:LINUX

133) Find the singularity matrix from a given set of matrices? (Hint det (A) ====0)

134) Which of the following are orthogonal pairs?

a. 3i+2j b. i+j c. 2i-3j d. -7i+j

Ans: a, c
135) (a) 2+3i (b) 1+i (c) 3-2i (d) 1-7i .Find which of the above is orthogonal.

Ans: a, c

136) Sum of slopes of 2 perpendicular st. lines is given. Find the pair of lines from the given set
of options which satisfy the above condition?

137) If Rs.1260 is divided between A, B and C in the ratio 2:3:4, what is C's share?

Ans: Rs. 560

138)A sum of money is divided among A, B and C such that for each rupee A gets, B gets
65paise and C gets 35paise. If C's share is Rs.560, the sum is …
a) 2400 b) 2800 c) 3200 d) 3800

139) Complete the series.

1) 3, 8, --, 24, --, 48, 63. Ans: 15, 35


2) Complete the series. 4, -5, 11, -14, 22, --- Ans - 27
3) SERIES: 2, 7, 24, 77, ------ (238) or (240)
4)77, 49, 36, 18,? Ans: 8 (7*7=49) (4*9=36) (3*6=1 (1*8=
5) series: 5 6 7 8 10 11 14?? Ans.15 or 18
6)15 14 12 11?? 9 8 Ans.10
7) what is the 12th term of the series 2, 5, 8 ... Ans. 35
8)58, 27, 12, x, 2, 1. Find x.
9)7, 9,13,_,27,37. Ans-19
10)2, 5, __, 19, 37, 75 Ans: 9
11) Complete the sequence 9, 10,11,13,15, __, 21, 28.

140) UNITS
1) (Momentum*Velocity)/ (Acceleration * distance) find units. Ans: mass
2) (energy * time * time)/ (mass * dist) = distance
3) (momentum * velocity)/ (force * time) = velocity
4) Find the physical quantity in units from the equation:
(Force*Distance)/ (Velocity*Velocity) Ans. Ns2/m
5) Find the physical quantity represented by

MOMENTUM *VELOCITY] / [LENGTH * ACCELERATION]?

141) Find the result of the following _expression if, M denotes modulus operation, R denotes
round-off, T denotes truncation:
M (373, 5) +R (3.4) +T (7.7) +R (5.8)

ANS: 19

142) Which of the following highest Standard deviation


a) 7, - 7, 7,-7, 7,-7 b) 7, 7, 7,7,7,7 c) -7, - 7, -7,-7,-7,-7 d) -7, 7, -7, 7,-7, 7
Ans: d

143) 232 expressed in base-5 is Ans: 1412

144) A building with height D shadow up to G. A neighbor building with what height shadows C
feet.
|----|----|----|----|----|----|----|
ABCDEFGH
Sol: B Ft. or CD/G

145) In a fraction, if 1 is added to both the numerator at the denominator, the fraction becomes
1/2. If numerator is subtracted from the denominator, the fraction becomes 3/4. Find the
fraction.
Ans. 3/7
146) The sum of the digits of a two digit number is 8. When 18 is added to the number, the digits
are reversed. Find the number?
Ans. 35

147) What number should be added to or subtracted from each term of the ratio 17 : 24 so that it
becomes equal to 1 : 2.
Ans. 10 should be subtracted

Verbal: Synonyms:
Brim: edge rim lip
Renounce: give up, reject,
Caprice: whim, impulse ,fancy
Brash: HASTY , RASH
Limber: supple, nimble, flexible
Admonish: reprove, caution, warn about
Tranquil: calm , still, relaxing, quiet
Moribund: declining, waning, past it
Mundane:ordinary, dull, everyday
1. Seedy :sordid, sleazy,
2. Buxom
3. fallible: imperfect,mortal,unsound
4. Misanthrope
verbal: chide
Doltish
Protract
Mawkish
disgeneous,
Repugnance,
Vouchsafe....
Antonyms (This came mostly from GRE Barron's Antonyms exercise)
Encumber
Opacity
Bilk
Luminary
Spur
Stupefy
Cacophonous
Delineate
decorum
equanimity: agitation, luster, stirring

Accenture:

Section 1 -Verbal Ability


No. of Questions: 20
Duration in Minutes: 20
Directions for Questions 1-3:
Choose the option which will correctly fill the blank.
1) I am writing to enquire _________the possibility of hiring a conference room at the hotel on
the 2nd of September.
A) Of B) About C) Into D) After
2) _________ having her lunch, she stood - the tree and waited _______ him.
A) With, below, for
B) After, under, for
C) Inside, further, to
D) About, across, into

3) The microscopic animals are the primary food for larval cod and their decline has meant that
fewer fish are making it to adulthood to be caught_________ trawlermen.
A) In B) Into C) By D).With

Directions for Questions 4-6:

Choose the word nearest in meaning to the word in ITALICS from the given options.

4) The jacket is impervious to water.


A) Dirty B) Pure C) Impenetrable D) Favorable
5) Chandan was chagrined with the continuous disruption of the power supply to his home.
A) Delighted B) Creation C) Peeved D) Security
6) The latest ordinance issued by the government has provided the bank with two options.
A) Decision B) Law C) Opinion D) Verdict

Directions for Questions 7-10:

Choose the answer option which will correctly fill the blank.
7)_________ great writer is convinced that whatever he says is not an echo or imitation of what
others have said.
A) An
C)A
B) The
D) No article required
8) ________ Reserve Bank of India directed banks to closely watch _______spending through
International Debit Cards.
A)A,the B) The, the C) The, a . D) .-\n, the
9) The officer received _____ official letter from _____ Ministry of IT in _____ Central
Government.
A) A, the, an
C) An, the, the
B) A, an, the
D) An, an, the
10) You CANNOT send out ______uneducated man into ______ world of technology and
expect him to perform.
A) An, an B) A, an C ) An, the D) The, an

Directions for Questions 11-15:

Readthe passage and answer the questions that follow on the basis of the information provided in
the passage.

Microprocessor is an electronic computer Central Processing Unit (CPU) made from


miniaturized transistors and other circuit elements on a single semiconductor Integrated Circuit
(IC). Before the advent of microprocessors, electronic CPUs were made from individual small-
scale Integrated Circuits
containing the equivalent of only a few transistors. By integrating the processor onto one or a
very few large-scale Integrated Circuit packages (containing the equivalent of thousands or
millions of discrete transistors), the cost of processor power was greatly reduced. The evolution
of microprocessors has been known to follow Moore's Law when it comes to steadily increasing
performance over the years. This law suggests that the complexity of an Integrated Circuit with
respect to minimum component cost will double in about 18 months. From humble beginnings as
the drivers for calculators, the continued increase in power has led to the dominance of
microprocessors over every other form of computer; every system from the largest mainframes
to the smallest handheld computers now uses a microprocessor at their core. .As with many
advances in technology, the microprocessor was an idea wbose time had come. Three projects
arguably delivered a complete microprocessor at about the same time: Intel's 4004, Texas
Instruments' TMS1000, and Garrett AiResearch's Central Air Data Computer. .
A computer-on-a-chip is a variation of a microprocessor, which combines the microprocessor
core (CPU), some memory, and I/O (input/output) lines, all on one chip. The proper meaning of
microcomputer is a computer using a (number of) microprocessor(s) as its CPU(s), while the
concept of the patent is somewhat more similar to a micro controller.
11) Which of the following descriptions would NOT fit a microprocessor?
A) Electronic computer
B) Central Processing Unit
C) Memory disk
D) A single integrated chip circuit.
12) Select the TRUE statement from the following.
A) 11icroprocessors and computers on a chip are variations of each other.
B) Integration of processing power on chips has made processing power cheaper.
C) Before microprocessors, CPUs were not made from individual small scale ICs.
D) A microprocessor circuit only has transistors in it.
13) Which of the following was NOT the first to develop a microprocessor?
A) Microsoft
B) Intel
C) Texas Instruments
D) Garret
14) According to the passage, which of these is NOT a use of microprocessors?
A) Drivers for calculators
B) Core for large mainframes
C) Advanced mobile phones
D) Used for small handheld computers
15) "A number of microprocessors at its CPU" is an apt description of a:
A) 11icro-controller
B) Micro-computer
C) Micro-processor
D) Micro-transistor

Directions for Questions 16-20:

Read the passage and answer the questions that follow on the basis of the information provided
in the passage.

Dynamic Link Libraries

Windows provides several files called dynamic link libraries (DLLs) that contain collections of
software code that perform common functions such as opening or saving a file. When Windows
application wants to use one of those functions or routines, the app sends a message to Windows
with the names of the DLL file and the function. This procedure is known as calling a function.
One of the most frequently used DLLs is Windows COMMDLG.DLL, which includes among
others, the functions to display File Open, File Save, Search, and Print dialog boxes. The
application also sends any information the DLL function will need to complete the operation. For
example, a program calling the Open File function in COMMDLG.DLL would pass along a file
spec, such as *. * or *.DOC, to be displayed in the dialog box's Filename text box. The
application also passes along a specification for the type of information it expects the DLL to
return to the application when the DLL's work is done. The application, for example, may expect
return information in the form of integers, true/false values, or text. Windows passes the
responsibility for program execution to the DLL, along with the parameters and the return
information the DLL will need. The specific DLL is loaded into memory, and then executed by
the processor. At this point the DLL, rather than the application, runs things. The DLL performs
all the operations necessary to communicate with Windows and, through Windows, with the PC's
hardware. After the DLL function is complete, the DLL puts the return information into
memory, where it can be found by the application, and instructs Windows to remove the DLL
routine from memory. The application inspects the return information, which usually tells
whether the DLL function was able to execute correctly. If the operation was a success, the
application continues from where it left off before issuing the function call. If the operation
failed, the application displays an error message.
16) By using DLLs, Windows:
A) Saves processing time
B) Multitasks
C) Shares program code
D) Communicates with PCs hardware
17) To use any routine of a DLL, Windows:
A) Searches and copies it in the application code and executes it
B) Loads the DLL file and searches and executes the routine
C) Loads just the required routine in memory and executes it
D) Searches the location of the routine and instructs the application to execute it
18) Which information does an application need to passto Windows to use a DLL routine?
A) Just the name of the routine
B) Just the name of the DLL, which finds in turn the routine to be executed in return
C) Both the name of the routine as well as DLL and any parameters
D) Name of the DLL, routine, any parameters and type of information to be returned
19) According to the passage, while the DLL routine is executing, the calling application:
A) Waits for the routine to execute
B) Continues with other tasks
C) Helps the DLL routine perform by communicating with Windows and through Windows with
the PC's hardware
D) Passes all responsibility of program execution to the DLL and is removed from memory
20) The DLL function after execution returns:
A) The parameters and information into memory, where it can be inspected by the calling
application
B) Information into memory, where it can be inspected by the calling application
C) To the calling application the information required by it so that it can inspect it
D) The information required into memory so that DLL can inspect whether the function
operation was a success
Section 2 -Analytical Ability
No. of Questions: 20
Duration in Minutes: 20

21) 70 students are required to paint a picture. 52 use green color and some children use red, 38
students use both the colors. How many students use red color?
A) 24 B) 42 C) 56 D)70
22) At an international conference, 100 delegates spoke English, 40 spoke French, and 20 spoke
both English and French. How many delegates could speak at least one of these two languages?
A) 110 B) 100 C) 140 D) 120

23) A group of 50 students were required to clear 2 tasks, one in rock-climbing and the other in
bridge crossing during an adventure sports expedition. 30 students cleared both the tasks. 37
cleared bridge crossing, 38 students cleared rockclimbing.
How many students could not clear any task?
A)0 B)3 C)5 D) 9

24)A dance instructor conducts annual workshops in which he holds sessions for basic learners
and trainers. In a particular year, 2000 people attended the workshop. 1500 participated as
learners and 800 as trainers. How many participated as only trainers?
A) 200 B) 500 C) 800 D) 1500

25) In a group of 400 readers who read science fiction or literacy works or both, 250 read science
fiction and 230 read literacy works. How many read both science fiction and literacy works?
A) 80 B) 160 C) 220 D) 400

26) A man said to a lady, ''Your mother's husband's sister is my aunt." How is the lady related to
the man?
A) Daughter
B) Grand daughter
C) Mother
D) Sister

27) A man is facing west. He turns 45degree in the clockwise direction and then another 180
degree in the same direction and then 270 degree in the anticlockwise direction. Which direction
is he facing now?
A) South B) North-West C) West D) South-West

28) In a row of 60, if Ram is standing at 17th from the first, what is his position from the last?
A) 25 B) 43 C)44 D) 45

29) A man is facing northwest. He turns 90 degrees in the clockwise direction and then
135degrees in the anti-clockwise direction. Which direction is he facing now?
A) East B) West C) North D) South

30) What three letter word bestcompletes the below words?


VA - __E
S___TER
- - _ER
A) STR B)TER C) CAT D) \\fAT

Directions for Questions 31-35:


In the following questions mark:
1. if the question can be answered with the help of statement I alone.
2, if the question can be answered with the help of statement II alone.
3, if the question can be answered with the help of both I and II.
4, if the question cannot be answered at all.

31) What is the value of P?


I. P and Q are integers
II. PQ = 10, P + Q =5
A)l
B) 2
C)3
D)4

32) Who got the highest score in the Mathematics examination, among Sumit, Amit and Namit.
No two students got the
same marks.
I. Sumit got more marks than Namit.
II. Amit did not get lesser marks than Sumit, who did not get lesser marks than Namit.
A)1
B) 2 .
C)3
D)4

33) How many hours does it take some boys and girls in a camp to put up the tent?
I. There are 4 boys and 7 girls.
II. A girl can put up the tent in 5 hours and a boy can put up the tent in 3 hours.
A)1
B) 2
C)3
D)4

34) If p, q, r, s and t are in an Arithmetic Progression, is r the largest among them?


I.t>O
II. p, q < 0
A)1
B) 2
C)3
D)4
35) Is X a whole number, if X > O?
I. 2X is an even number.
II. 3X is an odd number.
A) 1
B) 2
C)3
D) 4

Directions for Questions 36-40:

In a certain code, the symbol for 0 (zero) is. * and that for 1 is $. The numb.:rs greater than 1 are
to be written only by using the two symbols given above. The value of the symbol for 1 doubles
itself every time it shifts one place to the left.
(For example, 4 is written as $**; and; 3 is written as $$)
36) 11x 17 / 10 + 2 x 5 + 3 / 10 can also be represented as:
A) $*$$*
B) $*$$$
C) $$$*$
D) $**$$
37) 260 can be represented as:
A) $****$**
B) $$*$$$$$
C) $$*$$$$**
D) $*****$**
38) 60 / 17 can also be represented as:
A) $$$*$*** / $$**$$
B) $$$***** / $$**$$
C) $*$$*$** / $$**$$
D) $$*$*$** / $$**$$
39) $***$ can be represented as:
A) $$$ / $*
B) $*$**- $$
C) $*$*$- $$
D) $$$***$ - $$
40) 30^2 can be represented as:
A) ($$*$$ ) $*+ $*$*$$*$
B) ($$*$$ ) $* + $$****$
C) ( $$*$$ ) $$ + $*$****
D) ( $$*$$ ) $$ + $*$**$

Section 3 - Attention To Detail


No. of Questions: 11
Duration in Minutes: 11
Directions for Questions 41-45:

Follow the directions given below to answer the questions that follow.

Your answer for each question below would be:


A., if ALL THREE items given in the question are exactly ALIKE.
B, if only the FIRST and SECOND items are exactly ALIKE.
C, if only the FIRST and THIRD items are exactly ALIKE.
D, if only the SECOND and THIRD items are exactly ALIKE.
E, if ALL THREE items are DIFFERENT.

41)LLMLLLKLMPUU, LLMLLLKLMPUU, LLMLLLKLMPUU


A) A B)B C)C D)D E)E
42) 0452-9858762, 0452-9858762, 0452-9858762
A) A B)B C)C D)D E)E
43) NIINIININN, NIININNINN ,NIINIININN
A) A B)B C)C D)D E)E
44) 4665.8009291, 4665.7999291, 4665.8009291
A) A B) B . C)C D)D E)E
45)808088080.8080, 808008080.8080, 808088080.8080
A) A B)B C)C D)D E)E
46) If* standsfor /, / stands for -,+ stands for * and -stands for +, then 9/8*7+5-10=?
A) 13.3
B) 10.8
C) 10.7
D) 11.4
47) If* stands for /, / stands for -,+ stands for * and -stands for +, then 9/15*9+2-9=?
A) 14.7
B) 15.3
C) 14.1
D) 16.2
48) If * stands for /, / stands for -, + stands for * and - stands for +, then which of the following is
TRUE?
A) 36/12*4+50-8 =-106
B) 12*8/4+50-8 =45.5
C) 36*4/12+36-8 = 4.7
D) 8*36/4+50-8 = 300
Set: 3648(A) ver-Z.O For: Aecenture IDC
Directions for Questions 49-49:
In the following questions, the following letters indicate mathematical operations as indicated
below:
A: Addition
V: Equal to
S: Subtraction
W: Greater than
M: Multiplication
X: Less than
D: Division
Out of the four alternatives given in these questions, only one is coccect according to the above
letter symbols. Identify the coccect one.
49) See the options given below
A) 6 S 7 A 2 M 3 W 0 D 7
B) 6 A 7 S 2 M 3 W 0 A 7
C) 6 S 7 M 2 S 3 W 0 M 7
D) 6 M 7 S 2 A 3 X 0 D 7
50) If * stands for -,/ stands for +, + stands for / and -stands for *, then which of the following is
TRUE?
A) 16/8*6+90-12 =23.2
B) 8*12/6+90-12 =7.2
C) 16*6/8+16-12 =-4.1
D) 12*16/6+90-12 =8
51) If * stands for -,/ stands for +, + stands for / and -stands for * , then which of the following is
TRUE?
A) 16*4/18+16-8 = -10.1
B) 18*8/4+40-8 =-2.8
C) 16/18*4+40-8 =33.2
D) 8*16/4+40-8 =-2

Directions for Questions 52-55:

For the post of a m'anager of a leading call centre -Arkade Inc. - situated in Ludhiana, the
following are the criteria the
candidate must satisfy:
-The candidate should have a Management Degree.
-The candidate should have at least 4 years of similar experience at-another call center.
- The candidate should be more than 30 years of age as on the 1st of July 2003.
- The candidate should have 6 months of international exposure, i.e. should have been posted in a
foreign country.
. If a candidate does not satisfy the 1st condition but has more than 2 years of international
experience, then the VP
operations, will interview him.
. If a candidate does not satisfy the 4th condition, then the HR manager will interview him.
52) Shakuntala was selected for a managerial position in an international call center after she
passed out from AIM Management Institute. After working for 3 years in the call center, she
took a sabbatical. She is 29 years of age as on the date of application. She will be:
A) Interviewed only by the HR
B) Inte1'\;ewed only by the VP
C) Rejected
D) Data insufficient

53) Rajiv has been working as a Manager in Zephyr Inc. for 4 years now. He is an Engineering
graduate from a premier engineering institute. His certificate lists his date of birth as 17/12/1974.
He has worked in the hotel industry at the executive level. He is:
A) Give an aptitude test
B) Interviewed by the VP
C) Data insufficient
D) Not considered
54) Soma has 2 years of experience in Welsh Inc. and 2 years of experience in Franc Inc., both
leading call centers, as a manager. She has a management degree from a premier management
organization. She turned 30 this December (2002). She is a B.Com Graduate from St. Xavier's,
Calcutta. If she applies for the post, she will:
A) Be interviewed directly by the VP Operations
B) Not be considered
C) Be interviewed by the HR
D) Have to give an aptitude test
55) Salina has over 4 years of experience in Care Touch, a leading call center, as a manager. She
completed her MBA
from Ranchi and worked in Singapore for UNO for 2 years before joining Care Touch. She will
be:
A) Recruited
B) Rejected
C) Interviewed by dIe VP Operations
D) Data insufficient

1) Non Maskable Interrupt is a…..

a) NMI
b) Software interrupts
c) Hardware interrupts
d) Software and hardware interrupt

Ans: b

2) Which of the following is error correction and deduction?

a) Hamming code

b) CRC

c) VRC

d) None
Ans: b

3) When you switch on your computer, which of the following component affect first?

a) Mother board

b) SMPS

c) Peripherals
d) None

Ans : a

4) Which of the following function transports the data?

a) TCP/IP

b) Transport layer

c) TCP

d) None

Ans: c

5) Which of the following does not consists address?

a) IP

b) TCP/IP

c) Network

d) Transport

Ans:a
6) They given like this……. And some conditions?

a) pre order
b) post order
c) In order
d) None
Ans:c

7) Authentication means….
a) Verifying authority
b) Source
c) Destination
d) Request
Ans: a

8) Symorphous is used for

a) Analysis
b) Synchournization

c) Asynchrouns

d) None

Ans: b

9) There are five nodes. With that how many trees we can make?

a) 27
b) 28
c) 30
d) 29
Ans: c (Check the ans not sure)

10) Traverse the given tree using in order, Preorder and Post order traversals.
A
B
C
D
E
F
G
H
I

Given tree:

Ø Inorder : DHBEAFCIGJ

Ø Preorder: A B D H E C F G I J

Ø Postorder: H D E B F I J G C A

And some more questions….. I dint remember those questions

Given tree:

Ø Inorder : DHBEAFCIGJ

Ø Preorder: A B D H E C F G I J

Ø Postorder: H D E B F I J G C A

And some more questions….. I dint remember those questions

SECTION –II (C language Basics and programming) this section consists of 20 Questions……..
All are programs only…

1. main()

printf("%x",-1<<4);

}
a) fff0
b) fffb
c) ff0
d) none

Ans: a

Explanation: -1 is internally represented as all 1's. When left shifted four times the least
significant 4 bits are filled with 0's.The %x format
specifier specifies that the integer value be printed as a hexadecimal value.

2. main()

char *p;

p="Hello";

printf("%c\n",*&*p);

a) e

b) H
c) some address
d) ome garbage value

Ans: b

Explanation: * is a dereference operator & is a reference operator. They can be applied any
number of times provided it is meaningful. Here
p points to the first character in the string "Hello". *p dereferences it and so its value is H.
Again & references it to an address and * dereferences it to the value H.
3. void main()

int i=5;

printf("%d",i++ + ++i);

a) 11

b) 12

c) 10

d) output cant be predicted

Ans: d

Explanation: Side effects are involved in the evaluation of i

4. main( )

int a[2][3][2] = {{{2,4},{7,8},{3,4}},{{2,2},{2,3},{3,4}}};

printf(“%u %u %u %d \n”,a+1,*a+1,**a+1,***a+1);

a) 100, 100, 100, 2

b) 101,101,101,2
c) 114,104,102,3

d) none

Ans: c

Explanation: The given array is a 3-D one. It can also be viewed as a 1-D array.

2 4 8 3 4 2 2 3 3 4

100 102 104 106 108 110 112 114 116 118 120 122

thus, for the first printf statement a, *a, **a give address of first element . since the indirection
***a gives the value. Hence, the first line of the output.for the second printf a+1 increases in the
third dimension thus points to value at 114, *a+1 increments in second dimension thus points to
104, **a +1 increments the first dimension thus points to 102 and ***a+1 first gets the value at
first location and then increments it by 1. Hence, the output is C is correct answer…

5. main( )

static int a[ ] = {0,1,2,3,4};

int *p[ ] = {a,a+1,a+2,a+3,a+4};

int **ptr = p;

ptr++;
printf(“\n %d %d %d”, ptr-p, *ptr-a, **ptr);

*ptr++;

printf(“\n %d %d %d”, ptr-p, *ptr-a, **ptr);

*++ptr;

printf(“\n %d %d %d”, ptr-p, *ptr-a, **ptr);

++*ptr;

printf(“\n %d %d %d”, ptr-p, *ptr-a, **ptr);

a) 111

222

332

443

b) 111

222

333

344

c) 111

222

333
444

d) None

Ans: b

6. #include

main()

const int i=4;

float j;

j = ++i;

printf("%d %f", i,++j);

a) 8

b) 5

c) compile error

d) syntax error

Ans: c
7. main()

char *p;

int *q;

long *r;

p=q=r=0;

p++;

q++;

r++;

printf("%p...%p...%p",p,q,r);

a) 001… 100…0002

b) 0001...0002...0004

c) 001… 002…004

d) none

ans: b

8. main()

unsigned int i;
for(i=1;i>-2;i--)

printf("HCL Technologies");

a) HCL

b) Technologies

c) HCL Technologies

d) None

Ans: None(Plz Check the answer)

9. main()

int a[10];

printf("%d",*a+1-*a+3);

a) 4

b) 5

c) 6

d) None
Ans : a

10. main()

float f=5,g=10;

enum{i=10,j=20,k=50};

printf("%d\n",++k);

printf("%f\n",f<<2);

printf("%lf\n",f%g);

printf("%lf\n",fmod(f,g));

a) Line no 5: Error: Lvalue required

b) Line no 5: Error: Link error

c) Compile error

d) None

Ans: a

11. int swap(int *a,int *b)

*a=*a+*b;*b=*a-*b;*a=*a-*b;
}

main()

int x=10,y=20;

swap(&x,&y);

printf("x= %d y = %d\n",x,y)

a) x=10 y=20

b) x=20 y=10

c) x=30 y=20

d) none

Ans: b

12. main()

int i=300;

char *ptr = &i;

*++ptr=2;

printf("%d",i);

}
a) 665

b) 565

c) 556

d) none

Ans: c

13.main()

float me=1.1;

double you=1.1;

if(me==you)
printf(”IloveU”);

else
printf(“I Hate U”)

a) I love u

b) I hate u
c) floating point error

d) Compile error

Ans: b
14.

enum colors {BLACK,BLUE,GREEN}

main()

{
printf(”%d..%d..%d”,BLACK,BLUE,GREEN);

return(1);

a) 1..2..3

b) 0..1..2

c) 2..3..4

d) none

Ans: b

Some more questions given.. I dint remember…. Be prepare all basic concept in C… so that you
can answer very easily…
SECTION –III (Data structures and C++) this section consists of 10 Questions… Each question
carry 2 marks… so they deduct ½ mark for wrong answer…..

1) #include

main()

char s[]={'a','b','c','\n','c','\0'};

char *p,*str,*str1;

p=&s[3];

str=p;

str1=s;

printf("%d",++*p + ++*str1-32);

a)97

b) M

c)76

d) none

Ans: b
2) main( )

int a[ ] = {10,20,30,40,50},j,*p;

for(j=0; j<5; j++)

printf(“%d” ,*a);

a++;

p = a;

for(j=0; j<5; j++)

printf(“%d ” ,*p);

p++;

a) address of array
b) Compile error

c) Lvalue required

d) none

Ans: c

3) struct aaa{

struct aaa *prev;

int i;

struct aaa *next;

};

main()

struct aaa abc,def,ghi,jkl;

int x=100;

abc.i=0;abc.prev=&jkl;

abc.next=&def;

def.i=1;def.prev=&abc;def.next=&ghi;

ghi.i=2;ghi.prev=&def;
ghi.next=&jkl;

jkl.i=3;jkl.prev=&ghi;jkl.next=&abc;

x=abc.next->next->prev->next->i;

printf("%d",x);

a) 3

b) 2

c) 4

d) 5

Ans: b

4) main(int argc, char **argv)

printf("enter the character");

getchar();

sum(argv[1],argv[2]);

}
sum(num1,num2)

int num1,num2;

return num1+num2;

a) compile error

b) L value required

c) Syntax error

d) None

Ans: a

5)

class Sample

public:

int *ptr;

Sample(int i)
{

ptr = new int(i);

~Sample()

delete ptr;

void PrintVal()

cout << "The value is " << *ptr;

};

void SomeFunc(Sample x)

cout << "Say i am in someFunc " << endl;

int main()

Sample s1= 10;


SomeFunc(s1);

s1.PrintVal();

a) say I am in someFunc

b) say I am in someFunc and runtime error

c) say I am in someFunc and null value

d) none

ans: b

and some questions given in DATASTURES… those are based on linked lists only also very big
programs… so you have to do it very careful

SECTION-III (General aptitude+1 passage+Logical) this section consists of 20 questions.. Each


question carry 1 mark…

1) Sandhya and Bhagya is having the total amount of 12000. In that amount bhagya has deducted
3600 as less as sandhya. So what is their shared amount?

a) 2800

b) 3600
c) 4800

d) 9600

Ans : c

2) Six persons have to present at certain meeting. The conditions are,

A present P should present


M present T should present
K present P should present

If A and P present I should be there in meeting

If M and T present D should be absent

If K and P present P should present

Based on this they given some questions… Those are easy only.. you can do it easily…

3) Here they given passage following some questions.. Poetry explaining her experience
with HINDI latest songs… and comparing with old songs… also she is a good singer.
Like this they given a big passage… so read it carefully at a time.. so that u can save your
time
4) Dhana and Lavanya started running at same point… But dhana started in anti clock
wise direction, Lavanya started in clockwise direction.. Dhana met lavanya at 900 m .
where as lavanya met dhana at 800 m… so how much distance they covered each other?

a) 1700

b) 900

c) 1800

d) data is insufficient

Ans: d

5) This question is base on Arithmetic mean like algebra… a n+2 = (7+an)/5…. Initially a0=
0…… so what is the value of a2?

a) 5/2

b) 7/2

c) 7/5

d) none

Ans : c

6) Here they given two statements, based on that they gave some questions
Statement I : I is enough to answer

Statement II: I and II is enough to answer

i) Raja can do a piece of work in 9 days… and govardhan can do a piece of work in 8
days. In how many days they will complete the work alternatively..

Statement I: They both do in 72/17 days

Statement II : A alone can do 1/9 days


a) I b) I and II c) II d) none

1. Which of the following involves context switch,

(a) system call


(b) priviliged instruction
(c) floating poitnt exception
Freshersworld.com
(d) all the above
(e) none of the above

Ans: (a)

2. In OST, terminal emulation is done in

(a) sessions layer


(b) application layer
(c) presentation layer
(d) transport layer

Ans: (b)

3. For a 25MHz processor , what is the time taken by the instruction which needs 3 clock cycles,

(a)120 nano secs


(b)120 micro secs
(c)75 nano secs
(d)75 micro secs

4. For 1 MB memory, the number of address lines required,

(a)11
(b)16 Freshersworld.com
(c)22
(d) 24

Ans. (b)

5. Semaphore is used for

(a) synchronization
(b) dead-lock avoidence
(c) box
(d) none

Ans. (a)
6. Which holds true for the following statement

class c: public A, public B

a) 2 member in class A, B should not have same name


b) 2 member in class A, C should not have same name
c) both
d) none

Ans. (a)

7. Question related to java

8. OLE is used in

a) inter connection in unix Freshersworld.com


b) interconnection in WINDOWS
c) interconnection in WINDOWS NT

9. Convert a given HEX number to OCTAL

10. Macros and function are related in what aspect?

(a)recursion
(b)varying no of arguments
(c)hypochecking
(d)type declaration
11.Preproconia.. does not do which one of the following

(a) macro
(b) conditional compliclation
(c) in type checking
(d) including load file

Ans. (c)

12. Piggy backing is a technique for

a) Flow control
b) Sequence
c) Acknowledgement Freshersworld.com
d) retransmition

Ans. (c)

13. In signed magnitude notation what is the minimum value that can be represented with 8 bits

(a) -128
(b) -255
(c) -127
(d) 0

14. There is an employer table with key fields as employer number data
in every n'th row are needed for a simple following queries will get required results.

(a) select A employee number from employee A , where exists from employee B where A
employee no. >= B
employee having (count(*) mod n)=0
(b) select employee number from employe A, employe B where A employe number>=B employ
number
group by employee number having(count(*) mod n=0 )
(c) both (a) & (b)
(d) none of the above

15. Type duplicates of a row in a table customer with non uniform key field customer number
you can see

a) delete from costomer where customer number exists( select distinct customer number from
customer having count )
b) delete customer a where customer number in b rowid
c) delete customer a where custermor number in( select customer number from customer a,
customer b )
d) none of the above

Section B

1. Given the following statement


enum day = { jan = 1 ,feb=4, april, may} Freshersworld.com
What is the value of may?

(a) 4
(b) 5
(c) 6
(d) 11
(e) None of the above
2. Find the output for the following C program

main
{int x,j,k;
j=k=6;x=2;
x=j*k;
printf("%d", x);

3. Find the output for the following C program

fn f(x)
{ if(x<=0)
return;
else f(x-1)+x;
}

4. Find the output for the following C program

i=20,k=0;
for(j=1;j<i;j=1+4*(i/j))
{k+=j<10?4:3;
}
printf("%d", k);

Ans. k=4

5. Find the output for the following C program

int i =10
main()
{int i =20,n; Freshersworld.com
for(n=0;n<=i;)
{int i=10;
i++;
}
printf("%d", i);

Ans. i=20

6. Find the output for the following C program

int x=5;
y= x&y

7.Find the output for the following C program

Y=10;
if( Y++>9 && Y++!=10 && Y++>10)
{printf("%d", Y);
else
printf("%d", Y);
}

Ans. 13

8. Find the output for the following C program

f=(x>y)?x:y

a) f points to max of x and y


Freshersworld.com
b) f points to min of x and y
c)error

Ans. (a)

9. What is the sizeof(long int)

(a) 4 bytes
(b) 2 bytes
(c) compiler dependent
(d) 8 bytes

10. Which of the function operator cannot be over loaded

(a) <=
(b) ?:
(c) ==
(d) *

11. Find the output for the following C program

main()
{intx=2,y=6,z=6;
x=y==z;
printf(%d",x)
}
Section C (Programming Skills)

Answer the questions based on the following program

STRUCT DOUBLELIST
{ DOUBLE CLINKED
INT DET; LIST VOID
STRUCT PREVIOUS; (BE GIVEN AND A PROCEDURE TO DELETE)
STRUCT NEW; (AN ELEMENT WILL BE GIVEN)
}
DELETE(STRUCT NODE)
{NODE-PREV-NEXT NODE-NEXT;
NODE-NEXT-PREV NODE-PREV;
IF(NODE==HEAD)
NODE
}

Q. In what case the prev was

(a) All cases


(b) It does not work for the last element
(c) It does not for the first element
(d) None of these

Answer the questions based on the following program

VOID FUNCTION(INT KK)


{KK+=20;
}
VOID FUNCTION (INT K)
INT MM,N=&M
KN = K
KN+-=10;
}

Q. What is the output of the following program

main()
{ int var=25,varp;
varp=&var;
varp p = 10;
fnc(varp)
printf("%d%d,var,varp);
Freshersworld.com
}

(a) 20,55
(b) 35,35
(c) 25,25
(d)55,55

Section D

1. a=2, b=3, c=6


Find the value of c/(a+b)-(a+b)/c

2. What does the hexanumber E78 in radix 7.

(a) 12455
(b) 14153
(c) 14256
(d) 13541
(e) 131112

Ans. (d)

3. 10 : 4 seconds :: ? : 6 minutes

Ans. 900

4. Q is not equal to zero and k = (Q x n - s)/2.What is n?

(a) (2 x k + s)/Q
(b) (2 x s x k)/Q
(c) (2 x k - s)/Q
(d) (2 x k + s x Q)/Q Freshersworld.com
(e) (k + s)/Q

5. From the following statements determing the order of ranking

• M has double the amount as D


• Y has 3 rupess more than half the amount of D

Ans. Data insuffiecient

Questions 6 - 10 are to be answered on the following data

• A causes B or C, but not both


• F occurs only if B occurs
• D occurs if B or C occurs
• E occurs only if C occurs
• J occurs only if E or F occurs
• D causes G,H or both
• H occurs if E occurs
• G occurs if F occurs

6. If A occurs which of the following must occurs

I. F and G
II. E and H
III. D

(a) I only
(b) II only
(c) III only
(d) I,II, & III
(e) I & II (or) II & III but not both

Ans. (e)

7. If B occurs which must occur

(a) D
(b) D and G
(c) G and H
(d) F and G
(e) J

Ans. (a)

8. If J occurs which must have occured


(a) E
(b) either B or C
(c) both E & F
(d) B
(e) both B & C

Ans. (b)

9. Which may occurs as a result of cause not mentioned

I. D
II. A
III. F

a. I only
b. II only
c. I & II
d. II & III
e. I,II & III

Ans. (c)

10. E occurs which one cannot occurs

(a) A
(b) F
(c) D
(d) C
(e) J

Ans. (b)
1 The closing of the resturant by Mr.X on SEPT 1 was considered an unfinancial one, as the
weather remained unusually
clear and sunny for another one month. An author who criticizes the act of Mr. X would be
proved wrong if the
following was true?? ANS choice a) the weather did not usually remained fine after SEPT 1.

2 SUSAN works in a company who has restricted its employees from smoking cigerrates in
the canteen. As susan
is the employee of the company she does not smoke cigerrate in the canteen.Which of the
following unused
phrases strengthens the rules of the company?? ANS the employees normally do not do the
work for which the
company has forbidden them to do.
3 A q's on family relation was given like How many sons X has, I P is the daughter of X ,II
some condt., III some
condt. ANS al I ,II, III together are not sufficient.
4 A q's in which a name KAPIL is given he visits manoj's home.some condts given. ANS b)
5 A,B,C,D are the 4 plays which are organised starting from tuesday.find the day on which C
was played.in this
2 condt. will be given as , I.....................,
II...................., ANS both I and II
6 A quest on crypto graphy like
ABCD
EFGH
--------------
. .................... .is A=, find the other values. practice these types of quest.
7. A question on race was given.hell lot of condts. finally they make a team for 4*100 metres
medaly.
ANS E none of the above
8. Piggy backing is a technique for a) Flow control b) sequence c) Acknowledgement d)
retransmition
ans: c piggy backing
9.. The layer in the OST model handles terminal emulation
a) session b) application c) presentation d) transport
ans: b application
10 ans: a odd numbers of errors
11. In signed magnitude notation what is the minimum
value that can be represented with 8 bits
a) -128 b) -255 c) -127 d) 0 ANS a)
12 c 20(no of address lines in 1MB of memory)
13 A 120(25 hz processor,what is the time taken by the instr which needs 3 clock cycles)
14 B synchronise the access(semaphores used for)
15 A system call(context switching is used in)
16 B the operating system(mapping of virtual to physical address)
17 A 177333(conversion of HEX "0xFEDB"in octal)
18 D used as a network layer protocall in network and windows(OLE) system
19 B has to be unique in the sub network(internet address)
20. There is an employer table with key feilds as employer no. data in every n'th row are needed
for a simple
following queries will get required results.
a) select A employe no. from employe A , where exists
from employe B where A employe no. >= B employe
having (count(*) mod n)=0
b) select employe no. from employe A, employe B where
A employe no. >= B employ no. grouply employe no.
having (count(*) mod n=0 )
c) both a& b d)none of the above
21 . type duplicates of a row in a table customer with non uniform key feild customer no. you
can see
a) delete from costomer where customer no. exists ( select distinct customer no. from
customer having count )
b) delete customer a where customer no. in (select customer b where custermer no. equal to b
custemor no.)
and a rowid > b rowid c) delete customer a where custermor no. in ( select customer no.
from customer a,
customer b ) d) none of the above
22. which feature in ANSI C but not in JAVA.??ANS variable arguments.
23. preprocessor does not do one of the following??ANS type checking.
24. long int size a) 4 bytes b) 2 bytes c) compiler dependent d) 8 bytes
ans: compiler dependent
25. x=2,y=6,z=6 x=y==z;
printf(%d",x) ?ANS 1
26. class c : public A,publicB
a) 2 members in class a,b can have member functions with same name.
b) 2 members in class a,c can have member functions
with same name. c)both d)none(ANS)
27. What will be the out put of the following program
main()
{
char *p;
p=malloc(10);
free(p);
printf("%d",p);
}
ANS compilation error

28. a=(10,15), b=10,15 what are the values of a & b in ANSI C ANS 15,10
29 main()
{
int x=10,y=15,z=16;
x=y=z;
printf("%d",x);
}
ANS 0
30 f(n) f(x)
{
if(x<=0)
return;
else f(x-1)+x;
}
find the value of fn(5)? ANS 15.
31 struct {
int det;
struct prevoius;
struct new;
}
delete(struct node)
{
node-prev-next=node-next;
node-next-prev=node-prev;
if(node==head)node
}
one element will be given. ANS::it
does not work when rp is the last element in the link list.
32 A code will be given which searches a particular char in the string. ANS:: it always works.
33. main()
{
int var =25,varp;
varp=&var;
varp p=10;
fnc(varp);
printf("%d%d",var,varp);
}
ANS::55,55 (check this out)
34. #define VALUE 1+2
main()
{
printf("%d and %d\n",VALUE/VALUE,VALUE*3);
}
ANS:: 5,7
35 What is the value assigned to the variable a if b is 7 a=b>8?b<<2:b>4?b>>1:b; ANS::3
36 .the value of the following expr (2^3)+(a^a) is a) 1 b)2 c) 3 d) insufficient data
37 which of the following is not basic data type ANS char*
38. the declaration of the variable does not result in oneof the following ANS allocatrion of the
storage space for the
varable.
39. in C parameters are passed by ANS:: value only.
40. 2 variables cannot have the same name if they are ANS:: in the same block.
41.a static funct. say s(),in as file f.c can be invoked from ANS all functs. in f.c after the
definitions of s.
42.macros and functions do not differ in the following aspects ANS::variable no of arguments.
43.one q's in which he will give some different forms of STRCPY function you will have to find
out which form is
correct.

CTS:
Written Test

Verbal Ability (20 minutes 25 questions)


5 on Finding the Correct Sentences.
5 on Finding the the incorrect Sentences.
5 on Arranging the Jumbled Sentences.
2 Reading Comprehensions with 5 Questions in Each.

Tip : First answer finding the incorrect sentences and the Jumbled sentences, Then Correction of
sentences and then read a Reading Comprehension that is short in textual matter in that
order.First read the RC questions and then read the RC, You will get all the answers in no time.
Quantitative (30 minutes 25 questions) :

5 on Venn Diagrams
5 Data Sufficiency problems
5 on Binary Numbers
5 on Cubes
5 on Some Aptitude questions on Averages and Equations.

Ven Diagram Problem (5 Questions)

(1)In a party 10 take only tea 15 take tea and 8 take only coffee then what is the number of
persons take either coffee or tea.(nos are not exact).

(2) In a college there are students who can play either football or cricket or both. 500 play cricket
220 play both and 600 play football .what is the total strength of the college.

(3)At an international conference, 100 delegates spoke English, 40 spoke French, and 20 spoke
both English and French. How many delegates could speak at least one of these two languages?
(4)In a group of 400 readers who read science fiction or literacy works or both, 250 read science
fiction and 230 read literacy works. How many read both science fiction and literacy works?

(5)In town of 500 people, 285 read Hindu and 212 read Indian express and 127read Times of
India 20 read hindu and times of India and 29 read hindu and Indian express and 35 read times of
India and Indian express. 50 read no news paper. Then how many read only one paper?

**the question is not exact but the wordings are similar.Ther was Five such type of questions;

Binary Numbers (5 Questions)

In a certain code, the symbol for 0 (zero) is. * and that for 1 is $. The numb.:rs greater than 1 are
to be written only by using the two symbols given above. The value of the symbol for 1 doubles
itself every time it shifts one place to the left.

(For example, 4 is written as $**; and; 3 is written as $$)

(6) 260 can be represented as:

A) $****$**

B) $$*$$$$$

C) $$*$$$$**

D) $*****$**
(7) 60 / 17 can also be represented as:

A) $$$*$*** / $$**$$

B) $$$***** / $$**$$

C) $*$$*$** / $$**$$

D) $$*$*$** / $$**$$

(8) $***$ can be represented as:

A) $$$ / $*

B) $*$**- $$

C) $*$*$- $$

D) $$$***$ - $$

(9) 30^2 can be represented as:

A) ($$*$$ ) $*+ $*$*$$*$

B) ($$*$$ ) $* + $$****$

C) ( $$*$$ ) $$ + $*$****

D) ( $$*$$ ) $$ + $*$**$
(10) 11x 17 / 10 + 2 x 5 + 3 / 10 can also be represented as:

A) $*$$*

B) $*$$$

C) $$$*$

D) $**$$

Binary 3 - 011 & 4 - 100 & 5 - 001 so convert it into binary n solve it.

The data in these 5 questions are not what was given in the exam. But the pattern is same

Data Sufficiency problems (5 Questions)

(Direction)Each questions given, has a problem and two statements numbered I and II given
certain information. You have to decide if the information given in the statement is sufficient for
answering the problem. Indicate your answer as

(a) If the data in statement I along are sufficient to answer the questions;
(b) If the data in statement II along are sufficient to answer the questions;
(c) If both statements are required.
(d) If the data in both the statement together are not sufficient to answer the questions.

(11) In the last month the company decide to increase the cost of its mixer by 10%. What is the
present prise of muxer?
I. The cost of mixer and juicer together was Rs. 2850 a month ago.
II. The amount of 10% increase on the mixer comes to Rs 220.

(Ans: B Because Only Statement II is needed to answer the above questions.)

Other Questions was same type….

(For this Section Just Go through Data sufficiency Chapter (Chapter 16) from R.S. Aggarwal.
This is enough for answering these questions)

Cubes (5 questions)

A red cube cut into 125 equal size of small cube. Then from the top layer Remove the all the
cube from 2nd and 4th row. Then do the same for the 2nd layer and so on to the last layer. After
that the entire structure painted Black.

Answer the Following questions:

(1) 4 side painted red.


(2) 4 side painted black
(3) 3 side painted Black
………………………..
………………………..

Aptitude questions
(1) A and B start walking same time form the same point and walk in same direction. A walk at
a speed of 3mile/hour and B 4mile/hour. After 2hour and 15 min B take a rest for 45 min. But A
continuously walks. After taking rest B start to return. How much distance from the starting
point they meet together? (Ans : 9 Mile)

(2) Appu ranks sixteenth from the top and forty ninth from the bottom in a class. How many
students are there in the class? (Ans : 64)

Tip: First do the Binary numbers problem, its a direct lifting from the Book Quicker Maths by
Tyra. It says substitute $=1 and *=0 and just solve the problems. There will some catches like it
doubles and things like that just forget them and go on doing its just plain Binary numbers.

->Data Sufficiency was very easy. One Question on Right angled triangle in which given 2 sides
we are asked whether we can find the area with just one side given, or we need both.

->Then do the Venn Diagrams. Please practice these before you go, it’s very easy and it uses 3
sets on a whole. You need to solve for the intersection of those from the basic formula and you
are 90% done with the questions.

->Cubes problem was a bit problematic and confusing. It will be better if you try if after solving
other problems. Though I was not able to solve the cube problem, I scored 18 out of 25. But
some times they give some simple cube problem. Then do it first. Like the following problem

* A cube which is painted all side is divided into 125 parts

Q How many single side painted


Q How many 2 sided
Q how many 3 sided
Q how many none

this is very easy if u use the formulae


Total no.of cubes= n3
exactly one face=6*(n-2)whole square
exactly two faces painted=12(n-2)
no face painted =(n-2)whole cube

Analytical (20 minutes 20 questions)

5-6 logic questions like

1)All Fans are Coolers


Some Coolers are AC's

2)No Politician is Honest


No honest person is intelligent

3)All fruits are vegetables


All vegetables are green

choose the conclusion from following…….

Some figure odd man out matching questions (10 questions) like

(4)
Ans : (a); Because All other figures have the four cups opening in the different direction.

Nonverbal from RS Agarwal will help you a lot.Some easy questions in picture based to look
forward will be the figures which are closed and open figures.There will be only one Closed
figure and other open and surely and ODD man Out

Venn diagrams :( questions 1-5)


1. Out of 50 men in a town, 30 are Lions club members, 10 are Lions members as well as
Rotary club members. There are 3 who do not belong either. Then how many Rotary club
members are there?

(a) 15 (b) 18 (c) 20 (d) 25

2. Total of 100 people work in an organization. Of which 63 work in office, 57 work


in factory and there are none who work outside. Then how many of them work in
both?

(a) 10 (b) 25 (c) 18 (d) 20

3. Repeat the question no 2.how many of them work only in office?

(a) 43 (b) 40 (c) 52 (d) 45

4. In a college one day 120 students came to canteen. on that day 50 drank Tea,60
drank Coffee ,45 drank Cool drinks, 20 drank both Tea &Coffee, 15 drank both
Coffee& Cool drink ,none drank both ,none drank all the three.
How many drank only tea?

(a) 42 (b) 30 (c) 22 (d) 32

5. Repeat the question no 4.how many of them drink at least two?

(a) 40 (b) 30 (c) 25 (d) 35

Aptitude questions (6-10)


6. A train for Luck now leaves for every 2 1/2hrs from Delhi station. An announcement
was made that train left 40mins ago and next train comes at 18:00hrs.At what time was
the announcement made?

(a) 17:00hrs (b) 16:30hrs (c) 17:25hrs (d) 16:10hrs

7. A merchant purchased 20 dozens of pencils at rate of 375 Rs per dozen. Then he sold
each one at 33Rs.what is his % profit?

(a) 4.5 (b) 6.2 (c) 5.6 (d) 3.5

8. Value of vehicle at end of each year depreciates at the rate of 3/5th of value at the
beginning. If Initial value is 10,000Rs then at the end of three years its value is?

(a) 2600 (b) 2160 (c) 2100 (d) 2400

9. I don’t remember this question


10. 500 people came to exhibition. Entry fee was 3Rs each. A draw was held on tickets
and announced 1st prize of 300Rs/- and 3 second prizes of 100Rs/- each. How much gain
was raised with the draw?

Data Sufficiency problems (Questions 11-15)


Direction) each questions given has a problem and two statements numbered I and II
given certain information. You have to decide if the information given in the statement is
sufficient for answering the problem. Indicate your answer as

(I) if the data in statement I along are sufficient to answer the questions;
(II) If the data in statement II along are sufficient to answer the questions;
(III) If both statements are required.
(IV) If the data in both the statement together are not sufficient to answer the questions.
11. Is square of number an integer?

(i) Number=6
(ii) Number=11/12

(a) I (b) II (c) III (d) IV

12. Area of rectangle PQRS?

(i)PQ=RS=10
(ii)PR=QS=10^0.5

(a) I (b) II (c) III (d) IV

13. The number of persons entered into hall after 8:00PM

(i)if rate is 2 persons per min after 8:00PM


(ii) If rate is 4 persons per two mins after 8:00PM

(a) I (b) II (c) III (d) IV

14. If 3 friends A, B, C can completes work in 36 days then in how many days can B
complete?

(i)if A&B together complete in 18 days


(ii) if B&C together complete in 24 days
(a) I (b) II (c) III (d) IV

15. I exactly don’t remember it

Binary Numbers conversion (Questions 16-20)


(Directions) In a certain code, the symbol for 0 (zero) is. * And that for 1 is $. The numbers
greater than 1 are to be written only by using the two symbols given above. The value of the
symbol for 1 doubles itself every time it shifts one place to the left.

16. Lcm ($*$, $$$$, $*$**$*)


17. (9-2*3*5+6*4+2*3)
18. Average (70,170,270)
19.($*$$)$$ - ($**$)*$
20. Represent 343

Problems on Cubes (questions 20-25)


There are two cubes (A, B). Each of it is cut into 64 pieces .From the front face of cube A cubes
on both the diagonal are removed. Same is repeated to cube B. Now cube A is placed over cube
B to form a rectangular cuboid and painted black.
21. The number of cubes that are not painted black?
(a) 32 (b) 36 (c) (d) 48

22. The number of cubes that are one side painted black?
(a) 32 (b) 36 (c) (d) 48

23. The number of cubes that are two sides painted black?
(a) 32 (b) 36 (c) (d) 48

24. The number of cubes that are three sides painted black?
(a) 32 (b) 36 (c) (d) 48
25. The number of cubes that are one side painted black?
(a) 32 (b) 36 (c) (d) 48

SECTION-III
Analytical (20 mins 20 questions)

1-7 questions: pick the odd one out


8-10 questions: pick the next sequence
11-12 questions: on logical sense (I mean you have to choose the options that logically follow
the statements made)
11. I eat whenever am in the hotel
(i) I am in hotel
(ii)I eat
(iii) I don’t eat
(iv) I am not in hotel.

(a) ii, i (b) iii, iv (c)i ,ii (d)iv, iii


12. Either I study or watch T.V
(i) I study
(ii)I watch T.V
(iii) I don’t study
(iv) I don’t watch T.V

(a) iii, i (b) iii, i (c)iv ,ii (d)iv, ii


13-17 questions: on logical deductions
13. All intelligent are merit holders.
Some merit holders are poor
14. All mosquitoes are ants
Some ants are mammals
15. Some graduates are MBA holders.
Some graduates who are not MBA holders are unemployed
16.-17 i dont remember them

Practice these from RS Agarwals Verbal & Nonverbal


Reasoning
18-20 questions: on simple puzzle
I n a city there is a building of 4 storey height. there are 8 friends A,B,C,D,E,F,G,H who
live in that building with 2 flats on each floor . Flats on ground floor are numbered 5, 6 from left
end to right. Flats on first floor are numbered 3, 4 from right end to left. Flats on second floor are
numbered 1, 2 from left side to right. Flats on third floor are numbered 7, 8 from right side to
left.
Now A lives in flat 4.
B lives in the top floor along with D and two storeys above Flat of F.
C lives in flat 5.
H&E lives in the same floor.
18. What is flat number of B?
(a) 3 (b) 1 (c) 7 (d) 6
19. What is correct order?
(a) 3 -A (b) 1-D (c) 7 -E (d) 6-F
20. What if D&F, C&E exchange there flats then who stays below C?
(a) A (b) D (c) F (d) G
HCL:

) Non Maskable Interrupt is a…..


a) NMI
b) Software interrupts
c) Hardware interrupts
d) Software and hardware interrupt
Ans: b
2) Which of the following is error correction and deduction?
a) Hamming code
b) CRC
c) VRC
d) None
Ans: b
3) When you switch on your computer, which of the following component affect first?
a) Mother board
b) SMPS
c) Peripherals
d) None
Ans : a
4) Which of the following function transports the data?

a) TCP/IP
b) Transport layer
c) TCP
d) None
Ans: c
5) Which of the following does not consists address?
a) IP
b) TCP/IP
c) Network
d) Transport
Ans:a
6) They given like this……. And some conditions?
a) pre order
b) post order
c) In order
d) None
Ans:c
7) Authentication means….
a) Verifying authority
b) Source
c) Destination
d) Request
Ans: a
8) Symorphous is used for
a) Analysis
b) Synchournization
c) Asynchrouns
d) None
Ans: b
9) There are five nodes. With that how many trees we can make?
a) 27
b) 28
c) 30
d) 29
Ans: c (Check the ans not sure)

10) Traverse the given tree using in order, Preorder and Post order traversals.
A
B
C
D
E
F
G
H
I
Given tree:
Ø Inorder : DHBEAFCIGJ
Ø Preorder: A B D H E C F G I J
Ø Postorder: H D E B F I J G C A
And some more questions….. I dint remember those questions
Given tree:
Ø Inorder : DHBEAFCIGJ
Ø Preorder: A B D H E C F G I J
Ø Postorder: H D E B F I J G C A
And some more questions….. I dint remember those questions

SECTION –II (C language Basics and programming) this section consists of 20 Questions……..
All are programs only…

1. main()
{
printf("%x",-1<<4);
}
a) fff0
b) fffb
c) ff0
d) none

Ans: a

Explanation: -1 is internally represented as all 1's. When left shifted four times the least
significant 4 bits are filled with 0's.The %x format
specifier specifies that the integer value be printed as a hexadecimal value.
2. main()
{
char *p;
p="Hello";
printf("%c\n",*&*p);
}
a) e
b) H
c) some address
d) ome garbage value
Ans: b
Explanation: * is a dereference operator & is a reference operator. They can be applied any
number of times provided it is meaningful. Here
p points to the first character in the string "Hello". *p dereferences it and so its value is H.
Again & references it to an address and * dereferences it to the value H.

3. void main()
{
int i=5;
printf("%d",i++ + ++i);
}
a) 11
b) 12
c) 10
d) output cant be predicted
Ans: d

Explanation: Side effects are involved in the evaluation of i


4. main( )
{
int a[2][3][2] = {{{2,4},{7,8},{3,4}},{{2,2},{2,3},{3,4}}};
printf(“%u %u %u %d \n”,a+1,*a+1,**a+1,***a+1);
}
a) 100, 100, 100, 2
b) 101,101,101,2
c) 114,104,102,3
d) none
Ans: c

Explanation: The given array is a 3-D one. It can also be viewed as a 1-D array.

2 4 7 8 3 4 2 2 2 3 3 4
100 102 104 106 108 110 112 114 116 118 120 122

thus, for the first printf statement a, *a, **a give address of first element . since the indirection
***a gives the value. Hence, the first line of the output.for the second printf a+1 increases in the
third dimension thus points to value at 114, *a+1 increments in second dimension thus points to
104, **a +1 increments the first dimension thus points to 102 and ***a+1 first gets the value at
first location and then increments it by 1. Hence, the output is C is correct answer…

5. main( )
{
static int a[ ] = {0,1,2,3,4};
int *p[ ] = {a,a+1,a+2,a+3,a+4};
int **ptr = p;
ptr++;
printf(“\n %d %d %d”, ptr-p, *ptr-a, **ptr);
*ptr++;
printf(“\n %d %d %d”, ptr-p, *ptr-a, **ptr);
*++ptr;
printf(“\n %d %d %d”, ptr-p, *ptr-a, **ptr);
++*ptr;
printf(“\n %d %d %d”, ptr-p, *ptr-a, **ptr);
}
a) 111
222
332
443
b) 111
222
333
344
c) 111
222
333
444
d) None
Ans: b

6. #include
main()
{
const int i=4;
float j;
j = ++i;
printf("%d %f", i,++j);
}

a) 8
b) 5
c) compile error
d) syntax error
Ans: c

7. main()
{
char *p;
int *q;
long *r;
p=q=r=0;
p++;
q++;
r++;
printf("%p...%p...%p",p,q,r);
}
a) 001… 100…0002
b) 0001...0002...0004
c) 001… 002…004
d) none
ans: b
8. main()
{
unsigned int i;
for(i=1;i>-2;i--)
printf("HCL Technologies");
}
a) HCL
b) Technologies
c) HCL Technologies
d) None
Ans: None(Plz Check the answer)

9. main()
{
int a[10];
printf("%d",*a+1-*a+3);
}

a) 4
b) 5
c) 6

d) None
Ans : a

10. main()
{
float f=5,g=10;
enum{i=10,j=20,k=50};
printf("%d\n",++k);
printf("%f\n",f<<2);
printf("%lf\n",f%g);
printf("%lf\n",fmod(f,g));
}
a) Line no 5: Error: Lvalue required
b) Line no 5: Error: Link error
c) Compile error
d) None
Ans: a
11. int swap(int *a,int *b)
{
*a=*a+*b;*b=*a-*b;*a=*a-*b;
}
main()
{
int x=10,y=20;
swap(&x,&y);
printf("x= %d y = %d\n",x,y)
}
a) x=10 y=20
b) x=20 y=10
c) x=30 y=20
d) none
Ans: b
12. main()
{
int i=300;
char *ptr = &i;
*++ptr=2;
printf("%d",i);
}

a) 665

b) 565

c) 556

d) none
Ans: c

13.main()

float me=1.1;
double you=1.1;
if(me==you)
printf(”IloveU”);

else
printf(“I Hate U”)
}
a) I love u
b) I hate u
c) floating point error
d) Compile error

Ans: b
14.
enum colors {BLACK,BLUE,GREEN}

main()

{
printf(”%d..%d..%d”,BLACK,BLUE,GREEN);

return(1);

}
a) 1..2..3
b) 0..1..2
c) 2..3..4
d) none
Ans: b
Some more questions given.. I dint remember…. Be prepare all basic concept in C… so that you
can answer very easily…

SECTION –III (Data structures and C++) this section consists of 10 Questions… Each question
carry 2 marks… so they deduct ½ mark for wrong answer…..

1) #include
main()
{
char s[]={'a','b','c','\n','c','\0'};
char *p,*str,*str1;
p=&s[3];
str=p;
str1=s;
printf("%d",++*p + ++*str1-32);
}

a)97
b) M
c)76
d) none
Ans: b

2) main( )
{
int a[ ] = {10,20,30,40,50},j,*p;
for(j=0; j<5; j++)
{
printf(“%d” ,*a);
a++;
}
p = a;
for(j=0; j<5; j++)
{
printf(“%d ” ,*p);
p++;
}
}

a) address of array
b) Compile error
c) Lvalue required
d) none

Ans: c

3) struct aaa{
struct aaa *prev;
int i;
struct aaa *next;
};
main()
{
struct aaa abc,def,ghi,jkl;
int x=100;
abc.i=0;abc.prev=&jkl;
abc.next=&def;
def.i=1;def.prev=&abc;def.next=&ghi;
ghi.i=2;ghi.prev=&def;
ghi.next=&jkl;
jkl.i=3;jkl.prev=&ghi;jkl.next=&abc;
x=abc.next->next->prev->next->i;
printf("%d",x);
}

a) 3
b) 2
c) 4
d) 5

Ans: b

4) main(int argc, char **argv)


{
printf("enter the character");
getchar();
sum(argv[1],argv[2]);
}
sum(num1,num2)
int num1,num2;
{
return num1+num2;
}

a) compile error
b) L value required
c) Syntax error
d) None

Ans: a

5)
class Sample
{
public:
int *ptr;
Sample(int i)
{
ptr = new int(i);
}
~Sample()
{
delete ptr;
}
void PrintVal()
{
cout << "The value is " << *ptr;
}
};
void SomeFunc(Sample x)
{
cout << "Say i am in someFunc " << endl;
}
int main()
{
Sample s1= 10;
SomeFunc(s1);
s1.PrintVal();
}

a) say I am in someFunc
b) say I am in someFunc and runtime error
c) say I am in someFunc and null value
d) none

ans: b

and some questions given in DATASTURES… those are based on linked lists only also very big
programs… so you have to do it very careful
SECTION-III (General aptitude+1 passage+Logical) this section consists of 20 questions.. Each
question carry 1 mark…

1) Sandhya and Bhagya is having the total amount of 12000. In that amount bhagya has deducted
3600 as less as sandhya. So what is their shared amount?
a) 2800
b) 3600
c) 4800
d) 9600

Ans : c

2) Six persons have to present at certain meeting. The conditions are,


A present P should present
M present T should present
K present P should present

If A and P present I should be there in meeting


If M and T present D should be absent
If K and P present P should present
Based on this they given some questions… Those are easy only.. you can do it easily…

3) Here they given passage following some questions.. Poetry explaining her experience with
HINDI latest songs… and comparing with old songs… also she is a good singer. Like this
they given a big passage… so read it carefully at a time.. so that u can save your time

4) Dhana and Lavanya started running at same point… But dhana started in anti clock wise
direction, Lavanya started in clockwise direction.. Dhana met lavanya at 900 m . where as
lavanya met dhana at 800 m… so how much distance they covered each other?

a) 1700
b) 900
c) 1800
d) data is insufficient
Ans: d

5) This question is base on Arithmetic mean like algebra… a n+2 = (7+an)/5…. Initially a0=
0…… so what is the value of a2?

a) 5/2
b) 7/2
c) 7/5
d) none
Ans : c

6) Here they given two statements, based on that they gave some questions

Statement I : I is enough to answer


Statement II: I and II is enough to answer
i) Raja can do a piece of work in 9 days… and govardhan can do a piece of work in 8 days. In
how many days they will complete the work alternatively..

Statement I: They both do in 72/17 days


Statement II : A alone can do 1/9 days
a) I b) I and II c) II d) none
HP:

Q : What is not a part of OS ?


O: swapper,compiler,device driver,file system.
A : compiler.

Q : what is the condition called when the CPU is busy swapping in and out pages of memory
without doing any useful work ?
O : Dining philosopher's problem,thrashing,racearound,option d
A: thrashing.

Q : How are the pages got into main memory from secondary memory? DMA,
Interrupts,option3, option 4
A : as far as i know its Interrupts --by raising a page fault exception.

Q : What is the use of Indexing ?


O : fast linear access, fast random access, sorting of records , option 4
A : find out.

Q : in terms of both space and time which sorting is effecient. (The question is rephrased .)
O : merge sort, bubble sort, quick sort, option 4
A : find out

which case statement will be executed in the following code ?


main()
{
int i =1;
switch(i)
{
i++;
case 1 : printf ("");
break;
case 2 : printf("");
break;
default : printf("");
break;
}
}

Answer : Case1 will only be executed.

Q : In the given structure how do you initialize the day feild?


struct time {
char * day ;
int * mon ;
int * year ;
} * times;

Options : *(times).day, *(times->day), *times->*day.

Answer : *(times->day) -- after the execution of this statement compiler generates


error.i didn't understand why.can anybody explain.

Q: The char has 1 byte boundary , short has 2 byte boundary, int has 4 byte boundary.
what is the total no: of bytes consumed by the following structure:
struct st {
char a ;
char b;
short c ;
int z[2] ;
char d ;
short f;
int q ;
}

Options are given.


Answer : its very easy 20 and not 19 .

IT INformation

1. Who is associated with C language?


2. CEO of MIcrosoft
3.Company producing Routers,Bridges
4.From 70s 80s Company producing
mouse,GUIs............a)Dell b)IBM c) Xerox d) HP
5.Inventor of Pascal
6.Abt SUN micro systems
7.first digital comp machine a)von neumann b) Dijkstra
8.Which of these is not search engine a)hotbot
b)parametric c)Altavista d) google
9.In 2002 IBM acquired which company a) Rational b)
oracle c) netscape
10.founder of apple computer(2 persons)

Computer Concepts
1.Complexity nlogn a.quick sort b. bubble sort c.
shell sort
2.Abt Protocols
3.Abt SSL
4.Abt MBR
5.Cross compiler
6.synonymus to main memory
7.Semaphores
8.Database quesns Stored procedures
Trigger strategy
9.Bandwidth measured in ?
10.Term assoc with Context Transaction

Java

1.extending the class means


2.how many values does char of java has
3.Java pgm can be executed on any m/c how?
4.utility used for dynamically changing the tables
a.vector b. hash table c. enumeration
d. structures
5.synchronise means
6.if X & Y r objects X = Y means
7.which action Java platform restricts on applets
a. Image viewing
b. Accept user i/p
c. write to system files
d. initialise date in browser
8.quesns on JDBC
9.Two quesns on simple java pgms.
10. Java was initially code named as:
a)Oak b)green c)miller
11.) what is not true about the following statements
about java.
a) it is compiled using javac compiler
b) the compiled files have .class extension.
c) such files cannot be transfered from one comp to
another.
d) they use the java interprete

1) HP acquired this company in 2002. Which is the company


a)Compaq b)Dell c)option 3 d) Option4
Ans: a

2) what does 3G denote

a) 3rd generation mobile communication b) 3rd generation computer languages c) option 3 d)


option4
Ans: a

3)an application program that is used by the users to get the inofrmation from the backend of
some application like databases:
a) application server b)proxy server c)database server d)option 4
Ans: database server

4) which of the following is not true about the e-mail


a) it can be accessed by a client program using POP
b) it can be accessed by a client program using imap protocol
c) option 3 d) option 4
Ans: I don't remember the answer but first 2 are true.

5) Some quesion regarding the company and who developed it ( the thing to remember is that
Apple produce Macintosh computers).
5) What is X.25?
a)option 1 b)option 2 c)option 3 d)option 4
Ans: find out??:-)

>>> napster app is used for : downloading music

3> c/c++ section: questions on c/c++, programs o/p etc.


1) main( )

unsigned int i=3;

while( i >=0)

printf( "%d", i--);

how many times will the printf stmt be executed?


a)0 b)3 c)4 d)infinite

Ans: I think the answer is infinite, b'cos 'i' is an unsigned integer and it will not decrement below
'0' and hence end up in an infinite loop.(yes, i checked and it is getting stuck in an infinite loop)

2) main( )
{
int x,y, z;
x=2;
y=5;
z= x+++y;
printf("%d %d %d", x, y z);
}
a)3 5 7 b)option 2 c)option 3 d)option 4
Ans: a

3) # define swap(a,b) temp=a; a=b; b=temp;

main( )
{
int i, j, temp;
i=5;
j=10;
temp=0;
if( i > j)
swap( i, j );
printf( "%d %d %d", i, j, temp);
}
Ans: On compiling i got ans 10, 0, 0. I did not understand the concept. Please expalin this to me.

4>Java section: questions on java related stuff.


1) Java was initially code named as:
a)Oak b)green c)miller d)option4
Ans: Oak

2) what is not true about the following statements about java.


a) it is compiled using javac compiler
b) the compiled files have .class extension.
c) such files cannot be transfered from one comp to another.
d) they use the java interpreter
Ans: c
3) Why is the synchronize used?

a) to initialize multiple objects b)to lock an object c)option3 d)option 4


Ans: b (probably)

some java qns


1.
package java.lang; // Ans: <------------! Runtime gives error ( test it)
class HPTest2
{
public static void main(String args[])
{
String x="sdfsd";
System.out.println("Hello");
}
}

2
class HPTest
{
static HPTest x;
public static void main(String args[])
{
// below line not given
//x=new HPTest(); if not this line Null pointer exeception
x.foo();
}
void foo()
{
System.out.println("Hello");
}
}

3:
class A
{
A(int x)
{
System.out.println(" x=" + x );
}
A(int x,int y)
{
System.out.println(" x=" + x + " y =" + y);
}
}
class HPTest4
{
public static void main(String args[])
{
A a=new A(1);
A b=new A(2,3);
A c=new A();
}
}

1)which of the following is not associated with operting systems


a)swapper b)device drivers c)file system d)process mgmt
2)what is the size of virtual memory that must be associated
a)must be same as physical memory b)must be > physical memory
c)can be of any size d)it must be small enough to allocate process
3)which of following is true
a)time sharing is also multi progrmming
b) multi progrmmming is also time sharing
4)global variables are stored at
a)heap b)stack c)data d)code

computer organization:

1)32 registers are there.an instructuion can hold upto 3 registers . the opcode is of 4 bits.
what is minimum size of instruction.
ans:the minimum sizeof instruction has nothing to do with number of registers
2) some bit on pipelineling
3)some bit related to SIMD,MIMD,MISD,SISID
4)minimum no of nand gates require to implement xor gate
ans : 4 gates(check out)

unix:

1)two way communication is possible by means of means of which of the following


ann:unix_socket
2)wc -l x>x prints out
ans:wc command prints 0
3)the background process that continusly run
ans:deamon process
networks:
1)fragmented packet is reunited at
a)destination b)at next gateway c)at highest MTU gateway d)at the next router
2)the following address 93.58.5.0
a)class A b) class B c) class C d)class D
3)network to host protocal is
ans:RARP
data structres:

1)given a doubly linked list .you are given with a node n and a pointer p associated with it. what
are the operarions that to be performed to delete that node.

2)swapping the elements of left child of a tree with that of right child .
revswap(node*)
{
node* tree;
if(tree!=null)
swap(tree->left,tree->right)
revswap(tree->left);
revswap(tree->right);
}
if given tree is
1
23
4567
what is the output for about function applied
ans:
1
32
7654

3)t(n)=4+t(n/2) ,t(0)=1. what is the complexity

4)in hashing each element can be found with complexity of


ans: 1
5)In a program all the addresses that are to be binded to the caller function are done by
a)compiler b) linker c) loader d) run time allocator
6) s->v[integer]
interger->interger,integer/termial
ans: a[2,3,5]

7)char 1 byte , short of 2 bytes , integer of 4 byte,


stuct
{
char a;
char b;
int a[2];
short d;
int e;
char i;
} name;
sizeof(naame) ans:16
note: consider c compiler unde unix for all c questions
c:
---------
1)main()
{
int i=1;
swith(i)
{
i++;
case 1: printf("case1");
break;
case 2: printf("case 2");
break;
default: printf("default");
}
ans: when we compile we get warning that unreachable code and if we execute we get "case 1"
will be printed.

C And C++

1. i=23, j=4, c=0;


c=i++-++j;
o/p?

2. #define CUBE(x) x*x*x


#define MUL3(x,y) 3*x*y
#define ADD(x) x+y
some statement maikng use of the preprocessors defined above............find o/p.

3. virtual destructors r used for ?

4. 2 questions on friend functions & classes

5. char c[]="123456789";
i=4;
printf("%c %c", c[i], i[c]);
o/p?
one more question on these lines.

6. int *ptr;
p=0;
p++;
printf("%u", p);
o/p?
a. 0 b. garbage value c. 4 d. none of the above

7. double i=0.0;
switch(i)
{
case 0.0:
printf("jgdj");
case 1.0:
printf("ptoy");
break;
default:
printf("hdfv");
}
o/p?

8. volatile int i;
in d above statement, is "volatile"
a. a type declaration b. type qualifier c. storage class d. none of the above

Unix

1. "bash" is a kind of shell in UNIX

2. primitive flavours of unix


options : a. BSD &Sys V b. LINUX & BSD c..

3. which of the folllowing is used 4 redirection in UNIX? >, |, <, ^

4. wot is d UNIX terminology 4 multi-tasking?


a. time slicing b. pre-emptive ....... c. time division d.......

5. In UNIX if You try accessing a directory for which u dont have permission, wot
message is displayed?
a. permission denied b. invalid user c. access denied d.........

Analogies
1. slur : speech : : smudge :?
Ans: writing

2. epaulet : shoulder : : ring :?


Ans:finger

3. vernacular : place : : fingerprint : ?


Ans:identical
Opposites
1. corpulent
Ans: emaciated

2. officious
Ans: pragmate

3. dextrous
Ans: clumsy
The following sentences are broken into 4 sections- A, B, C, D Choose the part that has a
mistake Mark (E) if you
find no mistake.

1). a) Psychologists point out that b) there are human processes

c) Which does not involve d) the use of words

Ans. (C) which does not involve (do)

2). a) jack ordered for b) two plates of chicken c) and a glass d) of water

Ans. (A) jack ordered for

The following is a group of questions is based on a passage or a set of conditions for each
question. Select the best answer choice given.

(i). If law forbids it if the object of agreement is the doing of an act, that is forbidden by law the
agreement is void.
(ii). If it is of the nature that, it would defeat the provision of any law is the agreement is void. if
the object of
agreement is such that thing got directly forbidden by law it would defeat the provision of
statuary law.
(iii). If the object of agreement is fraudulent it is void.

(iv). An object of agreement is void if it involves or implies to the personal property of another.

(v). An object of agreement is void where the constant regards as ignored.

(vi). An object of agreement is void where the constant regard is as opposed to public policy.

1). An algorithm follows a six step process za, zb, zc, zd, ze, zf, it is governed by the following:

(i) zd should follow ze (ii) the first may be za,zd or zf


(iii) zb and zc have to be performed after zd (iv) zc must be immediately after zb

2). If za is the first set zd must be

a) 3rd b) 5th c) 2nd d) 4th

3). If zb must follow za then za can be

a) Third or fourth b) first or second c) can not be third d) fourth or fifth e) none

4). If ze is third term the number of different operations possible are…

The following questions are based on the given statements:

Ravi plants six separate saplings -- x, y, z, w, u, v in rows no 1 to 6, according to the following


conditions:
a) He must plant x before y and u… b) He must plant y and w… c) The third has to be z

5). Which of the following is acceptable

a) xuywzv b) xvzyuw c) zuyxwv d) zvxuwy e) wyzuvx

6). Which of the following is true


a) z before v b) z before x c) w before u d) y before u e) x before w

7). If he plants v first, then which can be planted second

a) x b) y c) z d) w e) u

8). Which of the following describes a correct combination of sapling and row?

a) x,3 b) y,6 c) z,1 d) w,2 e) u,6

9). If he plants b 6th which would be planted first and second

a) x and w b) x and y c)y and x d)w and z e) w and u

10). If he plants w before u and after v he should plant w at

a) First b) second c) fourth d) fifth e) sixth

11). At a certain moment a watch shows 2 min lag although it is running fast.If it showed a 3 min
lag at that moment,
but also gains by 1/2 min more a day than its current speed it would show the true time
one day sooner
than it usually does.How many minutes does the watch gain per day.
a) 2 b) 5 c) 6 d) 4 e) 75
12). In 400m race A gives B a start of 7 sec and beats him by 24 sec.In another race A beats B by
10 sec. The
speeds are in the ratio?

a) 8:7 b) 7:6 c) 10:8 d) 6:8 e) 12:10

13). 3x+4y=10 and x3 + y3=6… what is the minimum value of 3x+11y = ?

14). There are 600 tennis players. 4% wear wristband on one wrist. Of the remaining, 25% wear
wristbands on
both hands. How many players don't wear a wristband? Ans. 432
15). Three types of tea a, b, c costs Rs. 95/kg, 100/kg and70/kg respectively.How many kgs of
each should be
blended to produce 100 kg of mixture worth Rs.90/kg, given that the quantities of band c are
equal?
a) 70,15,15 b) 50,25,25 c) 60,20,20 d) 40,30,30
Ans. (b)
16). Two distinct no's are taken from 1,2,3,4...28 Find the probability that their sum is less than
13?
I-GATE:

1. Last month of an year


(a) January (b) February (c) December (d) November
2. Select the odd one
(a) January (b) February (c) Wednesday (d) November
3. Select the antonym of capture from the following
(a) attack (b) Release (c) condemn (d) None of the above
4. Find the antonym of autumn
(a) Spring (b) Winter (c) Summer (d) None of the above
5. One skirt requires 3.75 yards of cloth. How many skirts you can make from 45 yards?
Ans: 12 skirts6. How can you make a square from two triangles?
6. Is the meaning of Client and Customer,
(a) same (b) contradictory (c) no relation
7. Is the meaning of Client and Customer,
(a) same (b) contradictory (c) no relation
8. Is the meaning of Canvas and Canvass,
(a) same (b) contradictory (c) no relation
9. . Is the meaning of Ingenious and Ingenuous,
(a) same (b) contradictory (c) no relation
10. Is the meaning of Credible and Credulous,
(a) same (b) contradictory (c) no relation
11. Select the odd one out.
(a) 1/4 (b) 1/3 (c) 1/6 (d) 1/18
12. Select the least from the following.
(a) 0.99 (b) 1 (c) 81 (d) 0.333
13. Find the next number in the series. 1, 0.5, 0.25, 0.125
Ans: 0.0625
14. One do llar is saved in one month. Then how much dollar is saved in one day?
Ans: 1/30 =0.0333$
15. Y catches 5 times more fishes than X. If total number of fishes caught by X and Y is 48,
then number of fishes caught by X?
Ans: 8
16. Y catches 5 times more fishes than X. If total number of fishes caught by X and Y is 42,
then number of fishes caught by X?
Ans: 7
17. If a train covers 600m in 0.5 seconds, how long it will cover in 10 seconds?
Ans: 3000m = 3km
18. The girl's age is twice that of boy, if the boy is four years old. After four years the age
of the girl is
Ans: 12 years
19. Sister's age is twice than that of the brother. If the brother's age is six, what is the sister's
age after two years?
Ans: 14 Yrs.
20. Two lemons cost 10 cents. Then one and a half dozen cost
Ans: 90 cents
21. A clock is late by 1 minute 27 seconds in a month. Then how much will it be late in 1
day?
Ans: 2.9 seconds
22. Which of the following figures together will make a triangle?
Ans: a,b,c,d
23. Make a square by drawing only one line
Ans: line 2-5, square 2-3-4-5-2
24. Make a square by drawing only one line
Ans: line 2-5, square 2-3-4-5-2
25. Opposite of Remote?
(a) Far (b) Near (c) Huge (d) Village
26. Statement A: All great men are ridiculous;
Statement B: I am ridiculous ;
Inference : I am a great man;
(a) True (b) False (c) Not clear
27. Statement: Normal children are active;
Inference: All children are active;
(a) True (b) False (c) Uncertain
28. Next number in the series 1, 1/2, 1/4, 1/8 ?
Ans: 1/16
29. In 6 seconds a light flashes once. In one hour how many times it will flash?
Ans: 601 times
30. 31. At 20% discount, a cycle is sold at a selling price of 2500 Rs. What is the actual
price?
Ans: Rs. 3125
31. Statement A: A & B have same age;
Statement B: B is younger than C;
Inference : A is younger than C;
(a) True (b) False (c) Uncertain
32. All chickens lay eggs (True/False)
Ans: False
33. A invests $12000, B invests $8000, C invests $6000 and they got a profit of $1200. How
much share A got more than B and C?
Ans: 2/13 and 3/13
For technical there were 20 multiple choice questions: Out of which about 6 was
electronics and 14
was from computer related areas
1. Full form of TTL and CMOS
2. Which is a good conductor (Extrinsic or Intrinsic)
3. What are the different types of capacitors (Electrolytic, dielectric...etc)
4. Select a passive component from the following (four choices were there)
5. Minimum no. of lines required for communication using RS232 (Ans: i think its 2)
6. To convert 1's complement to 2's complement and vice versa
7. During which time we use 'size of' command. (ans: runtime)
8. Out of four choice we have to identify which is a macro.
9. There was one pointer _expression related question.
10. To find post fix _expression.
11. o find post fix _expression.
11. What type of operating system is unix (ans: pre-emptive,[not sure])
a)pre-emptive b)non-preemptive c)batch
12. Defnition of turing machine.
13. Where we use DFD(Data flow design)
a)structural languages b)object oriented languages c)UML d)all of the above
14. Name the error which occurs when we write on a page
a) segment fault b)permission fault c) page fault
15. A question based on the representation of an array in C
An array whose elements are fn pointers which inturn returns a character
1. If MASTER : VIXOSP then what's STREAM : ?
Ans : XOPSIV

2. Boys and girls are seated in 5 rows and 5 columns, such that A is seated at the center
and P is right to A. If R is in front of A and Q is behind P. In which direction is Q seated
with respect to R. If all the students are seated facing north.
Ans : south east

3. If 21 books must be placed in a row. In how many ways these books can be arranged
such that no two new books are arranged consecutively,,,
Ans : 210

4. Two dice are thrown such that a sum of 8 is obtained in a condition that the second die
must be always 4. Find the num of chances of obtaining sum of 8.
Ans : 1/6

5. Two cans A & B contain 25% & 30 % of milk. If 6 parts of can A and 4 parts of can B
acre mixed in a can C. What is the proportion of milk in can C.
Ans : 15

6. 1 rupee, 50 paisa and 25 paisa coins are there in the ratio of 2.5 : 3 : 4. For Rs 210, find
the number of 1 rupee coins.
Ans : 105.

7. 9 men and 6 women will finish a work in 5 days, similarly 3 men and 15 women can
finish the same work in 10 days then in how many days 9 men and 10 women will finish
the same work.
Ans :3

8. X amount takes 10 years to become double. How many years it takes to become
triple ?
Ans :15

9. A team plays cricket in a year with a % of win by the team as 50% and the % of
loosing matches as 40%. If the number of matches tied is 20, then find the number of
matches played.
Ans : 200

10. There are 8 pairs of socks and 2 socks are worn from that such that the pair of socks
worn are not of the same pair. what is the number of pair that can be formed.
Ans :56

One Nibble = ?
2. Hexadecimal and Octal representation of 1024 ?
3. What a compiler does ?
4. DOS ? (whether Graphical interface or Character interface and two more
choices are given)
5. CPU - abbrevation
6. Fortran used as ( Ans : Scientific Language)
7. ASCII character set ? (total number)
8. Windows NT- What NT stands for?
9. Which is not an input device ? (keyboard, Disk, Mouse,Lightpen)
10. Which is not a pointing device ?( mouse,Joy stick,lightpen none)
11. Microsoft Chief ?
12. 4GL ? (Fortran,SQL,ADA, ....)
13. Father of Computers?
14. Which of them is Object Oriented Language.?
15. Power PC is the product of ? (Microsoft,IBM,Apple,Motorola,Intel)
(A combination of any correct 3 companies)
16. Latest processor used ? (Pentium, Power PC, Pentium pro,none)
17. MS Word is ?
18. First Super Computer built in India. (Ans : param)
19. Which of the following companies donot manufacture chips?
(Microsoft, Motorola, Intel, HP)
20. LAN - abbrevation
21. WAN - abbrevation
22. Modem is ? (related to electrical hardware.- Like Modulator &demodulator)
23. FDD - abbrevation ( floppy Disc drive)
24. BIT - abbrevation
25. Information is ? (message, data,processed data,none)
26. Which is not networking ?(internet, Ethernet, arcnet,none)
27. One Gigabyte =?( 2 power30 , 2 power 20 ,2 power 10 none)
28. Which of the following is not RDBMS ? (sybase, SQL,Acess,none)_
29. Oracle is ? (Ans : Relationasl Data Base Management System)
30. In Oracle, Table means ( Ans : Collection of records)
31. DMA - abbrevation (Direct Memory Access,Discrete memory Access,
Disk memoryaccess)
32. What is meant byStatic Variable ?
33. What is meant by QUEUE? {refer any data structures text}
34. What is meant by STACK? {refer any data structures text}
35. The processor used in first IBM PC? (8086,8088,zig4,intel)
36. Difference between 80286 and 80287
37. In bubble sort , no. of comparisons required ?(ans : formula : N*(N-1)/2)
38. No. of comparisons of an item in 100 items by binary comparison?
( 10,25,50 100)
39. CRT - (Cathode Ray Tube)
40. No. of entryvalues are there in ideally in a subroutine.
41. Binary tree?
42. Flow in one direction ? (Single linked list,Double linked
list,.......,....)
43. Electron screen size ? (here, 2 lines of algorithm is given.
Name the algoirithm)
44. Which is not storage device.? (printer,CD ROM,Disk,none)
45. A question regarding memory ? ( least used memory,recently unused
memory,..)
46. ISO - (International Standard Organisation)
47. HTML - (Hyper Text Makeup Language)
48. Flow chart for factorial N? (ans :choice a)
49. What is meant by Recursion ?

50. Struct(s)
{
int a;
long b;
}
Union (u)
{int a;
long b;
}
Print sizeof(s)and sizeof(u) if sizeof(int)=4 and sizeof(long)=4

51.Switch (i)
i=1;
case 1
i++;
case 2
++i;
break; ( ans : 1,2,3,none)
case 3
--i;
Output of i after executing the program

52. char S;
char S[6]= " HELLO";
printf("%s ",S[6]);

output of the above program ? (0, ASCII 0, I,unpredictable)

53. Unsigned char c;


for ( c=0;c!=256;c++2)
printf("%d",c);

No. of times the loop is executed ? (127,128,256,infinitely)

54. int i;
i=0;
repeat
i=i+1; <====== PASCAL PROGRAM
print i;
until(i<10)
end
No. of times the loop is executed?

55. Convert (int A,var ,int B;int c)


{
A=10;
B=4-;
C=120;
}
Convert (inta,b,c)
{ <====== PASCAL PROGRAM
a=1;
b=4;
c=12;
}
convert (A,B,c) ? (10,40,120
10,40,12,..........,............)

56. Procedure A
Begin

--------

end <====== PASCAL PROGRAM


Procedure B No. Of errors in the program ?(1,2,3,none)
Begin

-----------

end
57. int i;
i=2;
i++;
if(i=4)
{
printf(i=4);
}
else
{
printf(i=3);
}
output of the program ? (4,3,unpredictable,none)

58. what is FAT?.

File allocation table

QUANTITATIVE APPTITUDE TEST :


---------------------------

Total 44 questions are there.


Refer R.S. Aggarwal

1. How many degrees hours hand rotate in 10 minutes?


2. 1/(10 power 18) - 1/(10 power 20) = ?
3. 0 <x < 1: Which is greater ? (1/(x square),1/x,x,x square)
4. c=a/b ; a-1=c What is the relation between a&b
5. N = ((8.42)**2)*(95. ....)/((0.982)*(64.....)**2)
If N contains the first three digits as 988,Which of the following is the
nearest answer (9.88,98.8,988,none)
6. The sum of 7 consecutive odd integers with 27 as the fourth number.
7. (66666666666666)**2 + 888888888888=?
8. 2486724...... Express it in product of 3 numbers.(The given number
is not exactly correct. Choices are given. Very easy)
9. Radius of sphere is increased by 50%. By how much percentage is surface
area is increased.
10. In which of the following , 2 as a common factor , can be eliminated.

Ans : log(x square)/log(y square) = log(x)/log(y)

Antonyms :
11. AUSPICISIOUS
12. REHALISHLY
13. & 14. - Sentence correction
15, 16 , 17 - Spelling Mistakes
18, 19 - Two Questions

6 letters are assigned certain numbers.


In each qusetion , 4 combination of digits are given . Of which , which gives
a meaningful word.
20 & 21. - 4 sentences will be given in each question. Arrange them in
logical order .
22. 5 straight lines are cut in a circle. A question regarding this.

Reasoning :

a) If statement A alone is required


b) If statement B alone is required
c) If bothe statements A & B are required
d) Neither

23. Does Mr. Mathew give tution ?


A) MAthew is a teacher
B) In school, the teacher is not expected to give tutions.
24. There is no power cut in advanced country
If T is a city, Is there power cut in that city?
A) Z is a advanced country
B) T ia a capital city of country Z

25. Tanzanians are in East Africa. Tanzanizns are good in either Education ,
Dance or Music
A) Tanzanians are good at Education
B) East Africa are good at dance & Music.

Remaining questions 26 - 29 are of the same type.

30. On onw day, 14 children are admitted in a school by their mothers.


2 are sisters, 3 are brothers,2 are brother and sister and 2 are twins.
The rest are singles. How many mothers came? (5 , 7, 9, 14)

31. Problem Cisterns & pipes.

It will take 8hrs to fill a cistern. But due to leak at the bottom, it
take 10 hrs to full it. In how many hrs, th eful tank will be emptied
because of leak.? (18,8,40,10)

32. Question regarding men & work

A does alone a work in 4 days


B does alone a work in 8 days
C does alone a work in 10 days
A & B together ------
C & B together ------

33. In 3.5 Kg rod , there is 74% silver. If it is alloyed with a 0.5 Kg rod,
the % of silver goes p to 84%. The percent of silver in 0.5 Kg rod?
34. Two chords of lengths 2L1 and 2L2 are drawn in a circle. Their lengths
are inversely proportional to the straight distance joining the centre.
Find the radius of circle.

35. A Kg of tea costs Rs 49.50 . But the supplier gives 10 gms less for
every Kg he sold. What is the actual cost.

36 - 40

A question on transportation table. 5 questions were given.

-------------------------------------------------------
ABCDE

-------------------------------------------------------
A X 12 8 20 6
-------------------------------------------------------
B 12 X 12 5 9
-------------------------------------------------------
C 20 8 X 4 7
-------------------------------------------------------
D 3 15 6 X 10
-------------------------------------------------------
E 12 5 8 3 X
-------------------------------------------------------

A,B,C,D,E denote the stages.


X denote the start of the stage.
The bus goes from A to E and E to A woith back stops at B,C, & D
For each the charge is Rs.0.70
The numbers in the table are how many passengers are there in the bus
upto that stage.( the numbers given the table are not correct)
36. Total no. of passengers in onward journey

37. Total amount in the conductors bag just before the bus reaches
the stage C

38. How many Rs. 1.40 tickets are issued to passengers in backward journey.

39. If the bus breaks down between the stages C & D , the amount refunded to
passengers.

40. If the ticket costs Rs.1.50, how much is the profit in Backward journey.

Remaining 4 questions may be fron English.

---------------------------------------------------------

1)At 6'o clock clock ticks 6 times. The time between first and
last ticks was 30sec. How much time it takes at 12'o clock.

Ans. 66 sec. 2 marks.

2)Three friends divided some bullets equally. After all of them


shot 4 bullets the total no.of remaining bullets is equal to that of
one has after division. Find the original number divided.

Ans. x x x
x-4 x-4 x-4
3x-12 = x
x= 6
ans is 18 2 marks
3)A ship went on a voyage after 180 miles a plane statrted with 10 times
speed that of the ship. Find the distance when they meet from
starting point.

Ans. 180 + (x/10) = x


x = 20
ans is 180+20 2 marks

4) Fill the empty slots.

played won lost draw Goals for Goals against


A 2 2 *0 *0 *7 1
B 2 *0 *1 1 2 4
C 2 *0 *1 *1 3 7

the slots with stars are answers. 4 marks


BC drew with 2-2
A won on B by 2-0
a won on C by 5-1

5) There are 3 societies a,b,c. a lent tractors to b and c as many


as they had. After some time b gave as many tractors to a and c
as many as they have. After sometime c did the same thing.
At the end of this transaction each one of them had 24.
Find the tractors each orginally had.

Ans a had 39 b had 21 c had 12 4 marks

6) There N stations on a railroad. After adding x stations 46 additional


tickets have to be printed. Find N and X.

Ans. let N(N-1) = t;


(N+x)(N+x-1) = t+46;
trail and error method x=2 and N=11 4 marks
7)Given that April 1 is tuesday. a,b,c are 3 persons told that their
farewell party was on
a - may 8, thursday
b - may 10,tuesday
c - june 8, friday
Out of a,b,c one is only correct one of the regarding month,day,date.
Other told two correct and the third person told all wrong.What is
correct date,month,day. 5 marks

8)There are 4 parties. df,gs,dl(depositloss),ew ran for a contest.


Anup,Sujit,John made the following statements regarding results.
Anup said either df or ew will definitely win
sujit said he is confident that df will not win
John said he is confident that neither ew nor dl will win
the result has come. only one of the above three has made a correct
statement. Who has made the correct statement and who has won
the contest. 5 marks.

9)Five people a,b,c,d,e are related to each other. Four of them make
one true statement each as follows.
i) b is my father's brother. (ans. d said this)
ii)e is my mother-in-law. ( b )
iii)c is my son-in-law's brother. ( e )
iv)a is my brother's wife. (c)

10 marks.

10) All members of d are also members of a


All '' e '' d
all '' c '' both a and b
not all '' a are members of d
not all '' d '' e
a)which of all following statements must be true
1) there are more common members between c and a than between
c and e
2) there are more members in a than in c
3) there are more members in a than in e

i) 1 only ii) 3 only iii) 1,2 only iv) 2 and 3 only v)1,2,3
b)which of all the following can be inferred from the statements above
1)some members of d are also members of b
2)all members of e are members of a
3)all members of a are members of b
4)some members of d are not members of c
5)all members of b are members of c

i) 1 only ii) 3 only iii)2 only iv) b and d only v)d,b and a only

d)Which of the following is not possible


i)some members of e are also members of c but are not members of b
ii) '' '' d '' '''' both c and b
iii)all '' e '' '' a
iv)some '' d are not '' a
v) '' e are '' b
1) i only 2) iii only 3) i and iii only 4) all the above 5)none

e)if all members of b are also members of c then


i) all members of a are also members of b
ii)all '' '' c
iii) '' b ''' a
iv) some members c '' e and b
v) '' '' d and b
1) ii only 2) iii and iv only 3) iii only 4) ii and iv only
5)none
11)
boys are allowed to watch football at c.v.Raman auditorium subjected to
conditions.
i)the boy over age 16 can wear overcoat
ii)no boy over age 15 can wear cap
iii)to watch the football either he has to wear overcoat or cap
or both
iv) a boy with an umberella or above 16 or both cannot wear sweater.
v) boys must either not watch football or wear sweater.

What is the appearence of the boy who is watching football.

Iam sending the Tata Ibm paper by three parts. Some more

paper will be sent by Bhadram. Here is the first part

*******************************************

12. After the following command is executed

$ ln old new

a listing is performed with the following output

$ls -li

total 3
15768 -rw-rw-rw- 2 you 29 sep 27 12:07 old
15768 -rw-rw-rw- 2 you 29 sep 27 12:07 New
15274 -rw-rw-rw- 1 you 40 sep 27 09:34 very old

Which of the following is true


a) Old and new have the same i-node number ,2
b) old and new have the same i-node number ,15768
c) old and new have nothing to do with each other
d) very old and new are linked
e) veryold and old are linked

29) Which of the following is true about fork ()


a) It transforms an executable binary file into a process

29) Which of the following is true about fork ()

a) It transforms an executable binary file into a process


that overlays the process which made the fork ()
system call. No new proces is created Ans B;
b) Causes the creation of a new process the CHILD process
with a new process ID.
c) Causes the creation of a new process, the child process with
the same process ID as the parent
d) fork() has nothing to do with processes.
e) fork() is not a system call

1)which message is displayed when a window is destroyed

a)WM-CLOSE B)WM-DESTROY C)WM-NCDESTROY D)WM-POST DESTROY

ans: check your self may be B

2) what is meant by SEND MESSAGE AND POST MESSAGE

ans:we don't know


3)which is the message used to limit the size of the window

a)WM-SIZE B)WM-RAWT C)WM-GETMINIMAX INFO D)WM-COMMAND

E)WM-CREATE

3)until |who|grep mary


do
step 60
done

a)is syntactically correct


b)waits 60 sec. irrespective of mary being logged in or not
c)waits until mary is logged in
d)waits till mary exists
e)none

ans:C

4)the unix system call that transform an executable binary file into a process:
ns: we don't know

5)fork() is a system call causes

ans: causes the creation of a new process , the child person with a new process ID
6)main()
{
char *x="string"
char y[]="add";
char *z;
z=(char *) malloc(sizeof(x)+size of(y)+1)
strcpy(z,y);
strcat(z,x);
printf("%s+%s=%s",y,x,z);
}

ans:Add+string=addstring (if it is in caps also true)

7)char *(*(*a[n]())();

ans:d)an array of n pointers to functions returning pointers to functins returning pointers to


charecters

8)interprocess command in unix be achieved by

a)pipes b)messages c)semaphores d)shared memory e0all

ans: E (all)
INFOSYS:

APTY PAPER : duration 1 hr

(1) a jeweler makes a window display. he has 7 gems of which he needs to display 6. Three on
the left side of the pane on three on the right. With following conditions. let the gems be
Armanet, diamond, emerald, sapphire, garnet, opal and ruby.
1.armanet should be always displayed on left side
2.diamond always on the right
3.ruby should not be kept with diamond or garnet.
4.emerald and sapphire should always be on the same side. (8 marks)

1] what are the possible combinations on the possible for left pane.
a] Armanet, garnet, opal
b] Armanet, ruby, garnet
c] Diamond, emerald, sapphire
d] Armanet, emerald, garnet
a
2] What are the possible combinations on the possible for right pane.
a] Diamond, garnet, opal
b] Diamond, ruby, opal
c] Armanet, emerald, sapphire
d] Diamond, garnet, ruby

3] If garnet, diamond, opal are on the right side what is possible option for right side.
a] Ruby, Armanet, sapphire
b] Armanet, emerald, ruby
c] Armanet, emerald, sapphire
d] Ruby, emerald, sapphire

4] If Armanet, ruby, opal are on the left side what is possible option for left side.
a] Diamond, garnet, emerald
b] Diamond, sapphire, garnet
c] Garnet, sapphire, emerald
d] Diamond, emerald, sapphire

Approach: using the conditions eliminate the options, you will get the correct answer

(2) There is a six-faced die. Numbering should be done from 1 to 6.How many ways the
numbering can be done, with a condition that 1 and 6, 2 and 5, 3 and 4 comes on the opposite
sides of the die.

3) Temperature systems A & B


14 in A = 36 in B and 133 in A =87 in B at what temp both will be equal
Approach: try to form two simultaneous equations and solve u will get the answer as 52.5

(4) A vendor who is an ex-mathematician has a number of apples and when he arranges them in
rows of 3 he is left with one more, when he arranges them in rows of 5 again he is left with 1
more, Same happens with when he tries to arrange them in rows of 7 and 9 that is 1 apple is left
extra.
But when he arranges them in a row of 11 he is left with none. Can u tell me how many apples
were there? Ans: 946
The no is a multiple of 3,5,7,9 + a remainder of 1. So
3* 5* 7* 9= 945 + 1 =946
946 /11= 86
So it is divisible by 11 also.

(5)There is a peculiar island where people speak a strange language called 'Kubi'. The men
always speak truth and a women never say two 2 consecutive truth or false statements, that is, if
she says truth statement once then she says false statement next and vice versa. The boys and
girls always lie. One day I asked a child "Are you a boy or girl ?" and the child replied in Kubi.
Since I didn't know the language, I asked the child's parents what it means. One of the parents
said "The child says, "I am a boy" and the other said that "The child is a girl. He lied".
a) Is the child a boy or a girl?
b) Which parent answered first?
Ans: The child is a girl and father answered first.

Approach: Take the statement "The child is a girl. He lied". If 'He lied' is true, then the child is a
girl is a false. Likewise, is 'He lied' is false, then the child is a girl is true => The second
statement was given by the child's mother and hence the father answered first.
Take the statement "The child says,"I am a boy". Since the father always speak the truth, the
child should have said that he was a boy. But then the child lies, so he cannot be a boy => the
child was a girl.

(6) Jan robinson, kate jones, john smith are engineer, pilot & biochemist not in the order. they
have three martian assistants . as they find difficulty with martian names they gave them nick
names as smith, robinson, jones as their masters name.
1.robinson is an yomi
2.all martians speak uri.
3.the martians name is the same as the bio chemist is naim
4.robinson beats engineer in the chess
(one more condition) who is the pilot?
dont remember exactly..
Ans: Jan Robinson

(7) One day A and B set their watches together. Both of them are unaware that A's watch was
getting faster by 2 min per hour and B's watch was getting slower by 1 min per hour. After
sometime, they discovered that A's watch was 1 hr ahead of B's watch. How long it takes to have
a difference of 1 hr?
Ans: 20 hrs
Approach: if the present time is 5 AM. After 1 hr in A's watch the time will be 6.02 AM
And in B's watch the time will be 5.59 AM. The difference is 3 mints. So for the difference of
three mints it takes 1 hr. Therefore for the difference of 1 hr (60 mints) it will take (60*1)/3=20
hrs

(8) A horse covers half of t total distance at 12mph without load and the remaining distance at
4mph with load. What is the average speed of the horse?
Formula use: 2xy/(x+y)
X=12mph y=4mph
So average speed = 6mph

(9) Series
3,10,7,8,--,12,9,16
Ans: 11
Approach: difference between (10,7) (8,11) (12,9) is 3
Also difference between (7,8),(11,12) is 1

46656,3125,256,27,--,1
Ans: 4
Approach:
6^6=46656
5^5=3125
4^4=256
3^3=27
2^2=4

(10) There are 100 pieces of plastics, connecting two pieces are considered as one move. How
many moves are used to connect all 100 pieces?
Ans: 99 moves

1. A software engineer returns from America . As he is fat he decided to have evening


walk.....he started at 3pm . he walks along the road at 4km/hr for some time then he
climbs a upward slope area at 3km/hr then downwards at the rate of 6km/hr. then back
to the home through the road at 4km/hr. what is the distance he covered in one way if
he reaches back home at 9pm .
2. ans. 12km

approach.

let plain road is=xkm

uphill=downhill=ykm

total time of walk =9-3=6hrs

that is

x/4+y/3+y/6+x/4= 6

=>x+y=12
thus one way distance =12 km

2.a jeweller makes a window display. he has 7 gems of which he needs to display 6. three on
the left side of the pane on three on the right. with following conditions. let the gems be
Armanet, diamond, emerald , sapphire , garnet, opal and ruby. armanet should be always
displayed on left side diamond always on the right ruby should not be kept with diamond or
garnet. emerald and sapphire should always be on the same side.

1]what are the possible combinations on the possible for left pane.

a] armanet,garnet , opal

b] armanet, ruby, garnet

c] diamond, emerald , sapphire

d] armanet , emerald, garnet

2]what are the possible combinations on the possible for right pane.

a]diamond, garnet , opal

b]diamond, ruby, opal

c]armanet,emerald, sapphire

d]diamond,garnet,ruby

3]if garnet, diamond, opal are on the right side what is possible

option for right side.

a]ruby,armanet,sapphire

b]armanet,emerald,ruby
c]armanet,emerald,sapphire

d]ruby,emerald,sapphire

4]if armanet, ruby, opal are on the left side what is possible

option for left side.

a]diamond, garnet, emerald

b]diamond, sapphire, garnet

c]garnet, sapphire, emerald

d]diamond, emerald, sapphire

Ans 1]a 2]a] 3]c 4]d

3. in how many ways we can arrange numbers on a dice such that 1 and 6are opposite, 5 2nd 2 r
opposite and 4 and 3 r are opposite on a dice

ans 6*4 =24

4. complete the series 1, 2, 3, 5, 16,_

ans 231

approach

2^2-1^2=3

3^2-2^2=5

5^2-3^2=16

16^2-5^2=231
1, 2, 3, 8, -, 224

ans 27

approach

2*1+1=3

2*3+2=8

3*8+3=27

27*8+8=224

5. one watch gains 1minute/hr another watch loses 2minute/hr. in how much time the difference
of time between them would be one hour.

ans 20 hrs

approach

diff between time in 1 nour is 1+2=3 minutes thus

3 minutes diference in 1hr

60 minutes dofference in 3/60=20 hrs

6. a horse goes half of the distance with the speed 12miles/hr, and after that rest half with
luggage at 4 miles/hr. what is its average speed?

ans 6 miles/hr

approach

avg speed=distance covered/ time taken

=2x/(x/4+x/12)
=6

7.in a jigsaw puzzle there are 100 pieces. minimum how many turns r needed to complete it. at a
time you can append 2 pieces only.

ans 99

8 there r two temperature scales A and B. 14 degrees in A is eqivalent to 36 in B. and 133 in A is


eqivalent to 87 in B what is the temperature where they both are equal?

ans 52.5 degrees

approach let the temp be =X

and caliberation in A=a

caliberation in B=b

A(X-14)=B(X-36)

A(X-133)=B(X-87) thus

(x-14)(x-87)=(x-36)(x-133)

solving x=52.2

9. a man picking up letters in a way that when picks up one misses one, picks up another one
misses 2, picks up another one misses 3 ...........so on. at the end he found he picked up 5% OF
THE TOLTAL, HOW MANY LETTERS DI HE PICK?

ans 39

approach

when picked 1st.......dropped 1


2nd....... 2

3rd....... 3

----------------------------------------

picked nth.........dropped totlly sum of n natural nos thus n/{n(n+1)/2} = 5/100 =>n
=39

1. The following information is available about a group of young men in a colony. [ Marks:
8]

All handsome, fair skinned, muscular, lean, employed, and rich men are tall.

All handsome men are fair skinned.

Some muscular men are handsome.

Some muscular men are not fair skinned.

All lean men are muscular.

No lean man is handsome.

No fair skinned man who is not handsome is rich.

All tall men who are neither fair skinned nor muscular are employed.

1. pramod is not fair skinned. Which of the following must be true ?

a) pramod is employed

b) if pramod is muscular, he is neither handsome nor lean

c) if pramod is tall, he is employed or muscular.


d) if pramod is not employed, he is muscular.

e) if pramod is tall, he may be muscular or handsome, but not both.

2. which must be false if the information given is true ?

a) no lean men are fair skinned.

b) some fair skinned are lean.

c) some rich men are both fair skinned and muscular.

d) some tall men are neither fair skinned nor employed

e) some rich men are lean

3. which of the following can be deduced from the information given ?

a) all rich men are handsome

b) some rich men are handsome

c) some rich men are employed

d) some rich men are muscular

e) all rich men are handsome, muscular, or employed

4. which cannot be shown to be true or false on the basis of the information given ?

I. No fair skinned or muscular man is employed

II. Some muscular men are fair skinned but not handsome

III. No fair skinned man both handsome and lean


a) I only b) II only c) III only d) I and II e) II and III
--------------------------------------------------------------------------------------------------------------------
-

2. [ Marks : 4 ]

After gathering 770 chestnuts, three girls divided them up so that amounts were in the same
proportion as their ages. As often as mary took four chestnuts, nelli took three, and for every six
that mary received, Susie took seven. How many chestnuts did each girl get ?

--------------------------------------------------------------------------------------------------------------------
-

3. [ Marks : 5 ]

A supportive young hare and tortoise raced in opposite directions around a circular track that was
100 yards in diameter. They started at the same spot, but the hare did not move until the tortoise
had a start of one eighth of the distance ( that is, the circumference of the circle). The hare held
such a poor opinion of the other’s racing ability that he sauntered along, nibbling the grass until
he met the tortoise. At this point the hare had gone one sixth of the distance. How many times
faster than he went before must the hare now run in order to win the race ?

--------------------------------------------------------------------------------------------------------------------
-

4. [ Marks : 4 ]

Ajit was driving down the country side when he saw a farmer tending his pigs and ducks in his
yard. Ajit asked the farmer how many of each he had.

The farmer replied that there were 60 eyes and 86 feet between them.

How many ducks and how many pigs were there ?


--------------------------------------------------------------------------------------------------------------------
-5. [ Marks : 4 ]

In Mulund, the shoe store is closed every Monday, the boutique is closed every Tuesday, the
grocery store is closed every Thursday and the bank is open only on Monday, Wednesday and
Friday. Everything is closed on Sunday.

One day A, B, C and D went shopping together, each with a different place to go. They made the
following statements

A D and I wanted to go earlier in the week but there wasn’t day when we could both take care of
our errands.
B I did not want to come today but tomorrow I will not be able to do what I want to do.
C I could have gone yesterday or the day before just as well as today.
D Either yesterday or tomorrow would have suited me.

Which place did each person visit ?

--------------------------------------------------------------------------------------------------------------------
-6. [ Marks : 6 ]

The Novice hockey tournaments are on for beginners. Just three teams are in the league, and
each plays the other two teams just once. Only part of the information appears in the result chart,
which is given below.

Team Games Won Lost Tied Goals For Goals against


A 2 1 0
B 2 1 1 2
C 2

The scoring pattern in the tournament is as follows:

Two points are awarded to the winning team. In case of a tie, both teams are awarded one point,
so the total points in the standings should always equal the total number of games played ( since
each game played is counted as one for each of the two participating teams). Of course, total
goals scored for and goals scored against must be the same, since every goal scored for one team
is scored against another.

The games are played in the following order: Game 1: A Vs B; Game 2: A Vs C; Game B Vs C

Can you determine the score of each of the above games ?

--------------------------------------------------------------------------------------------------------------------
-7. [ Marks : 8 ]

A recent murder case centered around the six men, clam, flip, gront, herm, mast, and walt. In one
order or another these man were the victim, the murderer, the witness, the police, the judge, and
the hangman. The facts of the case were simple. The victim had died instantly from the effect of
gunshot wound inflicted a shot. After a lengthy trial the murderer was convicted, sentenced to
death, and hanged.

 Mast knew both the victim and the murderer.

 In court the judge asked clam his account of the shooting.

 Walt was the last of the six to see flip alive.

 The police testified that he picked up gront near the place where the body was found.

 Herm and walt never met.

What role did each of the following play in this melodrama ?

a) Murderer

b) Victim

c) Judge

d) Witness
----------------------------------------------------------------------------------------------------------------
-

8. [ Marks : 5 ]

Fodder, pepsi and cereale often eat dinner out.

a) each orders either coffee or tea after dinner.

b) if fodder orders coffee, then pepsi orders the drink that cereale orders

c) if pepsi orders coffee, then fodder orders the drink that cereale doesnot oder

d) if cereale orders tea, then fodder orders the drink that pepsi orders

which person/persons always orders the same drink after dinner ?

[ Marks : 6 ] At a recent birthday party there were four mothers and their children. Aged
1,2,3 and 4. from the clues below can you work out whose child is whose and their relevant
ages ?

 It was jane’s child’s birthday party.

 Brian is not the oldest child.

 Sarah had Anne just over a year ago.

 Laura’s Child will be next birthday.

 Daniel is older than Charlie is.

 Teresa’s child is the oldest.

 Charlie is older than Laura’s child.


KANBAY:

SEC-1s

1.IF rose code as 6786 hot coded as 879 then search will code as in this q there is one no. is
assigned t each letter.so take search and letter by letter see the given condition.

2.600,180,54........... complete the series.

3.sedane:pain :: solace:........
ans: grief.

4.play:director :: news
ans: editor.

5.river:dam :: traffic
(a)signal (b)motin (c)vehicle (d)

6.find the greatest no.


(a)half 50%of 50 (b)3times 40%of 40
see the obtion & place 3times 40% of 40 blindly.

7.find the compound interest of 1000 rs. at the rate of 5% p.a. for 15 years.

8.find the greatest of 1000power of 1000,1001power of 999

9.product of two no is constt. if one no. is increased by 50% then other no is decreased how
much.
ans: 33.3
10.l.c.m & h.cf of two no is 84 & 21 respectively and the ratio of these two no is 1:4 find
the gretest no
ans: 84 place blindly.

11.if x is 90% of y then y is how much % of x.

12.the cost of 15 apples & 20 mangoes is much as equal to the 15 mangoes & 20 apple then what
is the relation between their cost..
(a)apple is as much equal as mangoes.place blindly (a) obtion.

13.there r 20 men and 5 women then how much maximum couple can be made.....
ans: 20c1*5c1=100. place 100 blindly.

14.a bag contains 8 white and 6 black balls find the prob. of drawing a white ball....
ans: 4/7 place blindly.

15.if numerator is less than 2 by denominator and then if 1 is subtract from the numerator
and 1 is added to the denominator then the ratio becomes half what is the no.....
ans: 5/7 place blindly.

16.if a certain money becomes twice in 5 years.then the 300 rs. will become 2400 at the same
rate in how many years........

17.if the average of three numbers is 135.and the difference between others is 25 then find the
lowest no.

18.if the thrice of three consecutive odd no is equal to the more three of twice the last no.then
find the 3rd (largest odd no).
ans: 11.

19.there are 5 questions in each of the two section.three questions must be attempt for passing
the exam how many ways student will choose the question
ans: 100 place blindly.

20.if the lenght of the rectangle is decreased by 4 and breath is increased by 3.then it becomes
sqaure whose areas is equal to that of rectangle.what is the perimeter of the original rectangle.

21.there is a hemisphere of radius of 2 cm how much litre will be occupied in the hemisphere.
given 1 litre= 1000 cubic cm.

22.there is a water tank which has enough water to be consumed in 60 days.if there is a leakage
water stays there for 40 days.if there are more similar leakage then how lomg water will
consumed.

23.a man saves money 1000 in each year.and he gives this amount at the end of year for the
compound interest at the rate of 5% how much he will save after 25 years.

24.after giving 35.66% discount. a saler makes a loss of 11.11 rs.

25 there is an intersection question cannot remember it but the


answer is b

26.the sides of a rectangle is 100 cm and 60 cm if there is a path of 5 cm around the rectanle.it is
included in that given rectangle.find the area of that path.......

27.there is a number when diveded by 5 gives a remainder 3 and when divede by 7 gives a
remainder of 5 Ans is 33.
Q28.a train when travelling from 2 station A and B when travelling a speed of 50km/hr is late by
10min,and when travelling at 30km/hr arrives a late by 50min. Find the diatance.

29.rational no. is a.........


ans: real no. place blindly.

30.a boat is going along the stream and returning in the opposite direction of stream it travels 10
km along stream.the sppeed of stream is 3 km/hr,then find the speed of boat.

31.a boy leaves his home 15 minute before his shedule time.he takes 10 minutes to reach the bus
stop.he arrive at the stop at 8:40 am when he leaves the home.

32.A and B runing around a circle of perimeter of 1200meter(may be differ).A is runing at 210
meter/minute and bB is running 190 metre/minute in oppsite direction to each other.at wich time
they will meet.

33.a boy start to count from 32 in the descending order.and another boy start counting from 1 at
the same time and he count only the odd no.then find which no they will count at the same time.
there were 5 questions on syllogism. if inference is true then A is right if inference is false then
B is right if inference is probably true then c otherwise D

34.all the wives are women


she is women.
inference:
she is wife

35.some of the teachers are fool.


A is a teacher.
inference:
A is a fool.

36.there is a ladder and 4 persons (A,B,C,D) on the ladder. A is further up than B and C.
D is further up than A.Which is third from the bottom of the ladder?
Ans: person A.

37.A person goes to the house of Sita who is the neighbour of Gita. Amar married with Anita
who is the sister of Sita. Ashu is the father of Amar.

(a) What is the relation between Sita and Amar?

(b) What is the relation between Amar and Gita?

38.In an elevator either 12 adults or 20 children can go. If 15 children already occupied the
space , how many adults can go with children? ans: 3

39.Amit's D.O.B. is 3rd March 1980. Amit is four day's older than Sumit. The Republic Day of
1980 is Thrusday. What is the day of Sumit's D.O.B.?

40.A complete a race of 120m taking 4sec less than B. B takes one sec less than C. How much
time taken by A to complete the race?
41.A person pointing towards a lady says She is the only sister of my father. Find the no. of
childs of grandfather?

ans: can't say


42.If GEN = 9 , ABOUT = 15 then GENERAL = ?

43.If DUST is called AIR . AIR is called RAIN.RAIN is called WATER. WATER is called
COLOUR.
COLOUR is called ROAD. ROAD is called DUST.Where the Fish lives?

44.A is richer than B. B is richer than C. C is richer than D. D is richer than E. Then who is the
middle of this relation.

45.there is question to find the code of SEARCH the ans is c.There are 5 or 6 questions based on
coding and decoding.

46.A to Z. First letter A is swapped with Z, second letter B with Y, and so on. Which letter
thetenth from right.

Math's
1. S 1=={1,2,3,4}, S 2 =={A,B.C……….Z},S 3=={……………},(S1*S2)U S3==?
2. 8*7=B,how much is 6*9== ?
3. 4 to 5 questions on simplifying complex nos. equations
4. Sphere of radius 4.5 cm is divide in three spheres ,two have radius 1.5 and 2 cm,find the
other ones??
5. If COMPLEX == 81,and……………….how much is ANALYSE==?? 2-3 questions
based on similar pattern
6. Two taps a and b fill up a cistern in 2 and3 hrs ,at wat time should b be clsed if the tank
cistern is filled up in 17 minutes??
7. A doctor checks 5 patients every 3 hrs with abreak of 10 minutes between each two
check ups.how many he patients he checks in 10 hrs and 15 minutes??
8. no. of diagonals friends if vertices of a octagon are joined??
9. An equilateral triangle is formed by joining the centers of sides of a equilateral
triangle.wat is the ratio of area ,s of both the triangles??
10. A can finish the work in 3 days , B can finish the work in double efficiency,can finish
with equal efficiency of both A and B.if all work together within how many days will they
complete the work?
11. A.B,C,D stand at the edges of a square.they start moving along the square sides.after
clockwise rotation where will each be??
12. if 1 is stop,2 is run,3 is go,4 is wait,5 is walk,then would be the next stage if the following
pattern 5 5 4 3 2 1 ………….. 4 1 2 2 3 1 ? Freshersworld.com C remembers meeting B after 14
but before 18 ,but D Remembers meeting B after 16 but before 18.when did both meet B??
13. avg age of boys in a class is 12.when the age of teacher is included it is 13.wat could be
the age of teacher? 35 , 45,53,50

Round 1 Logical Test

1. Series 1-10-3-9-5-8-7-7-9-6-?-? Ans 11,5


2. Series Ans EK
3. If '+' is replaced by 'x' and .... Ans 24
4. Rahul 12th from right and 4th from left . How many men should be added in the queue to
make a group of 28. Ans 13
5. A person spent 21 pounds in a drink party. If the vodka costs twice that of beer. And
lemonade cost is 1/2 of beer. How much this person is spent for beer. Ans 6 pounds
6. What is odd in this sequence?
i) T.T. ii) Volleyball iii) Badminton ..... Ans Volleyball(check
7. Cat : Mouse Ans Bird : Worm
8. A dog grows ........ ? Ans 8(check)
9. A boy is going with a dog Lucky. He has a bottle full of water. He drinks 1/3 and gives
1/2 of rest to the dog and then he drinks 1/4 of the rest then what will the fraction of total
water remaining. Ans 1/12
10. A person goes towards north 10m, then turns to west 5m, then turns south ...... ? Ans
North
11. A person goes forward 10m , turns to his right and walks 5m, then he always turns left
5,15,15 then what will be the displacement from starting point? Ans 10 or 5(check)
12. A cuboid of dimensions 8x5x4 cm3 How many 2cm side cubes can be placed in this
cuboid? Ans 16
13. Decoding ... Ans SMILE
14. how many 2 digit prime no.s divided by 7 and remainder 2. Ans 4
15. how many 2 digit prime no.s divided by 7 and remainder 2. Ans
16. The position of A is 7th from left and position of b is 9th from right and they exchange
their each other's positions. The position of A is 11th from left.How many men? Ans 19
17. A is brother of P, B is daughter of A, C is brother of B. Who is the uncle of B? Ans P
18. Day : Night Ans Man : Woman(check)
19. HCTAM = MATCH then ELBAC = ? Ans CABLE
20. 3 problems on algorithms ie. eg.
1. A=0, B=1, C=1
2. A is replaced by C
3. B is replaced by 2A+1
4. C is replaced by A+B
5. If C>=100 goto step 7 else goto step 6
6. goto 2
7. stop.
then you have to ans. some 3 questions based on the above algorithm.
i) What is the value of A? Ans 40
ii) What is the value of B? Ans 27
iii) What is the max no. of iterations till C<= 100. Ans 3
21. A father has age thrice that of his son? After how many years will his age be double that
of his son . If son's present age is 20 years...... Ans 20 years

Round 2 after Logical test:

1. A & B are 2 men . A speaks truth 75% cases and B speaks 80%. Find the probablity of
contradicting each other. Ans 35%
2. Present population of town is 35,000 having males and females. If The population of
males is increased by 6% and if the population of females is increased by 4%, then after 1
year the population becomes 36,700.Find the number males and females. Ans18,000
3. Tickets(from 1 to 15) and the probablity of drawing two tickets without replacement
having even no.s . Ans 1/5
4. Room ( 8m * 5m) Brick Size (20cm * 10cm) Find the no. of bricks. Ans 2000
5. Length of rectangular plot is 3 times its breadth having area 3 hectare . A man walks on
its perimeter at 4 km/hr .Find the time required ?Ans 12 min
6. Equation Simultaneous Ans None of these
7. Ratio of wine and water. Ans 40
8. Circumference 6 km .3 men A, B, C respectively speeds Km/hr 3, 5/2, 5/4. Distance
travelled to meet. Ans 24 km.
9. Speed of trains A and B are 29 & 56 km/hr travelling in the same direction. Man in
slower train passes in 16 min. Find the length of the train. Ans 120m
10. Passing marks 40%.... Ans Max marks = 625
11. Max. marks 500. A gets 90% of B. B gets 25% more than C. C gets 80% of D. Marks of
A are given then find the percentage marks obtained by D. Ans = 80%
12. Carpet 2m wide. Area to be covered = 30m * 18m.Cost of the carpet = 50p per m.Find
total cost of carpeting. Ans Rs 135.
13. Carpet 2m wide. Area to be covered = 30m * 18m.Cost of the carpet = 50p per m.Find
total cost of carpeting. Ans Rs 135.
14. Square of side 100m. It has a circular fountain at the centre with radius 21m.

Logical:30questions
Math:30questions
Mark/correct question:+1

Logical

1. What is implied in the following sentence, During war the two countries formed allied.
I) There was war
II) the two countries are companions
a) I only b) II only c) I & II both d) neither I or II

2. What is the conclusion from the following sentence


A group of people are rich
A group of people are educated
I) Rich are educated
II) A group of people are neither rich or educated
a) I only b) II only c) both I & II d) none of these

3. What is implied from the following .All fathers are sons. No father is educated
I) All sons are educated
II) All fathers are uneducated
a) I only b) II only c) I & II both d) neither I or II

4. All Donkey are Elephant .All Elephants are Cat


a) All Cats are elephant
b) All Elephants are donkey
c) All cats are Donkey
d) All Donkeys are cat
i) only a ii) only b & c iii) all the above iv) a ,b&c ( i'm not sure of the answer & option)

5. 4 set of words are given out of which one is odd man ,find it?

a) sigh b) cough c) talk d) sniff


ans:c

6 Two question in which 4 option of jumble words are given find the odd man out
a) llatfoob b) rictek c) llasm d) nisnet
(Except one all are representing games football,cricket & tennis)
ans)c.

7. ( don't remember)

8. ABCD are on the 4 corner of the square each has to pass the sides of a square once
they move in clockwise direction which one is true
ans... B is south-east of D

9. On seeing a portray a person says "he is the son of my father ,my father has only one
daughter"
How many children does the person's father have
ans) cann't be determined

10. Average of three numbers A,B,C is 17 when A is replaced by D the average increases by 2
what is the value of D?
a)16 b)18 c)12 d) none of these
ans) d.

11. Mr.Mukherjee has two sons Ashok ,Sham.Mr & Mrs sharma have 2 children Rekha &
Anand
If Rekha is married to sham & they have 2 children priya & Kamal then how is anand
related to
priya

a)uncle b)maternal uncle c)brother d) none of these

12. Question from the same passage


Two cubes are kept together that it has two faces and 6 sides they are brushed with red paint
then it was cut into 25 pieces

13. How many side of the cube will have at least 2 side painted
14. How many side would not have only one side painted

15. If two days from tomorrow is Thursday then what will be 2 days before yesterday
a) Friday b) Saturday c) Sunday d) none of these

16. What is the day on 1978 Jan 12


a) Monday b) Tuesday c) Friday d) Wednesday

17.Find the number in the space


438
9?1
276
a)4 b)5 c)2 d) none of these
In the below question the Facts are not Exactly the same but it's the same type a&b play
hockey
and football .d&c play football & tennis . b&c play Cricket & tennis c&a play hockey and
tennis.
based on these 2 question like

18 Who play all the four games

19. Who play hockey ,tennis and football (i am not sure with the question)
Here if a is true then answer is c.
if both a&b are true then answer is b.
if b is true then answer is d.
if both are wrong then answer is a.

20,21 & 22 are like this type

20. If the flight starts at Hong Kong at 1 am local time it reach London 7pm find the travel time?
a) it takes 7 hours to reach from Hong Kong to London
b) in Hong Kong it is 7am local time
(i am not sure with timing and the question exactly same)

23. If Kamal is coded as IVSVT then what is VIMAL

24. Series question like 1,5,11,......... (not sure)

25 _11_11_01_01_01
a)00000 b)01110 c)10001 d)11111 like this type of question

26 like the above type another question with alphabets

27) 28) 29) 30)

Math

1. An unbiased coin is tossed 8 times what is the probability of getting at least 6 heads.

2. There are 6red ,4 white,3black balls what is the probability of drawing 2 red balls.
ans:6c2/13c2.(calculate)

3. There are two pipes p,q which can empty a container at 24hr,32hr respectively.both the pipes
are
opened simultaneously after what time should the first pipe be closed so that the container is
emptied in 16minutes.
4. A lady bought 7pineapples and 5 oranges for 38rupees.what is the cost of each pineapple.
ans:Rs.4
5. x+y=40,(1/x)+(1/y)=60.what is the value of x,y

6. n+m+p=0,n is positive,m>n.what is the value of p?


a)p<0 b)p>0 c) d)
ans:p<0 (not exactely the same.a model)
Total number of students appeared for the test 500
2 passed in maths
6 passed in english
48 passed in social science
4 failed in english
120 failed in maths
7. No of people failed totally

8.No of people attended the test

SATYAM:

Questions:

1. 2 nos: , LCM=693 , GCM=11 one no: 77 find the other ?


ans: 99
2. 5 children,youngest 3yrs old and no 2 children hve the same age ,sum of their age is
26.Find the age of eldest. 7,8,10,...
ans:8
3. second no: is twice the first no: and first no: is thrice the third no:.Their avg is 20. Find
the greatest no:
ans: 36
4. RS232 cable - bit transfer rate ?
0-50000,5000-50000.....
5. If CPU wants to stop the IO process and continue with main processing ,then which
device it uses : Control Unit,IO Divertor,Channel Divertor,None
6. connnecting 2 diff networks
ans: Gateway
7. 1. There is a 4 inch cube painted on all sides. This is cut down into of 1 inch cubes. What
is the no of cubes which have no pointed sides.
Ans: 8
8. At 6'o a clock ticks 6 times. The time between first and last ticks is 30 seconds. How long
does it tick at 12'o clock.
Ans: 66 sec.
9. Complete the series: 5, 20, 24, 6, 2, 8, ?
Ans: 12 (as 5*4=20, 20+4=24, 24/4=6, 6-4=2, 2*4=8, 8+4=12).
10. Find the values of each of the alphabets.
NOON
SOON
+MOON
JUNE
Ans: 9326
11. If a clock takes 7seconds to strike 7, how long will the same clock take to strike 10?
Ans: The clock strikes for the first time at the start and takes 7 seconds for 6 intervals-
thus for one interval time taken=7/6.
Therefore, for 10 seconds there are 9 intervals and time taken is 9*7/6=10 and 1/2
seconds.
12. An escalator is descending at constant speed. A walks down and takes 50 steps to reach
the bottom. B runs down and takes 90 steps in the same time as A takes 10 steps. How
many steps are visible when the escalator is not operating?
Ans: 150 steps
13. A chain is broken into three pieces of equal lengths containing 3 links each. It is taken to
a blacksmith to join into a single continuous one. How many links are to to be opened to
make it ?
Ans : 2.
14. There is a safe with a 5 digit number as the key. The 4th digit is 4 greater than second
digit, while 3rd digit is 3 less than 2nd digit. The 1st digit is thrice the last digit. There are
3 pairs whose sum is 11. Find the number.
Ans: 65292
15. An orange glass has orange juice and white glass has apple juice both of equal volumes.
50ml of the orange juice is taken and poured into the apple juice. 50ml from the white
glass is poured into the orange glass. Of the two quantities, the amount of apple juice in
the orange glass and the amount of orange juice in the white glass, which one is greater
and by how much?
Ans: The two quantities are equal
16. The shape in the sketch below is that of a square attached to half of a similar square.
Divide it into four equal pieces
Ans: Hint : the figure can be divided into 12 equal triangles
17. Fifty minutes ago if it was four times as many minutes past three o'clock, how many
minutes is it to six o'clock?
Ans: Twenty six minutes.
18. Everyday in his business a merchant had to weigh amounts from 1 kg to 121kgs, to the
nearest kg. What are the minimum number of weight required and how heavy should they
be?
Ans: .The minimum number is 5 and they should weigh 1,3,9,27 and 81kgs.
19. A car is traveling at a uniform speed. The driver sees a milestone showing a 2-digit
number. After traveling for an hour the driver sees another milestone with the same digits
in reverse order. After another hour the driver sees another milestone
containing the same two digits. What is the average speed of the driver.
Ans: 45kmph
20. A hotel has 10 storeys. Which floor is above the floor below the floor, below the floor
above the floor, below the
floor above the fifth.
Ans: The sixth floor.
21. Albert and Fernandes have two leg swimming race. Both start from opposite ends of the
pool. On the first leg, the boys pass each other at 18 m from the deep end of the pool.
During the second leg they pass at 10 m from the shallow end of the pool. Both go at
constant speed but one of them is faster. Each boy rests for 4 seconds at the end of the
first leg. What
is the length of the pool?
Ans.
22. Shahrukh speaks truth only in the morning and lies in the afternoon, whereas Salman
speaks truth only in the afternoon. A says that B is Shahrukh. Is it morning or afternoon
and who is A - Shahrukh or Salman.
Ans: Afternoon ; A is Salman.
23. A person with some money spends1/3 for cloths, 1/5 of the remaining for food and 1/4 of
the remaining for
travel. He is left with Rs 100/- . How much did he have with him in the beginning ?
Ans: Rs 250/-
24. Ram, Shyam and Gumnaam are friends. Ram is a widower and lives alone and his sister
takes care of him. Shyam is a bachelor and his niece cooks his food and looks after his
house. Gumnaam is married to Gita and lives in large house in the same town. Gita gives
the idea that all of them could stay together in the house and share monthly
expenses equally. During their first month of living together, each person contributed
Rs.25. At the end of the month, it was found that Rs 92 was the expense so the remaining
amount was distributed equally among everyone. The distribution was such that everyone
received a whole number of Rupees. How much did each person receive?
Ans. Rs 2 (Hint: Ram's sister, Shyam's niece and Gumnaam's wife are the same person)
25. There are 3 societies A, B, C. A lent cars to B and C as many as they had already. After
some time B gave as many tractors to A and C as many as they have. After sometime c
did the same thing. At the end of this transaction each one of them had 24. Find the cars
each originally had.
Ans: A had 39 cars, B had 21 cars & C had 12 cars
26. Sam and Mala have a conversation. Sam says I am certainly not over 40 Mala says I am
38 and you are at least 5 years older than me Now Sam says you are at least 39 All the
statements by the two are false. How old are they really?
Ans: Mala = 38 yrs, Sam = 41 yrs.
27. Each alphabet stands for one digit in the following multiplication.
THIS
XIS
XFXX
XXUX
X X N X X What is the maximum value T can take?
Ans: T max value = 4
28. Grass in lawn grows equally thick and in a uniform rate. It takes 24 days for 70 cows and
60 days for 30 cows to eat the whole of the grass. How many cows are needed to eat the
grass in 96 days.?
Ans : 20
[Hint: g - grass at the beginning r - rate at which grass grows, per day y - rate at which
one cow eats grass, per day n - no of cows to eat the grass in 96 days
g + 24*r = 70 * 24
* y g + 60*r
= 30 * 60 * y
g + 96*r = n * 96
* y, Solving, n = 20. ]
29. Three criminals were arrested for shop lifting. However, when interrogated only one told
the truth in both his statements, while the other two each told one true statement and one
lie. The statements were:
ALBERT :(a) Chander passed the merchandise. (b) Bruce created the diversion.
BRUCE :(a) Albert passed the merchandise. (b) I created the diversion.
CLIVE :(a) I took the goods out of the shop. (b) Bruce passed them over.
Ans: Albert passed the goods. Bruce created the diversion. Clive took the goods out of
the shop.
30. I bought a car with a peculiar 5 digit numbered license plate which on reversing could
still be read. On reversing value is increased by 78633.Whats the original number if all
digits were different?
Ans: Only 0 1 6 8 and 9 can be read upside down. So on rearranging these digits we get
the answer as 10968
31. There N stations on a railroad. After adding X stations on the rail route 46 additional
tickets have to be printed.
Find N and X.
Ans. x=2 and N=11 ( Let initially, N(N-1) = t; After adding, (N+X)(N+X-1) = t+46;Trail
and error method )
32. Complete the Table given below: Three football teams are there. Given below is the
group table. Fill in the x's
Played Won Lost Draw Goals For Goals Against
A22xxx1
B2xx124
C2xxx37
Ans: The filled table is given below
33. Played Won Lost Draw Goals For Goals Against
A220071
B201124
C201137
34. A bird keeper has got P pigeons, M mynas and S sparrows. The keeper goes for lunch
leaving his assistant to watch the birds. Suppose p=10, m=5, s=8 when the bird keeper
comes back, the assistant informs the x birds have escaped. The bird keeper exclaims:
"Oh no! All my sparrows are gone." How many birds flew away? When the bird keeper
comes back, the assistant told him that x birds have escaped. The keeper realized that at
least 2 sparrows have escaped. What is minimum no of birds that can escape?
Ans.
35. Seven members sat around a table for three days for a conference. The member's names
were Abhishek, Amol, Ankur,
Anurag, Bhuwan ,Vasu and Vikram. The meetings were chaired by Vikram. On the first
evening members sat around the table alphabetically. On the following two nights,
Vikram arranged the seating so that he could have Abhishek as near to him
as possible and absent minded Vasu as far away as he could. On no evening did any
person have sitting next to him a person who had previously been his neighbor. How did
Vikram manage to seat everybody to the best advantage on the second and third
evenings?
Ans: Second evening: Vikram, Ankur, Abhishek, Amol, Vasu, Anurag and Bhuwan.
Third evening :Vikram, Anurag, Abhishek, Vasu, Bhuwan, Ankur, Amol.
36. Two twins have certain peculiar characteristics. One of them always lies on Monday,
Wednesday, Friday. The other always lies on Tuesdays, Thursdays and Saturdays. On the
other days they tell the truth. You are given a conversation.
Person A-- today is Sunday my name is Anil
Person B -- today is Tuesday, my name is Bill
What day is today?
Ans: Today is Tuesday.
37. Four prisoners escape from a prison. The prisoners, Mr. East, Mr. West, Mr. South, Mr.
North head towards different directions after escaping. The following information of their
escape was supplied: The escape routes were The North Road, South Road, East Road
and West Road. None of the prisoners took the road which was their namesake.
Mr. East did not take the South Road
Mr. West did not the South Road.
The West Road was not taken by Mr. East
What road did each of the prisoners take to make their escape?
Ans: Mr. East took the North Road
Mr. West took the East Road
Mr. North took the South Road
Mr. South took the West Road.
38. A hotel has two wings, the east wing and the west wing. Some east wing rooms but not
all have an ocean view. All west wing rooms have a harbor view. The charge for all
rooms is identical, except as follows : Extra charge for all harbor view rooms on or above
the 3rd floor Extra charge for all ocean view rooms except those without balcony Extra
charge for some harbor rooms on the first two floor & some east wing rooms without
ocean view but having kitchen facilities. Which of the following cannot be determined on
the basis of the information given:
I. Whether there are any rooms without a balcony for which an extra charge is imposed.
II. Whether any room without a kitchen or a view involves an extra charge.
III. Whether two extra charges are imposed for any room.
(A) I only (B) II only
(C) III only (D) II and III only
(E) I, II and III
Ans: (A)
39. Given that April 1 is Tuesday. A, B, C are 3 persons told that their farewell party was on
A - May 8, Thursday
B - May 10, Tuesday
C - June 5, Friday

Out of A, B, C only one made a completely true statement concerning date, day and
month. The other told two one told the day right and the other the date right. What is
correct date, month, day.
Ans: B - (May 10) SUNDAY , C - June 6 (Friday).

40. Answer the following questions based on the conditions from the choices A, B, C, D, E
as described below:
(A) if a definite conclusion can be drawn from condition 1
(B) if a definite conclusion can be drawn from condition 2
(C) if a definite conclusion can be drawn from condition 1 and 2
(D) if a definite conclusion can be drawn from condition 1 or 2
(E) no conclusion can be drawn using both conditions
· person 1 says N<5
· person says N>5
· person 3 says 3N>20
· person 4 says 3N>10
· person 5 says N<8
1. What is the value of N?
a) 1. No of persons who speak false being less than no of persons who tells the truth.
2. Person 2 is telling the truth.
Ans.
b) 1. no of persons telling the truth is greater than no of persons telling lies
2. person 5 is telling the truth. Ans.

Three friends divided some bullets equally. After all of them shot 4 bullets the total
number of bullets remaining is equal to the bullets each had after division. Find the
original number divided.
Ans: 18 ( Initially . x
xx
Now x-4
x-4 x-4 ) Equation is 3x-12 = x
2. A ship went on a voyage. After it had traveled 180 miles a plane started with 10 times
the speed of the ship. Find the distance when they meet from starting point.
Ans: 200miles. ( Distance traveled by plane = 1/10 distance traveled by ship + 180 )

3. Replace each letter by a digit. Each letter must be represented by the same digit and no
beginning letter of a word can be 0.
ONE
ONE
ONE
ONE
TEN
Ans: O =1, N = 8 ,E = 2, T = 7 4. In a railway station, there are two trains going. One in
the harbor line and one in the main line, each having a frequency of 10 minutes. The
main line service starts at 5 o'clock and the harbor line starts at 5.02A.M. A man goes to
the station every day to catch the first train that comes. What is the probability of the man
catching the first train?
Ans: 0.8

41. Next number in the series is: 1 , 2 , 4 , 13 , 31 , 112 , ?


Ans: 224. (No number has digits more than 4. All of them are 1 , 2, 4, 8 , 16 , 32 , 64
converted to base 5 )
Father's age is three years more than three times the son's age. After three years, father's
age will be
ten years more than twice the son's age. What is the father's present age?
Ans: 33 years.
42. Light glows for every 13 seconds . How many times did it glow between 1:57:58 and
3:20:47 am.
Ans : 383 + 1 = 384
43. From a vessel, 1/3rd of the liquid evaporates on the first day. On the second day 3/4th of
the remaining liquid evaporates. What fraction of the volume is present at the end of the
second day.
Ans: 50%

Supposing a clock takes 7 seconds to strike 7. How long will it take to strike 10?
Ans: 10 1/2 seconds.
44. There are 20 poles with a constant distance between each pole. A car takes 24 second to
reach the
12th pole. How much will it take to reach the last pole.
Ans: 41.45 seconds (Let the distance between two poles = x, Hence 11x:24::19x:? )
45. How can 1000000000 be written as a product of two factors neither of them containing
zeros
Ans: 2 power 9 x 5 power 9
46. Two trains starting at same time, one from Bangalore to Mysore and other in opposite
direction
arrive at their destination 1 hr and 4 hours respectively after passing each other. How
much faster is one train from
other? Ans: Twice
47. Every day a cyclist meets a train at a particular crossing. The road is straight before the
crossing and both are traveling in the same direction. The cyclist travels with a speed of
10kmph. One day the cyclist comes late by 25 min. and meets the train 5km before the
crossing. What is the speed of the train?
Ans: 60kmph
48. A man collects cigarette stubs and makes one full cigarette with every 8 stubs. If he gets
64 stubs how many full cigarettes can he smoke.
Ans: 8+1=9
49. The minute and the hour hand of a watch meet every 65 minutes. How much does the
watch lose or gain time and by how much?
Ans: Gains; 5/11 minutes
50. A survey was taken among 100 people to find their preference of watching T. V.
programs. There are 3 channels. Given the no of people who watch
at least channel 1
at least channel 2
at least channel 3
no channels at all
at least channels 1and 3
at least channels 1 and 2
at least channels 2 and 3

Find the no of people who watched all three.


Ans.
51. Two trains start from stations A and B spaced 50kms apart at the same time and speed.
As the trains start, a bird flies from one train towards the other and on reaching the
second train, it flies back to the first train. This is repeated till the trains collide. If the
speed of the trains is 25 km/h and that of the bird is 100km/h. How much did the bird
travel till the collision.
Ans: 100kms.
52. Four persons A, B, C and D are playing cards. Each person has one card, laid down on
the table below him,
which has two different colors on either side. The colors visible on the table are Red,
Green, Red and Blue. They see the color on the reverse side and give the following
comment.
A: Yellow or Green
B: Neither Blue nor Green
C: Blue or Yellow
D: Blue or Yellow
Given that out of the 4 people 2 always lie find out the colors on the cards each person.
Ans.
53. Sometime after 10:00 PM a murder took place. A witness claimed that the clock must
have stopped at the time of the shooting. It was later found that the position of both the
hands were the same but their positions had interchanged. Tell the time of the shooting
(both actual and claimed).
Ans: Time of shooting = 11:54 PM Claimed Time = 10:59 PM
54. Some statements are given below:
L says all of my other four friends have money
M says that P said that exactly one among them has money
N says that L said that precisely two among them have money
O says that M said that three of the others have money
P, L and N said that they have money All the above statement are false. Who has money
& who doesn't have any money?

55. The Bulls, Pacers, Lakers and Jazz ran for a contest. Anup, Sujit, John made the
following statements regarding results.
Anup said either Bulls or Jazz will definitely win
Sujit said he is confident that Bulls will not win
John said he is confident that neither Jazz nor Lakers will win

56. When the result came, it was found that only one of the above three had made a correct
statement. Who has made the correct statement and who has won the contest.
Ans: Sujith; Lakers
57. There are five persons with surnames Mukherjee, Misra, Iyer, Patil and Sharma. There
are 4 persons having first or middle name of Kumar, 3 persons with Mohan, 2 persons
with Dev and 1 Anil. Either Mukherjee and Patil have a first or middle name of Dev or
Misra and Iyer have their first or middle name of Dev. Of Mukherkjee and Misra, either
both of them have a first or middle name of Mohan or neither have a first or middle name
of Mohan. Either Iyer or Sharma has a first or middle name of Kumar but not both. Who
has the first or middle name of Anil?
Ans: Kumar Misra Dev, Mohan Iyer Dev, Kumar Patil Mohan, Mohan Sharma Kumar
58. Ann, Boobie, Cathy and Dave are at their monthly business meeting. Their occupations
are author, biologist, chemist and doctor, but not necessarily in that order. Dave just told
the biologist that Cathy was on her way with doughnuts. Ann is sitting across from the
doctor and next to the chemist. The doctor was thinking that Boobie was a goofy name
for parent's
to choose, but didn't say anything. What is each person's occupation?
Ans: Since Dave spoke to the biologist and Ann sat next to the chemist and across the
doctor, Cathy must be the author and Ann the biologist. The doctor didn't speak, but
David did, so Bobbie is the doctor and Dave the chemist.
59. There are 6 volumes of books on a rack kept in order ( i.e. vol.1, vol. 2 and so on ). Give
the position after the following changes were noticed.
All books have been changed
Vol.5 was directly to the right of Vol.2
Vol.4 has Vol.6 to its left and both weren't at Vol.3's place
Vol.1 has Vol.3 on right and Vol.5 on left
An even numbered volume is at Vol.5's place
Find the order in which the books are kept now.
Ans: 2 , 5 , 1 , 3 , 6 , 4
60. A soldier looses his way in a thick jungle. At random he walks from his camp but
mathematically in an interesting fashion. First he walks one mile East then half mile to
North. Then 1/4 mile to West, then 1/8 mile to South and so on making a loop. Finally
how far he is from his camp and in which direction.

Ans: Distance traveled in north and south directions: 1/2 - 1/8 + 1/32 - 1/128 + 1/512
- ............. = 1/2/((1-(-1/4))
Similarly in east and west
directions:
1- 1/4 + 1/16 - 1/64 + 1/256 - ................... = 1/(( 1- ( - 1/4)) Add both the answers
61. Conversation between two mathematicians: First : I have three children. The product of
their ages is 36. If you sum their ages, it is exactly same as my neighbor's door number
on my left. The second mathematician verifies the door number and says that it is not
sufficient. Then the first says " Ok one more clue is that my youngest is really the
youngest". Immediately the second mathematician answers. Can you answer the question
asked by the first mathematician? What are the children ages?
Ans 1,6 and 6
62. 500 men are arranged in an array of 10 rows and 50 columns according to their heights.
Tallest among each
row of all are asked to fall out. And the shortest among them is A. Similarly after
resuming that to their original positions that the shortest among each column are asked to
fall out. And the tallest among them is B. Now who is taller among A and B ?
Ans. A
63. There are six boxes containing 5 , 7 , 14 , 16 , 18 , 29 balls of either red or blue in color.
Some boxes contain only red balls and others contain only blue. One sales man sold one
box out of them and then he says, " I have the same number of red balls left out as that of
blue ". Which box is the one he sold out ?
Ans: Total no of balls = 89 and (89-29 /2) = 60/2 = 30 and also 14 + 16 = 5 + 7 + 18 =
30
64. Ram Singh goes to his office in the city, every day from his suburban house. His driver
Gangaram drops him at the railway station in the morning and picks him up in the
evening. Every evening Ram Singh reaches the station at 5 O' Clock. Gangaram also
reaches at the same time. One day Ram Singh started early from his office and came to
the station at 4 O' Clock. Not
wanting to wait for the car he starts walking home. Mangaram starts at normal time, picks
him up on the way and takes him back house, half an hour early. How much time did
Ram Singh walk?
65. A family X went for a vacation. Unfortunately it rained for 13 days when they were there.
But whenever it rained in the mornings, they had clear afternoons and vice versa. In all
they enjoyed 11 mornings and 12 afternoons. How many days did they stay there totally?
Ans: 18
66. There are N coins on a table and there are two players A & B. You can take 1 or 2 coins
at a time. The person who takes the last coin is the loser. A always starts first.
If N=7
(a) A can always win by taking two coins in his first chance
(b) B can win only if A takes two coins in his first chance.
(c) B can always win by proper play
(d) none of the above
67. A can win by proper play if N is equal to
(a) 13 (b) 37 (c) 22 (d) 34 (e) 48
Ans: (e.)
68. B can win by proper play if N is equal to
(a) 25 (b)26 (c) 32 (d) 41 (e) none
69. if N<4, can A win by proper play always?
(a) Yes (b) No
70. Mr. Mathurs jewels have been stolen from his bank locker. The bank has lockers of 12
people which are
arranged in an array of 3 rows and 4 columns like:
1234
5678
9 10 11 12
· The locker belonging to JONES was to the right of BLACK'S locker and directly above
MILLAR'S.
· BOOTH'S locker was directly above MILLAR'S.
· SMITH'S locker was also above GRAY's (though not directly).
· GREEN'S locker was directly below SMITH'S.
· WILSON'S locker was between that of DAVIS and BOOTH.
· MILLAR'S locker was on the bottom row directly to the right of HERD'S.
· WHITE'S locker was on the bottom right hand corner in the same column as BOOTH'S.
Which box belonged to Mr. Mathurs?
Ans: Box number 9 belongs to Mr. Mathurs.
71. Five people A ,B ,C ,D ,E are related to each other. Four of them make one true statement
each as follows.
(i) B is my father's brother.
(ii) E is my mother-in-law.
(iii) C is my son-in-law's brother
(iv) A is my brother's wife.
Ans: (i) D (ii) B (iii)
E (iv) C

1.Avarage weight of p,q,r is given (cannot remember 48 i think) the ratio of students
in class p&q is 3:4.no of students in r is 25% more than p.Find the avarage of all the 3 classes.

2. Odd man out. 1307, 8627 ,6792 ,7297.


Numbers were not exact but in this model

3. Can't remember

4. Pie chart problem. One chart was given on the expenditures of a person.His total income
is 6000/- .Now his income is 1500/- we have to find the ratio of radii of two pie charts.
a. 2:1 b.sqrt(2):1 c.1:sqrt(2) etc.

5. A,B,C,D,E are 5 in family.A& D are unmarried women & dont work.in family there are
businessman,professor,artist.B is the brother of a & is not artist or businessman.E is husband.
Who is professor?
a)A b) B c)D d)E e)NONE OF THIS

8. Stamp combination problem. There are 5,10,15 paise stamps with a boys grandmother
and the boy has to paste an equivalent of 45 paise stamps to post a letter.The total
combinations are ?
a. 11 b. 12 etc.
9. Some long series of sentences were there I haven’t read them.Some reasoning type.

10. 3 work completion times were given regarding project preperation in hours by 3 teams and if
each team person
is equicapable then find the time or days in which the complete some 18 hours or days work
or project something
like that not sure.

11. Algorithm problem

Let R=2;
Let R=R+2;
Let K=K+1;
Let K=K*R;
Printf K
Let J= J+K;
If R <= 8
Goto step 2;
End.

Which of the fallowing output is not true.

a. 4 b. 36 etc

12. Simple problem on Simple Interest.


If the person gets one fifth of th sum after lending it for 8 years find the rate of interest

a. 2 ½ etc

13.Pets problem analytical one easy one.


Three persons sarada,sravani and suchitra have pets.The pets are pet1,pet2,pet3 They buy them
in the months March,August ,and December.Sarada is busy in the month august and sravani
baugt pet3 in December and Suchitra will buy in the month august but wont buy pet1.
What pet and which one will Sarada buy. Question is not exact,its only model

14. related to R.S Aggarwal Line-Graphs problems

15.Boats and streams


Given stram velocity x ,boat goes down stream by 30 KM and comes back.Total time is 4:30
Hours.Find the speed of the boat.

1)Avarage weight of p,q,r is given (cannot remember) the ratio of students in class p&q is 3:4.no
of students in r is 25% more than p.Find the avarage of all the 3 classes.
2)Odd man out(5 options where given)
3)A,B,C,D,E are 5 in family.A& D are unmarried women & dont work.in family there are
businessman,professor,artist.B is the brother of a & is not artist or businessman.E is
husband.Who is professor?a)Ab)Bc)Dd)Ee)NONE OF THIS
4)pie chat is given for a salary 6000 what will be the ratio of radius with a salary of
1500.a)2:1b)1:2c)3:2d)none of these.
5)Sum in S.I. becomes 5 times its pincipal in 8 years what is the rate?a)5/2b)6/3c)d)none of
these.

Find the area of the quadrilateral if the length of the diagonal is 30cms and the length of the lines
intersecting the diagonal at right angles are 20cms and25cms?

2.some series question

3.some reasoning question which was based on the population and the habitat of the people and
it was easy.

4.question on vocabulary.
Jumbled words were given and we need to find the correct order
This was also a bit easy.

5.A question on compound interest.Something like


A man borrowed Rs X from 2 money lenders for 6 months with an interest of Rs Y .
and Rs Z.Finally he had to clear a debt which was double the amount that he borrowed.so finally
what was the compound interest that has been gained by the lender?

6.An empty fuel tank of a car was filled with A type petrol.when the tank was half empty,it wass
filled with B type petrol.again when the tank was half-empty,it was filled with A type
petrol.when the tank was half empty again,it was filled with B type petrol.what is the percentage
of A type petrol at present in the tank?

7.A question on time and work.(Numbers were different but the model was exact)
Something like A,B and C can complete a work in 10 days.A worked for 2 days with B andC
who can complete the work in 7days .while the remaining work was completed by B and C in 5
days.how many days does B take to complete the work alone?

1) If length of a rectangle is more than its breadth by 10cm. Perimeter of a rectangle is 84.
Find the Length of rectangle.
a) 16cm
b) 26cm
c) 32cm
d) 25cm
e) None of above

Ans: b.
Let breadth= x, then leanth=10+x
Now perimeter is 84
Therefore 2(10+x+x)=84. u will get the ans
2) Simple Interest and Compound Interest were given and sum was asked to find out.

3) A brief passage was given and a statement which will support the paragraph was
asked(from given option)

4) Workman work on a particular jog place. They take lunch outside on workdays when
canteen doesn’t serve them fish. They take lunch on workdays when canteen serves them fish.
Which of the following statement is true?

a) It is a workday and canteen serves no fish


b) It is not a workday and canteen serves a fish
c) It is not a workday and canteen serves no fish
d) It is a workday and canteen serves a fish
e) None of above

Ans: d.

5) If person has a loss of 10% on a particular product and a profit of 20% on another product.
What is the total amount of profit/loss he got?
a) 260%
b) 60%
c) 150%
d) 0%
e) 250%

Ans: a.
6) Find the area question.

7) Permeability question(quite difficult)


8) ?
126 161
63 71 79
25 29 33 37
8 10 12 14 16
1 2 3 4 5 6

a) 220
b) 226
c) 227
d) 300
e) None of above

Ans:d.

(1+2)+5=8, (2+3)+5=10
(8+10)+7=25, (10+12)+7=29
(25+29)+9=63, (29+33)+9=71
(63+71)+11=126
(126+161)+13=300

9) T=T+2;
T1=T+2;
T2=T2+10;
T3=T2*4;
T5=T3+6;
T6=T5*5;
T7=T5/T6;
If T7<6
Then go to T1
T=T+2
What is the final output?
a) 4
b) 6
c) 7
d) 32
e) None of above

Ans:e
T2 is not defined

10) One side of triangle is 4cm. What is the area? Which of the following is true
1. triangle is equatorial
2. triangle is selegant

a) only (1) is sufficient


b) only (2) is sufficient
c) (1) and (2) both are sufficient
d) (1)and(2) both are not sufficient
e) None of the above

Ans:a

11) Problem on clock

12) What is the day on 15 aug 1947.


a) Tuesday
b) Monday
c) Friday
d) Sunday
e) Saturday
Ans: c.

1.complete the series


3,10,29,__,127,218
{Hint:(1^3)+2,(2^3)+2,(3^3)+2,(4^3)+2,(5^3)+2,....}
***(every time 1Q on calenders)
2.Problem on dates(calender problem)
what is the day on 25th september 1936(not exactly same)
3.Four diagrams were given as sequence and we have to find the fifth from given options
4.An equilateral triangular base prism of base ___units and length____units is melt into a cube
Find the Lateral surface area of cube

5.Given compound interest on certain amount for 2years as Rs.___ and its simple interest as
Rs.___
Find the Compound interest for 3years.
6.One problem on algorithm
10 Let R=3;
20 Let R=R+2;
30 Let R=R*2;
40 Let k=R^2;
50 print k-R;
60 check R<32 goto 20
what is the final o/p

7.A passage was given and summary of it was asked in options select the correct one
8.There was a work when done by 6 men and 7 women is completed in 6 days.The same work
when done by 3 men and
4 women is completed in 15days.Find the no. of days the work is finished in when done by 5men
and 8 women
(Not the same numbers).
{Hint:let capacity of men=x and women=y;Then 6x+7y=(1/6) and 3x+4y=(1/15).Find x and y
and substitute in
5x+8y=?}
9.A NEWS article on AIR services was given and 4 statements as 4 options were given and we
have tu choose
the most appropriate option.
10.ASSERTION and REASONING type question
two statements A and B were given
a problem is given
To describe it
a>both A and B are needed
b>A is sufficient
c>B is sufficient
d>A and B are not sufficient

SYNTEL:

Questions:

1.Avarage weight of p,q,r is given (cannot remember 48 i think) the ratio of students
in class p&q is 3:4.no of students in r is 25% more than p.Find the avarage of all the 3 classes.

2. Odd man out. 1307, 8627 ,6792 ,7297.


Numbers were not exact but in this model

3. Pie chart problem. One chart was given on the expenditures of a person.His total income
is 6000/- .Now his income is 1500/- we have to find the ratio of radii of two pie charts.
a. 2:1 b.sqrt(2):1 c.1:sqrt(2) etc.

4. A,B,C,D,E are 5 in family.A& D are unmarried women & dont work.in family there are
businessman,professor,artist.B is the brother of a & is not artist or businessman.E is husband.
Who is professor?
a)A b) B c)D d)E e)NONE OF THIS

5. Stamp combination problem. There are 5,10,15 paise stamps with a boys grandmother
and the boy has to paste an equivalent of 45 paise stamps to post a letter.The total
combinations are ?
a. 11 b. 12 etc.

6. Some long series of sentences were there I haven’t read them.Some reasoning type.

7. 3 work completion times were given regarding project preperation in hours by 3 teams and if
each team person
is equicapable then find the time or days in which the complete some 18 hours or days work
or project something
like that not sure.

8. Algorithm problem

Let R=2;
Let R=R+2;
Let K=K+1;
Let K=K*R;
Printf K
Let J= J+K;
If R <= 8
Goto step 2;
End.
Which of the fallowing output is not true.

a. 4 b. 36 etc

9. Simple problem on Simple Interest.


If the person gets one fifth of th sum after lending it for 8 years find the rate of interest

a. 2 ½ etc

10.Pets problem analytical one easy one.


Three persons sarada,sravani and suchitra have pets.The pets are pet1,pet2,pet3 They buy them
in the months March,August ,and December.Sarada is busy in the month august and sravani
baugt pet3 in December and Suchitra will buy in the month august but wont buy pet1.
What pet and which one will Sarada buy. Question is not exact,its only model

11. related to R.S Aggarwal Line-Graphs problems

12.Boats and streams


Given stram velocity x ,boat goes down stream by 30 KM and comes back.Total time is 4:30
Hours.Find the speed of the boat.

1.Find the area of the quadrilateral if the length of the diagonal is 30cms and the length of the
lines intersecting the diagonal at right angles are 20cms and25cms?

2.some series question

3.some reasoning question which was based on the population and the habitat of the people and
it was easy.
4.question on vocabulary.
Jumbled words were given and we need to find the correct order
This was also a bit easy.

5.A question on compound interest.Something like


A man borrowed Rs X from 2 money lenders for 6 months with an interest of Rs Y .
and Rs Z.Finally he had to clear a debt which was double the amount that he borrowed.so finally
what was the compound interest that has been gained by the lender?

6.An empty fuel tank of a car was filled with A type petrol.when the tank was half empty,it wass
filled with B type petrol.again when the tank was half-empty,it was filled with A type
petrol.when the tank was half empty again,it was filled with B type petrol.what is the percentage
of A type petrol at present in the tank?

7.A question on time and work.(Numbers were different but the model was exact)
Something like A,B and C can complete a work in 10 days.A worked for 2 days with B andC
who can complete the work in 7days .while the remaining work was completed by B and C in 5
days.how many days does B take to complete the work alone?

8.A question on the number pyramid.


It was a bit tough.

9.some reasoning question related to women and children.


It was quite easy and can be completed in time.
10.A question on clocks.
The minutes hand gains 45% at 11.20am and looses 30% at 11.20pm.so finally what is the
percentage of the time gained by the minutes hand.

11.A question on figures.


It requires a bit of logical thinking.it was also a easy one.
12.A problem on time and distance.

13.A problem on partnership.

14,15.These were also the reasoning questions.

1. one calender problem like-- If 13th feb 2004 is sunday then wat day is 13th feb 1884.
This problem is solved in Aggarwal book.
2.The sum of compound interest and simple interest is (some value- sorry forgot).Find
the sum.
This prolem is solved in Aggarwal book.(as seen from many previous papers of
Satyam,calender and
compound interest problem has appeared many times)
3.Problem on complete the series.(this was difficult. sorry do not remember the values)
4.One reasoning problem- 4 figures are given,find the next one.(this was simple)
5.One small phrase and questions on that.
6.There are 3 boys A,B,C and 3 girls P,Q,R.they all plan to study together by meeting at
each's place each day.
always they meet at B's place before R's place in the week. They meet at boys and girls
place on alternate day basis.
if they meet A's place on Monday then which of the following not true.

1) If length of a rectangle is more than its breadth by 10cm. Perimeter of a rectangle is 84.
Find the Length of rectangle.
a) 16cm
b) 26cm
c) 32cm
d) 25cm
e) None of above
Ans: b.
Let breadth= x, then leanth=10+x
Now perimeter is 84
Therefore 2(10+x+x)=84. u will get the ans

2) Simple Interest and Compound Interest were given and sum was asked to find out.

3) A brief passage was given and a statement which will support the paragraph was
asked(from given option)

4) Workman work on a particular jog place. They take lunch outside on workdays when
canteen doesn’t serve them fish. They take lunch on workdays when canteen serves them fish.
Which of the following statement is true?

a) It is a workday and canteen serves no fish


b) It is not a workday and canteen serves a fish
c) It is not a workday and canteen serves no fish
d) It is a workday and canteen serves a fish
e) None of above

Ans: d.

5) If person has a loss of 10% on a particular product and a profit of 20% on another product.
What is the total amount of profit/loss he got?
a) 260%
b) 60%
c) 150%
d) 0%
e) 250%
Ans: a.
6) Find the area question.

7) Permeability question(quite difficult)

8) ?
126 161
63 71 79
25 29 33 37
8 10 12 14 16
1 2 3 4 5 6

a) 220
b) 226
c) 227
d) 300
e) None of above

Ans:d.

(1+2)+5=8, (2+3)+5=10
(8+10)+7=25, (10+12)+7=29
(25+29)+9=63, (29+33)+9=71
(63+71)+11=126
(126+161)+13=300

9) T=T+2;
T1=T+2;
T2=T2+10;
T3=T2*4;
T5=T3+6;
T6=T5*5;
T7=T5/T6;
If T7<6
Then go to T1
T=T+2
What is the final output?
a) 4
b) 6
c) 7
d) 32
e) None of above

Ans:e
T2 is not defined

10) One side of triangle is 4cm. What is the area? Which of the following is true
1. triangle is equatorial
2. triangle is selegant

a) only (1) is sufficient


b) only (2) is sufficient
c) (1) and (2) both are sufficient
d) (1)and(2) both are not sufficient
e) None of the above

Ans:a

11) Problem on clock

12) What is the day on 15 aug 1947.


a) Tuesday
b) Monday
c) Friday
d) Sunday
e) Saturday

Ans: c.

13) Problem on ages(simple one)

1. time and distance ( 2 bullets fired in 38 minutes gap. It was traveling in 330 m/s.
A man traveling in a car heard the sound in 36 minutes delay. What is the speed of the car?
2. letter encoding (ex: XYZ -> ABC then PQR -> ? , like this(but easy))

3. Given some number as input and they given 5 set of output strings. For a new set of input
strings we have to find the output.

4. clock problem

5. Reasoning (relation between statements)

6. Reasoning (relation between statements)

7. compound interest.

8. pyramid problem.

9. finding the average salary

10. Encoding binary system.

11. Figure problem. Three figures given. Find the fourth one.

1.complete the series


3,10,29,__,127,218
{Hint:(1^3)+2,(2^3)+2,(3^3)+2,(4^3)+2,(5^3)+2,....}
***(every time 1Q on calenders)
2.Problem on dates(calender problem)
what is the day on 25th september 1936(not exactly same)
3.Four diagrams were given as sequence and we have to find the fifth from given options
4.An equilateral triangular base prism of base ___units and length____units is melt into a cube
Find the Lateral surface area of cube

5.Given compound interest on certain amount for 2years as Rs.___ and its simple interest as
Rs.___
Find the Compound interest for 3years.
6.One problem on algorithm
10 Let R=3;
20 Let R=R+2;
30 Let R=R*2;
40 Let k=R^2;
50 print k-R;
60 check R<32 goto 20
what is the final o/p

7.A passage was given and summary of it was asked in options select the correct one
8.There was a work when done by 6 men and 7 women is completed in 6 days.The same work
when done by 3 men and
4 women is completed in 15days.Find the no. of days the work is finished in when done by 5men
and 8 women
(Not the same numbers).
{Hint:let capacity of men=x and women=y;Then 6x+7y=(1/6) and 3x+4y=(1/15).Find x and y
and substitute in
5x+8y=?}
9.A NEWS article on AIR services was given and 4 statements as 4 options were given and we
have tu choose
the most appropriate option.
10.ASSERTION and REASONING type question
two statements A and B were given
a problem is given
To describe it
a>both A and B are needed
b>A is sufficient
c>B is sufficient
d>A and B are not sufficient

1. 2 nos: , LCM=693 , GCM=11 one no: 77 find the other ?


ans: 99
2. 5 children,youngest 3yrs old and no 2 children hve the same age ,sum of their age is
26.Find the age of eldest. 7,8,10,...
ans:8
3. second no: is twice the first no: and first no: is thrice the third no:.Their avg is 20. Find
the greatest no:
ans: 36
4. RS232 cable - bit transfer rate ?
0-50000,5000-50000.....
5. If CPU wants to stop the IO process and continue with main processing ,then which
device it uses : Control Unit,IO Divertor,Channel Divertor,None
6. connnecting 2 diff networks
ans: Gateway
7. 1. There is a 4 inch cube painted on all sides. This is cut down into of 1 inch cubes. What
is the no of cubes which have no pointed sides.
Ans: 8
8. At 6'o a clock ticks 6 times. The time between first and last ticks is 30 seconds. How long
does it tick at 12'o clock.
Ans: 66 sec.
9. Complete the series: 5, 20, 24, 6, 2, 8, ?
Ans: 12 (as 5*4=20, 20+4=24, 24/4=6, 6-4=2, 2*4=8, 8+4=12).
10. Find the values of each of the alphabets.
NOON
SOON
+MOON
JUNE
Ans: 9326
11. If a clock takes 7seconds to strike 7, how long will the same clock take to strike 10?
Ans: The clock strikes for the first time at the start and takes 7 seconds for 6 intervals-
thus for one interval time taken=7/6.
Therefore, for 10 seconds there are 9 intervals and time taken is 9*7/6=10 and 1/2
seconds.
12. An escalator is descending at constant speed. A walks down and takes 50 steps to reach
the bottom. B runs down and takes 90 steps in the same time as A takes 10 steps. How
many steps are visible when the escalator is not operating?
Ans: 150 steps
13. A chain is broken into three pieces of equal lengths containing 3 links each. It is taken to
a blacksmith to join into a single continuous one. How many links are to to be opened to
make it ?
Ans : 2.
14. There is a safe with a 5 digit number as the key. The 4th digit is 4 greater than second
digit, while 3rd digit is 3 less than 2nd digit. The 1st digit is thrice the last digit. There are
3 pairs whose sum is 11. Find the number.
Ans: 65292
15. An orange glass has orange juice and white glass has apple juice both of equal volumes.
50ml of the orange juice is taken and poured into the apple juice. 50ml from the white
glass is poured into the orange glass. Of the two quantities, the amount of apple juice in
the orange glass and the amount of orange juice in the white glass, which one is greater
and by how much?
Ans: The two quantities are equal
16. The shape in the sketch below is that of a square attached to half of a similar square.
Divide it into four equal pieces
Ans: Hint : the figure can be divided into 12 equal triangles
17. Fifty minutes ago if it was four times as many minutes past three o'clock, how many
minutes is it to six o'clock?
Ans: Twenty six minutes.
18. Everyday in his business a merchant had to weigh amounts from 1 kg to 121kgs, to the
nearest kg. What are the minimum number of weight required and how heavy should they
be?
Ans: .The minimum number is 5 and they should weigh 1,3,9,27 and 81kgs.
19. A car is traveling at a uniform speed. The driver sees a milestone showing a 2-digit
number. After traveling for an hour the driver sees another milestone with the same digits
in reverse order. After another hour the driver sees another milestone
containing the same two digits. What is the average speed of the driver.
Ans: 45kmph
20. A hotel has 10 storeys. Which floor is above the floor below the floor, below the floor
above the floor, below the
floor above the fifth.
Ans: The sixth floor.
21. Albert and Fernandes have two leg swimming race. Both start from opposite ends of the
pool. On the first leg, the boys pass each other at 18 m from the deep end of the pool.
During the second leg they pass at 10 m from the shallow end of the pool. Both go at
constant speed but one of them is faster. Each boy rests for 4 seconds at the end of the
first leg. What
is the length of the pool?
Ans.
22. Shahrukh speaks truth only in the morning and lies in the afternoon, whereas Salman
speaks truth only in the afternoon. A says that B is Shahrukh. Is it morning or afternoon
and who is A - Shahrukh or Salman.
Ans: Afternoon ; A is Salman.
23. A person with some money spends1/3 for cloths, 1/5 of the remaining for food and 1/4 of
the remaining for
travel. He is left with Rs 100/- . How much did he have with him in the beginning ?
Ans: Rs 250/-
24. Ram, Shyam and Gumnaam are friends. Ram is a widower and lives alone and his sister
takes care of him. Shyam is a bachelor and his niece cooks his food and looks after his
house. Gumnaam is married to Gita and lives in large house in the same town. Gita gives
the idea that all of them could stay together in the house and share monthly
expenses equally. During their first month of living together, each person contributed
Rs.25. At the end of the month, it was found that Rs 92 was the expense so the remaining
amount was distributed equally among everyone. The distribution was such that everyone
received a whole number of Rupees. How much did each person receive?
Ans. Rs 2 (Hint: Ram's sister, Shyam's niece and Gumnaam's wife are the same person)
25. There are 3 societies A, B, C. A lent cars to B and C as many as they had already. After
some time B gave as many tractors to A and C as many as they have. After sometime c
did the same thing. At the end of this transaction each one of them had 24. Find the cars
each originally had.
Ans: A had 39 cars, B had 21 cars & C had 12 cars
26. Sam and Mala have a conversation. Sam says I am certainly not over 40 Mala says I am
38 and you are at least 5 years older than me Now Sam says you are at least 39 All the
statements by the two are false. How old are they really?
Ans: Mala = 38 yrs, Sam = 41 yrs.
27. Each alphabet stands for one digit in the following multiplication.
THIS
XIS
XFXX
XXUX
X X N X X What is the maximum value T can take?
Ans: T max value = 4
28. Grass in lawn grows equally thick and in a uniform rate. It takes 24 days for 70 cows and
60 days for 30 cows to eat the whole of the grass. How many cows are needed to eat the
grass in 96 days.?
Ans : 20
[Hint: g - grass at the beginning r - rate at which grass grows, per day y - rate at which
one cow eats grass, per day n - no of cows to eat the grass in 96 days
g + 24*r = 70 * 24
* y g + 60*r
= 30 * 60 * y
g + 96*r = n * 96
* y, Solving, n = 20. ]
29. Three criminals were arrested for shop lifting. However, when interrogated only one told
the truth in both his statements, while the other two each told one true statement and one
lie. The statements were:
ALBERT :(a) Chander passed the merchandise. (b) Bruce created the diversion.
BRUCE :(a) Albert passed the merchandise. (b) I created the diversion.
CLIVE :(a) I took the goods out of the shop. (b) Bruce passed them over.
Ans: Albert passed the goods. Bruce created the diversion. Clive took the goods out of
the shop.
30. I bought a car with a peculiar 5 digit numbered license plate which on reversing could
still be read. On reversing value is increased by 78633.Whats the original number if all
digits were different?
Ans: Only 0 1 6 8 and 9 can be read upside down. So on rearranging these digits we get
the answer as 10968
31. There N stations on a railroad. After adding X stations on the rail route 46 additional
tickets have to be printed.
Find N and X.
Ans. x=2 and N=11 ( Let initially, N(N-1) = t; After adding, (N+X)(N+X-1) = t+46;Trail
and error method )
32. Complete the Table given below: Three football teams are there. Given below is the
group table. Fill in the x's
Played Won Lost Draw Goals For Goals Against
A22xxx1
B2xx124
C2xxx37
Ans: The filled table is given below
33. Played Won Lost Draw Goals For Goals Against
A220071
B201124
C201137
34. A bird keeper has got P pigeons, M mynas and S sparrows. The keeper goes for lunch
leaving his assistant to watch the birds. Suppose p=10, m=5, s=8 when the bird keeper
comes back, the assistant informs the x birds have escaped. The bird keeper exclaims:
"Oh no! All my sparrows are gone." How many birds flew away? When the bird keeper
comes back, the assistant told him that x birds have escaped. The keeper realized that at
least 2 sparrows have escaped. What is minimum no of birds that can escape?
Ans.
35. Seven members sat around a table for three days for a conference. The member's names
were Abhishek, Amol, Ankur,
Anurag, Bhuwan ,Vasu and Vikram. The meetings were chaired by Vikram. On the first
evening members sat around the table alphabetically. On the following two nights,
Vikram arranged the seating so that he could have Abhishek as near to him
as possible and absent minded Vasu as far away as he could. On no evening did any
person have sitting next to him a person who had previously been his neighbor. How did
Vikram manage to seat everybody to the best advantage on the second and third
evenings?
Ans: Second evening: Vikram, Ankur, Abhishek, Amol, Vasu, Anurag and Bhuwan.
Third evening :Vikram, Anurag, Abhishek, Vasu, Bhuwan, Ankur, Amol.
36. Two twins have certain peculiar characteristics. One of them always lies on Monday,
Wednesday, Friday. The other always lies on Tuesdays, Thursdays and Saturdays. On the
other days they tell the truth. You are given a conversation.
Person A-- today is Sunday my name is Anil
Person B -- today is Tuesday, my name is Bill
What day is today?
Ans: Today is Tuesday.
37. Four prisoners escape from a prison. The prisoners, Mr. East, Mr. West, Mr. South, Mr.
North head towards different directions after escaping. The following information of their
escape was supplied: The escape routes were The North Road, South Road, East Road
and West Road. None of the prisoners took the road which was their namesake.
Mr. East did not take the South Road
Mr. West did not the South Road.
The West Road was not taken by Mr. East
What road did each of the prisoners take to make their escape?
Ans: Mr. East took the North Road
Mr. West took the East Road
Mr. North took the South Road
Mr. South took the West Road.
38. A hotel has two wings, the east wing and the west wing. Some east wing rooms but not
all have an ocean view. All west wing rooms have a harbor view. The charge for all
rooms is identical, except as follows : Extra charge for all harbor view rooms on or above
the 3rd floor Extra charge for all ocean view rooms except those without balcony Extra
charge for some harbor rooms on the first two floor & some east wing rooms without
ocean view but having kitchen facilities. Which of the following cannot be determined on
the basis of the information given:
I. Whether there are any rooms without a balcony for which an extra charge is imposed.
II. Whether any room without a kitchen or a view involves an extra charge.
III. Whether two extra charges are imposed for any room.
(A) I only (B) II only
(C) III only (D) II and III only
(E) I, II and III
Ans: (A)
39. Given that April 1 is Tuesday. A, B, C are 3 persons told that their farewell party was on
A - May 8, Thursday
B - May 10, Tuesday
C - June 5, Friday

Out of A, B, C only one made a completely true statement concerning date, day and
month. The other told two one told the day right and the other the date right. What is
correct date, month, day.
Ans: B - (May 10) SUNDAY , C - June 6 (Friday).
40. Answer the following questions based on the conditions from the choices A, B, C, D, E
as described below:
(A) if a definite conclusion can be drawn from condition 1
(B) if a definite conclusion can be drawn from condition 2
(C) if a definite conclusion can be drawn from condition 1 and 2
(D) if a definite conclusion can be drawn from condition 1 or 2
(E) no conclusion can be drawn using both conditions
· person 1 says N<5
· person says N>5
· person 3 says 3N>20
· person 4 says 3N>10
· person 5 says N<8
1. What is the value of N?
a) 1. No of persons who speak false being less than no of persons who tells the truth.
2. Person 2 is telling the truth.
Ans.
b) 1. no of persons telling the truth is greater than no of persons telling lies
2. person 5 is telling the truth. Ans.

Three friends divided some bullets equally. After all of them shot 4 bullets the total
number of bullets remaining is equal to the bullets each had after division. Find the
original number divided.
Ans: 18 ( Initially . x
xx
Now x-4
x-4 x-4 ) Equation is 3x-12 = x
2. A ship went on a voyage. After it had traveled 180 miles a plane started with 10 times
the speed of the ship. Find the distance when they meet from starting point.
Ans: 200miles. ( Distance traveled by plane = 1/10 distance traveled by ship + 180 )

3. Replace each letter by a digit. Each letter must be represented by the same digit and no
beginning letter of a word can be 0.
ONE
ONE
ONE
ONE
TEN
Ans: O =1, N = 8 ,E = 2, T = 7 4. In a railway station, there are two trains going. One in
the harbor line and one in the main line, each having a frequency of 10 minutes. The
main line service starts at 5 o'clock and the harbor line starts at 5.02A.M. A man goes to
the station every day to catch the first train that comes. What is the probability of the man
catching the first train?
Ans: 0.8
41. Next number in the series is: 1 , 2 , 4 , 13 , 31 , 112 , ?
Ans: 224. (No number has digits more than 4. All of them are 1 , 2, 4, 8 , 16 , 32 , 64
converted to base 5 )
Father's age is three years more than three times the son's age. After three years, father's
age will be
ten years more than twice the son's age. What is the father's present age?
Ans: 33 years.
42. Light glows for every 13 seconds . How many times did it glow between 1:57:58 and
3:20:47 am.
Ans : 383 + 1 = 384
43. From a vessel, 1/3rd of the liquid evaporates on the first day. On the second day 3/4th of
the remaining liquid evaporates. What fraction of the volume is present at the end of the
second day.
Ans: 50%

Supposing a clock takes 7 seconds to strike 7. How long will it take to strike 10?
Ans: 10 1/2 seconds.
44. There are 20 poles with a constant distance between each pole. A car takes 24 second to
reach the
12th pole. How much will it take to reach the last pole.
Ans: 41.45 seconds (Let the distance between two poles = x, Hence 11x:24::19x:? )
45. How can 1000000000 be written as a product of two factors neither of them containing
zeros
Ans: 2 power 9 x 5 power 9
46. Two trains starting at same time, one from Bangalore to Mysore and other in opposite
direction
arrive at their destination 1 hr and 4 hours respectively after passing each other. How
much faster is one train from
other? Ans: Twice
47. Every day a cyclist meets a train at a particular crossing. The road is straight before the
crossing and both are traveling in the same direction. The cyclist travels with a speed of
10kmph. One day the cyclist comes late by 25 min. and meets the train 5km before the
crossing. What is the speed of the train?
Ans: 60kmph
48. A man collects cigarette stubs and makes one full cigarette with every 8 stubs. If he gets
64 stubs how many full cigarettes can he smoke.
Ans: 8+1=9
49. The minute and the hour hand of a watch meet every 65 minutes. How much does the
watch lose or gain time and by how much?
Ans: Gains; 5/11 minutes
50. A survey was taken among 100 people to find their preference of watching T. V.
programs. There are 3 channels. Given the no of people who watch
at least channel 1
at least channel 2
at least channel 3
no channels at all
at least channels 1and 3
at least channels 1 and 2
at least channels 2 and 3

Find the no of people who watched all three.


Ans.
51. Two trains start from stations A and B spaced 50kms apart at the same time and speed.
As the trains start, a bird flies from one train towards the other and on reaching the
second train, it flies back to the first train. This is repeated till the trains collide. If the
speed of the trains is 25 km/h and that of the bird is 100km/h. How much did the bird
travel till the collision.
Ans: 100kms.
52. Four persons A, B, C and D are playing cards. Each person has one card, laid down on
the table below him,
which has two different colors on either side. The colors visible on the table are Red,
Green, Red and Blue. They see the color on the reverse side and give the following
comment.
A: Yellow or Green
B: Neither Blue nor Green
C: Blue or Yellow
D: Blue or Yellow
Given that out of the 4 people 2 always lie find out the colors on the cards each person.
Ans.
53. Sometime after 10:00 PM a murder took place. A witness claimed that the clock must
have stopped at the time of the shooting. It was later found that the position of both the
hands were the same but their positions had interchanged. Tell the time of the shooting
(both actual and claimed).
Ans: Time of shooting = 11:54 PM Claimed Time = 10:59 PM
54. Some statements are given below:
L says all of my other four friends have money
M says that P said that exactly one among them has money
N says that L said that precisely two among them have money
O says that M said that three of the others have money
P, L and N said that they have money All the above statement are false. Who has money
& who doesn't have any money?
55. The Bulls, Pacers, Lakers and Jazz ran for a contest. Anup, Sujit, John made the
following statements regarding results.
Anup said either Bulls or Jazz will definitely win
Sujit said he is confident that Bulls will not win
John said he is confident that neither Jazz nor Lakers will win
56. When the result came, it was found that only one of the above three had made a correct
statement. Who has made the correct statement and who has won the contest.
Ans: Sujith; Lakers
57. There are five persons with surnames Mukherjee, Misra, Iyer, Patil and Sharma. There
are 4 persons having first or middle name of Kumar, 3 persons with Mohan, 2 persons
with Dev and 1 Anil. Either Mukherjee and Patil have a first or middle name of Dev or
Misra and Iyer have their first or middle name of Dev. Of Mukherkjee and Misra, either
both of them have a first or middle name of Mohan or neither have a first or middle name
of Mohan. Either Iyer or Sharma has a first or middle name of Kumar but not both. Who
has the first or middle name of Anil?
Ans: Kumar Misra Dev, Mohan Iyer Dev, Kumar Patil Mohan, Mohan Sharma Kumar
58. Ann, Boobie, Cathy and Dave are at their monthly business meeting. Their occupations
are author, biologist, chemist and doctor, but not necessarily in that order. Dave just told
the biologist that Cathy was on her way with doughnuts. Ann is sitting across from the
doctor and next to the chemist. The doctor was thinking that Boobie was a goofy name
for parent's
to choose, but didn't say anything. What is each person's occupation?
Ans: Since Dave spoke to the biologist and Ann sat next to the chemist and across the
doctor, Cathy must be the author and Ann the biologist. The doctor didn't speak, but
David did, so Bobbie is the doctor and Dave the chemist.
59. There are 6 volumes of books on a rack kept in order ( i.e. vol.1, vol. 2 and so on ). Give
the position after the following changes were noticed.
All books have been changed
Vol.5 was directly to the right of Vol.2
Vol.4 has Vol.6 to its left and both weren't at Vol.3's place
Vol.1 has Vol.3 on right and Vol.5 on left
An even numbered volume is at Vol.5's place
Find the order in which the books are kept now.
Ans: 2 , 5 , 1 , 3 , 6 , 4
60. A soldier looses his way in a thick jungle. At random he walks from his camp but
mathematically in an interesting fashion. First he walks one mile East then half mile to
North. Then 1/4 mile to West, then 1/8 mile to South and so on making a loop. Finally
how far he is from his camp and in which direction.

Ans: Distance traveled in north and south directions: 1/2 - 1/8 + 1/32 - 1/128 + 1/512
- ............. = 1/2/((1-(-1/4))
Similarly in east and west
directions:
1- 1/4 + 1/16 - 1/64 + 1/256 - ................... = 1/(( 1- ( - 1/4)) Add both the answers
61. Conversation between two mathematicians: First : I have three children. The product of
their ages is 36. If you sum their ages, it is exactly same as my neighbor's door number
on my left. The second mathematician verifies the door number and says that it is not
sufficient. Then the first says " Ok one more clue is that my youngest is really the
youngest". Immediately the second mathematician answers. Can you answer the question
asked by the first mathematician? What are the children ages?
Ans 1,6 and 6
62. 500 men are arranged in an array of 10 rows and 50 columns according to their heights.
Tallest among each
row of all are asked to fall out. And the shortest among them is A. Similarly after
resuming that to their original positions that the shortest among each column are asked to
fall out. And the tallest among them is B. Now who is taller among A and B ?
Ans. A
63. There are six boxes containing 5 , 7 , 14 , 16 , 18 , 29 balls of either red or blue in color.
Some boxes contain only red balls and others contain only blue. One sales man sold one
box out of them and then he says, " I have the same number of red balls left out as that of
blue ". Which box is the one he sold out ?
Ans: Total no of balls = 89 and (89-29 /2) = 60/2 = 30 and also 14 + 16 = 5 + 7 + 18 =
30
64. Ram Singh goes to his office in the city, every day from his suburban house. His driver
Gangaram drops him at the railway station in the morning and picks him up in the
evening. Every evening Ram Singh reaches the station at 5 O' Clock. Gangaram also
reaches at the same time. One day Ram Singh started early from his office and came to
the station at 4 O' Clock. Not
wanting to wait for the car he starts walking home. Mangaram starts at normal time, picks
him up on the way and takes him back house, half an hour early. How much time did
Ram Singh walk?
65. A family X went for a vacation. Unfortunately it rained for 13 days when they were there.
But whenever it rained in the mornings, they had clear afternoons and vice versa. In all
they enjoyed 11 mornings and 12 afternoons. How many days did they stay there
totally?
Ans: 18
66. There are N coins on a table and there are two players A & B. You can take 1 or 2 coins
at a time. The person who takes the last coin is the loser. A always starts first.
If N=7
(a) A can always win by taking two coins in his first chance
(b) B can win only if A takes two coins in his first chance.
(c) B can always win by proper play
(d) none of the above
67. A can win by proper play if N is equal to
(a) 13 (b) 37 (c) 22 (d) 34 (e) 48
Ans: (e.)
68. B can win by proper play if N is equal to
(a) 25 (b)26 (c) 32 (d) 41 (e) none
69. if N<4, can A win by proper play always?
(a) Yes (b) No
70. Mr. Mathurs jewels have been stolen from his bank locker. The bank has lockers of 12
people which are
arranged in an array of 3 rows and 4 columns like:
1234
5678
9 10 11 12
· The locker belonging to JONES was to the right of BLACK'S locker and directly above
MILLAR'S.
· BOOTH'S locker was directly above MILLAR'S.
· SMITH'S locker was also above GRAY's (though not directly).
· GREEN'S locker was directly below SMITH'S.
· WILSON'S locker was between that of DAVIS and BOOTH.
· MILLAR'S locker was on the bottom row directly to the right of HERD'S.
· WHITE'S locker was on the bottom right hand corner in the same column as BOOTH'S.
Which box belonged to Mr. Mathurs?
Ans: Box number 9 belongs to Mr. Mathurs.
71. Five people A ,B ,C ,D ,E are related to each other. Four of them make one true statement
each as follows.
(i) B is my father's brother.
(ii) E is my mother-in-law.
(iii) C is my son-in-law's brother
(iv) A is my brother's wife.
Ans: (i) D (ii) B (iii)
E (iv) C
WIPRO:

Sample Test

1. An electron moving in an electromagnetic field moves in a

(a) In a straight path


(b) Along the same plane in the direction of its propagation
(c) Opposite to the original direction of propagation
(d) In a sine wave

Ans. (b)

2. The total work done on the particle is equal to the change in its kinetic energy

(a) Always
(b) Only if the forces acting on the body are conservative.
(c) Only if the forces acting on the body are gravitational.
(d) Only if the forces acting on the body are elastic.

Ans. (a)

3. The following unit measure energy:

(a) Kilo-watt hour.


(b) Volt*volt/sec*ohm.
(c) Pascal*foot*foot
(d) (Coulomb*coulomb)*farad

Ans. (a)
4. Astronauts in stable orbits around the earth are in a state of weightlessness because

(a) There is no gravitational force acting on them.


(b) The satellite and the air inside it have an acceleration equal to that of gravitational
acceleration there.
(c) The gravitational force of the earth and the sun balance giving null resultant.
(d) There is no atmosphere at the height at which the satellites move.

Ans. (b)
5. An organ pipe, open at both ends and another organ pipe closed at one end,
will resonate with each other, if their lengths are in the ratio of

(a) 1:1
(b) 1:4
(c) 2:1
(d) 1:2

Ans. (c)
6. During an isothermal expansion of an ideal gas

(a) Its internal energy increases.


(b) Its internal energy decreases.
(c) Its internal energy does not change.
(d) The work done by the gas is not equal to the quantity of heat absorbed by it.

Ans. (c)

7. A parallel plate capaciator is charged and the charging battery is then disconnected.
If the plates of the capacitor are moved further apart by means of insulating handles

(a) The charge on the capacitor increases.


(b) The voltage across the plates increases.
(c) The capacitance increases.
(d) The electrostatic energy stored in the capacitor decreases.

Ans. (b)
8. Two equal negative charges q are fixed at point (0,a) and (0,-a) on the y-axis.
A positive charge Q is released from rest at the point (2a,0) on the x-axis. The charge Q will

(a) Execute simple harmonic motion about the origin


(b) Move to the origin and remain at rest
(c) Move to infinity
(d) Execute oscillatory but not simple harmonic motion

Ans. (d)

9. A square conducting loop of length Lon a side carries a current I.


The magnetic field at the centre of the loop is

(a) Independant of L
(b) Proportional to L*L
(c) Inversely proportoinal to L
(d) Directly proportional to L

Ans. (c)
10. The focal length of a convex lens when placed in air and then in water will

(a) Increase in water with respect to air


(b) Increase in air with respect to water
(c) Decrease in water with respect to. air
(d) Remain the same

Ans. (a)
11. The maximum kinectic energy of the photoelectron emitted from the surface is dependant on

(a) The intensity of incident radiation


(b) The potential of the collector electrode
(c) The frequency of incident radiation
(d) The angle of incidence of radiation of the surface

Ans. (c)

12. An electron orbiting in a circular orbit around the nucleus of the atom

(a) Has a magnetic dipole moment


(b) Exerts an electric force on the nucleus equal to that on it by the nucleus
(c) Does not produce a magnetic induction at the nucleus
(d) All of the above

Ans. (d)

13. The X-rays beam coming from an X-ray tube will be:

(a) Monochromatic
(b) Having all wavelengths smaller than a certain minimum wavelength
(c) Having all wavelengths larger than a certain minimum wavelength
(d) Having all wavelengths lying between a minimum and a maximum wavelength

Ans. (c)

14. The mass number of a nucleus is


(a) Always less than its atomic number
(b) Always more than its atomic number
(c) Always equal to its atomic number
(d) Sometimes more and sometimes equal to its atomic number

Ans. (d)

15. Two successive elements belonging to the first transition series have the same number
of electrons partially filling orbitals. They are

(a) V and Cr
(b) Ti and V
(c) Mn and Cr
(d) Fe and Co

Ans. (c)

16. When n+l has the same value for two or more orbitals,the new electron enters the orbital
where

(a) n is maximum
(b) n is minimum
(c) l is maximum
(d) l is minimum

Ans. (b)

17. A balloon filled with ethylene is pricked with a sharp pointed needle and quickly placed in a
tank
full of hydrogen at the same pressure. After a while the balloon would have
(a) Shrunk
(b) Enlarged
(c) Completely collapsed
(d) Remain unchanged in size

Ans. (b)

18. Which of the following statements is not true?

(a) The ratio of the mean speed to the rms speed is independant of temperature
(b) Tthe square of the mean speed of the molecules is equal to the mean squared speed at a
certain temperature
(c) Mean kinetic energy of the gas molecules at any given temperature is independant of the
mean speed
(d) None

Ans. (b)

19. Which of the following statements represent Raoult's Law

(a) Mole fraction of solvent = ratio of vapour pressure of the solution to vapour pressure of the
solvent
(b) Mole fraction of solute = ratio of vapour pressure of the solution to vapour pressure of the
solvent
(c) Mole fraction of solute = lowering of vapour pressure of the solution
(d) Mole fraction of solvent = lowering of vapour pressure of the solution

Ans. (a)

20. Elements having the same atomic number and the same atomic mass are known as
(a) Isotopes
(b) Isotones
(c) Isomers
(d) None of the above

21.Which is the most acidic amongst

(a) Nitrophenol
(b) O-toulene
(c) Phenol
(d) Cresol

22. Pure water does not conduct electricity because it is

(a) Almost not ionised


(b) Low boiling
(c) Neutral
(d) Readily decomposed

Ans. (a)

23. In a salt bridge, KCl is used because

(a) It is an electrolyte
(b) The transference number of K+ and Cl¯ is nearly the same
(c) It is a good conductor of electricity
(d) All of the above

Ans. (d)
24. A depolarizer used in the dry cell batteries is

(a) KCl
(b) MnO2
(c) KOH
(d) None of the above

Ans. (b)

25. The hydrolysis of alkyl halides by aqueous NaOH is best termed as

(a) Electrophylic substitution reaction


(b) Electrophylic addition reaction
(c) Nnucleophylic addition reaction
(d) Nucleophylic substitution reaction

Ans. (d)

26. The hydrocarbon that gives a red precipitate with ammoniacal cuprous chloride is (where 'º'
means a triple bond)

(a) CH3-CH2-CH2-CH3
(b) CH3-CºC-CH3
(c) CH2=CH-CH=CH2
(d) CH3-CH2-CºCH

Ans. (d)

27. Which of the following reagents is neither neutral nor basic


(a) Lucas' reagent
(b) Tollen's reagent
(c) Bayer's reagent
(d) Fehling's solution

Ans. (a)

28. The substance which is most easily nitrated

(a) Toluene
(b) Bbenzene
(c) Nitrobenzene
(d) Chlorobenzene

Ans. (a)

29. Carbylamine reaction is a test for

(a) Primary amine


(b) Secondary amine
(c) Tertiary amine
(d) Quarternary ammonium salt

Ans. (a)

30. Which of the following oxides cannot be reduced by carbon to obtain metal

(a) ZnO
(b) Al2O3
(c) Fe2O3
(d) PbO
Ans. (b)

31. Which of the following is not an oxide ore?

(a) Cassiterite
(b) Siderite
(c) Pyrolusite
(d) Bauxite

Ans. (b)

32. Which among the following is called philosopher's wool

(a) Cellulose
(b) Calamine
(c) Stellite
(d) Cerussite

Ans. (c)

33. Out of 10 white, 9 black and 7 red balls, in how many ways can we select one or more balls

(a) 234
(b) 52
(c) 630
(d) 879

Ans. (d)

34. A and B throw a dice. The probabilty that A's throw is not greater than B's is
(a) 5/12
(b) 7/12
(c) 11/12
(d) 5/36

Ans. (b)

35. Given two numbers a and b. Let A denote the single AM between these and S denote the sum
of n AMs
between them. Then S/A depends upon

(a) n
(b) n,a
(c) n,b
(d) n,a,b

Ans. (a)

36. If the sum of the roots of the equation ax²+bx+c=0 is equal to the sum of the squares of their
reciprocals,
then, a/c, b/a, c/b are in

(a) AP
(b) GP
(c) HP
(d) None of the these

Ans. (c)

In the following questions ~ represents the integral sign-for eg. 1~2[f(x)] means integration of
the function f(x) over the interval 1 to2.
37. Value of -1~2[|2-x²|]dx, ie integration of the function |2-x²| over the interval -1 to 2.

(a) 0
(b) 1
(c) 2
(d) None of the above

Ans. (d)

38. If 0~P[log sinx]dx=k,then the value of 0~P/4[log(1 + tan x)]dx ,where P stands for pi,is

(a) -k/4
(b) k/4
(c) -k/8
(d) k/8

Ans. (c)

39. If a,b,c be in GP and p,q be respectively AM between a,b and b,c then

(a) 2/b=1/p+1/q
(b) 2/b=1/p-1/q
(c) 2=a/p-c/q
(d) None of the above

Ans. (a)

40. A solution of KMnO4 is reduced to MnO2 .The normality of solution is 0.6.The molarity is

(a) 1.8M
(b) 0.6M
(c) 0.1M
(d) 0.2M

Ans. (d)

The questions 41-46 are based on the following pattern.The problems below contain a question
and two statements giving certain data. You have to decide whether the data given in the
statements are sufficient for answering the questions.The correct answer is

(A) If statement (I) alone is sufficient but statement (II) alone is not sufficient.
(B) If statement(II) alone is sufficient but statement(I) alone is not sufficient.
(C) If both statements together are sufficient but neither of statements alone is sufficient.
(D) If both together are not sufficient.

41. What is John's age?

(I) In 15 years John will be twice as old as Dias would be


(II) Dias was born 5 years ago

Ans. (C)

42. What is the distance from city A to city C in kms?

(I) City A is 90 kms from City B


(II) City B is 30 kms from City C

Ans. (D)

43.Is A=C ? A,B,C are real numbers


(I) A-B=B-C
(II) A-2C = C-2B

Ans. (C)

44. What is the 30th term of a given sequence ?

(I) The first two terms of the sequence are 1,1/2


(II) The common difference is -1/2

Ans. (A)

45.Was Avinash early, on time or late for work?

(I) He thought his watch was 10 minutes fast


(II) Actually his watch was 5 minutes slow

Ans. (D)

46. What is the value of A if A is an integer?

(I) A4 = 1
(II) A3 + 1 = 0

Ans. (B)

47. A person travels 12 km in the southward direction and then travels 5km to the right and then
travels 15km toward the right and finally travels
5km towards the east, how far is he from his starting place?
(a) 5.5 kms
(b) 3 km
(c) 13 km
(d) 6.4 km

Ans. (b)

48. X's father's wife's father's granddaughter uncle will be related to X as

(a) Son
(b) Nephew
(c) Uncle
(d) Grandfather

Ans. (c)

49. Find the next number in the series 1, 3 ,7 ,13 ,21 ,31

(a) 43
(b) 33
(c) 41
(d) 45

Ans. (a)

50. If in a certain code "RANGE" is coded as 12345 and "RANDOM" is coded as 123678.
Then the code for the word "MANGO" would be

(a) 82357
(b) 89343
(c) 84629
(d) 82347

Ans. (d)

51. If "PROMPT" is coded as QSPLOS ,then "PLAYER" should be

(a) QMBZFS
(b) QWMFDW
(c) QUREXM
(d) URESTI

Ans. (a)

The questions 52-53 are based on the following data

6 people A,B,C,D,E and F sit around a table for dinner.Since A does not like C, he doesn't sit
either opposite or beside C.B and F always like to
sit opposite each other.

52. If A is beside F then who is are the two neighbours of B?

(a) D and C
(b) E and C
(c) D and E
(d) Either (a) or (b)

Ans. (c)

53. If D is adjacent to F then who is adjacent to C?


(a) E and B
(b) D and A
(c) D and B
(d) either (a) or (c)

Ans.(d)

54. Complete the sequence A, E ,I ,M ,Q ,U , _ , _

(a) B, F
(b) Y, C
(c) G, I
(d) K, O

Ans.(b)

55. A person travels 6km towards west, then travels 5km towards north ,then finally travels
6km towards west. Where is he with respect to his starting position?

(a) 13km east


(b) 13km northeast
(c) 13km northwest
(d) 13km west

Ans. (c)

56. If A speaks the truth 80% of the times, B speaks the truth 60% of the times.
What is the probability that they tell the truth at the same time

(a) 0.8
(b) 0.48
(c) 0.6
(d) 0.14

Ans.(b)

57. If the time quantum is too large, Round Robin scheduling degenerates to

(a) Shortest Job First Scheduling


(b) Multilevel Queue Scheduling
(c) FCFS
(d) None of the above

Ans. (c)

58. Transponders are used for which of the following purposes

(a) Uplinking
(b) Downlinking
(c) Both (a) and (b)
(d) None of the above

Ans. (c)

59. The format specifier "-%d" is used for which purpose in C

(a) Left justifying a string


(b) Right justifying a string
(c) Removing a string from the console
(d) Used for the scope specification of a char[] variable

Ans. (a)
60. Virtual functions allow you to

(a) Create an array of type pointer-to-base-class that can hold pointers to derived classes
(b) Create functions that have no body
(c) Group objects of different classes so they can all be accessed by the same function code
(d) Use the same function call to execute member functions to objects from different classes

62. A sorting algorithm which can prove to be a best time algorithm in one case
and a worst time algorithm in worst case is

(a) Quick Sort


(b) Heap Sort
(c) Merge Sort
(d) Insert Sort

Ans. (a)

63. What details should never be found in the top level of a top-down design?

(a) Details
(b) Coding
(c) Decisions
(d) None of the above

Ans. (c)

64. In an absolute loading scheme, which loader function is accomplished by assembler

(a) Reallocation
(b) Allocation
(c) Linking
(d) Both (a) and (b)

Ans. (d)

65. Banker's algorithm for resource allocation deals with

(a) Deadlock prevention


(b) Deadlock avoidance
(c) Deadlock recovery
(d) None of these

Ans. (b)

66. Thrashing can be avoided if

(a) The pages, belonging to the working set of the programs, are in main memory
(b) The speed of CPU is increased
(c) The speed of I/O processor are increased
(d) All of the above

Ans. (a)

67. Which of the following communications lines is best suited to interactive processing
applications?

(a) Narrowband channels


(b) Simplex channels
(c) Full-duplex channels
(d) Mixedband channels
Ans. (b)

68. A feasibility document should contain all of the following except

(a) Project name


(b) Problem descriptions
(c) Feasible alternative
(d) Data flow diagrams

Ans. (d)

69. What is the main function of a data link content monitor?

(a) To detect problems in protocols


(b) To determine the type of transmission used in a data link
(c) To determine the type of switching used in a data link
(d) To determine the flow of data

Ans. (a)

70. Which of the following is a broadband communications channel?

(a) Coaxial cable


(b) Fiber optic cable
(c) Microwave circuits
(d) All of the above

Ans. (d)

71. Which of the following memories has the shortest access time?
(a) Cache memory
(b) Magnetic bubble memory
(c) Magnetic core memory
(d) RAM

Ans. (a)

72. A shift register can be used for

(a) Parallel to serial conversion


(b) Serial to parallel conversion
(c) Digital delay line
(d) All the above

Ans. (d)

73. In which of the following page replacement policies, Balady's anomaly occurs?

(a) FIFO
(b) LRU
(c) LFU
(d) NRU

Ans. (a)

74. Subschema can be used to

(a) Create very different, personalised views of the same data


(b) Present information in different formats
(c) Hide sensitive information by omitting fields from the sub-schema's description
(d) All of the above

Ans. (d)

Вам также может понравиться